You are on page 1of 156

iế n

K

-T
Tuyển tập đề thi vào lớp 10 chuyên năm 2018 - 2022
A1
Nguyễn Tiến Lâm, Trịnh Huy Vũ

Ngày 14 tháng 7 năm 2022


á n
To
bộ
c
lạ
u

2 Tuyển tập đề thi vào lớp 10 chuyên Toán

iế n
K

-T
A1

To
bộ
c
lạ
u

Mục lục

iế n
K
1 Đề thi 7
1.1 Đề thi tuyển sinh vào lớp 10 trường THPT chuyên Khoa học Tự nhiên năm
2018, vòng 1 . . . . . . . . . . . . . . . . . . . . . . . . . . . . . . . . . . . 8


1.2 Đề thi tuyển sinh vào lớp 10 trường THPT chuyên Khoa học Tự nhiên năm

-T
2018, vòng 2 . . . . . . . . . . . . . . . . . . . . . . . . . . . . . . . . . . . 9
1.3 Đề thi tuyển sinh vào lớp 10 chuyên Toán Hà Nội, năm 2018 . . . . . . . . 10
1.4 Đề thi tuyển sinh vào lớp 10 trường THPT chuyên Khoa học Tự nhiên năm
2019, vòng 1 . . . . . . . . . . . . . . . . . . . . . . . . . . . . . . . . . . . 11
A1
1.5 Đề thi tuyển sinh vào lớp 10 trường THPT chuyên Khoa học Tự nhiên năm
2019, vòng 2 . . . . . . . . . . . . . . . . . . . . . . . . . . . . . . . . . . . 12
1.6 Đề thi tuyển sinh vào lớp 10 trường THPT chuyên Sư phạm Hà Nội năm
2019, vòng 2 . . . . . . . . . . . . . . . . . . . . . . . . . . . . . . . . . . . 13
n

1.7 Đề thi tuyển sinh vào lớp 10 chuyên Toán Hà Nội, năm 2019 . . . . . . . . 14
á

1.8 Đề thi tuyển sinh vào lớp 10 trường THPT chuyên Khoa học Tự nhiên năm
2020, vòng 1 . . . . . . . . . . . . . . . . . . . . . . . . . . . . . . . . . . . 15
To

1.9 Đề thi tuyển sinh vào lớp 10 trường THPT chuyên Khoa học Tự nhiên năm
2020, vòng 2 . . . . . . . . . . . . . . . . . . . . . . . . . . . . . . . . . . . 16
1.10 Đề thi tuyển sinh vào lớp 10 trường THPT chuyên Sư phạm Hà Nội năm
bộ

2020, vòng 2 . . . . . . . . . . . . . . . . . . . . . . . . . . . . . . . . . . . 17
1.11 Đề thi tuyển sinh vào lớp 10 chuyên Toán Hà Nội, năm 2020 . . . . . . . . 18
1.12 Đề thi tuyển sinh vào lớp 10 chuyên Tin Hà Nội, năm 2020 . . . . . . . . . 19
1.13 Đề thi tuyển sinh vào lớp 10 trường Phổ thông Năng Khiếu ĐHQG TPHCM
c

năm 2020, vòng 2 . . . . . . . . . . . . . . . . . . . . . . . . . . . . . . . . 20


lạ

1.14 Đề thi tuyển sinh vào lớp 10 chuyên Toán Thành phố Hồ Chí Minh, năm
2020 . . . . . . . . . . . . . . . . . . . . . . . . . . . . . . . . . . . . . . . 21
1.15 Đề thi tuyển sinh vào lớp 10 trường THPT chuyên Khoa học Tự nhiên năm
u

2021, vòng 1 . . . . . . . . . . . . . . . . . . . . . . . . . . . . . . . . . . . 22

1.16 Đề thi tuyển sinh vào lớp 10 trường THPT chuyên Khoa học Tự nhiên năm
2021, vòng 2 . . . . . . . . . . . . . . . . . . . . . . . . . . . . . . . . . . . 23
1.17 Đề thi tuyển sinh vào lớp 10 trường THPT chuyên Sư phạm Hà Nội năm
2021, vòng 2 . . . . . . . . . . . . . . . . . . . . . . . . . . . . . . . . . . . 24
1.18 Đề thi tuyển sinh vào lớp 10 chuyên Toán Hà Nội, năm 2021 . . . . . . . . 25
1.19 Đề thi tuyển sinh vào lớp 10 chuyên Tin Hà Nội, năm 2021 . . . . . . . . . 26
1.20 Đề thi tuyển sinh vào lớp 10 trường THPT chuyên Khoa học Tự nhiên năm
2022, vòng 1 . . . . . . . . . . . . . . . . . . . . . . . . . . . . . . . . . . . 27

3
4 Tuyển tập đề thi vào lớp 10 chuyên Toán

1.21 Đề thi tuyển sinh vào lớp 10 trường THPT chuyên Khoa học Tự nhiên năm
2022, vòng 2 . . . . . . . . . . . . . . . . . . . . . . . . . . . . . . . . . . . 28
1.22 Đề thi tuyển sinh vào lớp 10 trường THPT chuyên Sư phạm Hà Nội năm
2022, vòng 2 . . . . . . . . . . . . . . . . . . . . . . . . . . . . . . . . . . . 29
1.23 Đề thi tuyển sinh vào lớp 10 chuyên Toán Hà Nội, năm 2022 . . . . . . . . 30
1.24 Đề thi tuyển sinh vào lớp 10 chuyên Tin Hà Nội, năm 2022 . . . . . . . . . 31
1.25 Đề thi tuyển sinh vào lớp 10 chuyên Toán Thành phố Hồ Chí Minh, năm

iế n
2022 . . . . . . . . . . . . . . . . . . . . . . . . . . . . . . . . . . . . . . . 32

2 Lời giải 33

K
2.1 Đề thi tuyển sinh vào lớp 10 trường THPT chuyên Khoa học Tự nhiên năm
2018, vòng 1 . . . . . . . . . . . . . . . . . . . . . . . . . . . . . . . . . . . 34
2.2 Đề thi tuyển sinh vào lớp 10 trường THPT chuyên Khoa học Tự nhiên năm


2018, vòng 2 . . . . . . . . . . . . . . . . . . . . . . . . . . . . . . . . . . . 39

-T
2.3 Đề thi tuyển sinh vào lớp 10 chuyên Toán Hà Nội, năm 2018 . . . . . . . . 44
2.4 Đề thi tuyển sinh vào lớp 10 trường THPT chuyên Khoa học Tự nhiên năm
2019, vòng 1 . . . . . . . . . . . . . . . . . . . . . . . . . . . . . . . . . . . 49
2.5 Đề thi tuyển sinh vào lớp 10 trường THPT chuyên Khoa học Tự nhiên năm
2019, vòng 2 . . . . . . . . . . . . . . . . . . . . . . . . . . . . . . . . . . . 54
A1
2.6 Đề thi tuyển sinh vào lớp 10 trường THPT chuyên Sư phạm Hà Nội năm
2019, vòng 2 . . . . . . . . . . . . . . . . . . . . . . . . . . . . . . . . . . . 59
2.7 Đề thi tuyển sinh vào lớp 10 chuyên Toán Hà Nội, năm 2019 . . . . . . . . 63
2.8 Đề thi tuyển sinh vào lớp 10 trường THPT chuyên Khoa học Tự nhiên năm
n

2020, vòng 1 . . . . . . . . . . . . . . . . . . . . . . . . . . . . . . . . . . . 68
á

2.9 Đề thi tuyển sinh vào lớp 10 trường THPT chuyên Khoa học Tự nhiên năm
To

2020, vòng 1 . . . . . . . . . . . . . . . . . . . . . . . . . . . . . . . . . . . 73
2.10 Đề thi tuyển sinh vào lớp 10 trường THPT chuyên Sư phạm Hà Nội năm
2020, vòng 2 . . . . . . . . . . . . . . . . . . . . . . . . . . . . . . . . . . . 78
2.11 Đề thi tuyển sinh vào lớp 10 chuyên Toán Hà Nội, năm 2020 . . . . . . . . 82
bộ

2.12 Đề thi tuyển sinh vào lớp 10 chuyên Tin Hà Nội, năm 2020 . . . . . . . . . 89
2.13 Đề thi tuyển sinh vào lớp 10 trường Phổ thông Năng Khiếu ĐHQG TPHCM
năm 2020, vòng 2 . . . . . . . . . . . . . . . . . . . . . . . . . . . . . . . . 94
2.14 Đề thi tuyển sinh vào lớp 10 chuyên Toán Thành phố Hồ Chí Minh, năm
c

2020 . . . . . . . . . . . . . . . . . . . . . . . . . . . . . . . . . . . . . . . 101
lạ

2.15 Đề thi tuyển sinh vào lớp 10 trường THPT chuyên Khoa học Tự nhiên năm
2021, vòng 1 . . . . . . . . . . . . . . . . . . . . . . . . . . . . . . . . . . . 106
u

2.16 Đề thi tuyển sinh vào lớp 10 trường THPT chuyên Khoa học Tự nhiên năm
2021, vòng 2 . . . . . . . . . . . . . . . . . . . . . . . . . . . . . . . . . . . 109

2.17 Đề thi tuyển sinh vào lớp 10 trường THPT chuyên Sư phạm Hà Nội năm
2021, vòng 2 . . . . . . . . . . . . . . . . . . . . . . . . . . . . . . . . . . . 115
2.18 Đề thi tuyển sinh vào lớp 10 chuyên Toán Hà Nội, năm 2021 . . . . . . . . 119
2.19 Đề thi tuyển sinh vào lớp 10 chuyên Tin Hà Nội, năm 2021 . . . . . . . . . 124
2.20 Đề thi tuyển sinh vào lớp 10 trường THPT chuyên Khoa học Tự nhiên năm
2022, vòng 1 . . . . . . . . . . . . . . . . . . . . . . . . . . . . . . . . . . . 129
2.21 Đề thi tuyển sinh vào lớp 10 trường THPT chuyên Khoa học Tự nhiên năm
2022, vòng 2 . . . . . . . . . . . . . . . . . . . . . . . . . . . . . . . . . . . 133
Nguyễn Tiến Lâm - Trịnh Huy Vũ 5

2.22 Đề thi tuyển sinh vào lớp 10 trường THPT chuyên Sư phạm Hà Nội năm
2022, vòng 2 . . . . . . . . . . . . . . . . . . . . . . . . . . . . . . . . . . . 138
2.23 Đề thi tuyển sinh vào lớp 10 chuyên Toán Hà Nội, năm 2022 . . . . . . . . 142
2.24 Đề thi tuyển sinh vào lớp 10 chuyên Tin Hà Nội, năm 2022 . . . . . . . . . 147
2.25 Đề thi tuyển sinh vào lớp 10 chuyên Toán Thành phố Hồ Chí Minh, năm
2022 . . . . . . . . . . . . . . . . . . . . . . . . . . . . . . . . . . . . . . . 152

iế n
K

-T
A1
á n
To
bộ
c
lạ
u


u
lạ
c
bộ
To
án
A1
-T

K
iế n

u

Đề thi
lạ
c Chương 1
bộ
To

7
án
A1
-T

K
iế n
8 Tuyển tập đề thi vào lớp 10 chuyên Toán

1.1. Đề thi tuyển sinh vào lớp 10 trường THPT


chuyên Khoa học Tự nhiên năm 2018, vòng 1
Câu 1.
1. Giải phương trình √ √
x2 − x + 2 x3 + 1 = 2 x + 1.

iế n
2. Giải hệ phương trình (
xy + y 2 = 1 + y

K
x2 + 2y 2 + 2xy = 4 + x

Câu 2.


1. Tìm tất cả các cặp số nguyên (x, y) thỏa mãn

-T
(x + y)(3x + 2y)2 = 2x + y − 1.
r
√ b
2. Với a, b là các số thực dương thỏa mãn a + 2b = 2 + , tìm giá trị nhỏ nhất của
A1 3
biểu thức
a b
M=√ +√ .
a + 2b b + 2a
n

Câu 3. Cho tam giác ABC có đường tròn nội tiếp (I) tiếp xúc với các cạnh BC, CA, AB
á

lần lượt tại các điểm D, E, F . Gọi K là hình chiếu vuông góc của B trên đường thẳng
To

DE, M là trung điểm của đoạn thẳng DF .

1. Chứng minh rằng hai tam giác BKM và DEF đồng dạng.

2. Gọi L là hình chiếu vuông góc của C trên đường thẳng DF , N là trung điểm của
bộ

đoạn thẳng DE. Chứng minh rằng hai đường thẳng M K và N L song song.

3. Gọi J, X lần lượt là trung điểm của các đoạn thẳng KL và ID. Chứng minh rằng
đường thẳng JX vuông góc với đường thẳng EF .
c
lạ

Câu 4. Trên mặt phẳng cho hai điểm P, Q phân biệt. Xét 10 đường thẳng nằm trong
mặt phẳng trên thỏa mãn tính chất sau:
u

i) không có hai đường thẳng nào song song hoặc trùng nhau.

ii) mỗi đường thẳng đều đi qua P hoặc Q, không có đường thẳng nào đi qua cả P và
Q.

Hỏi 10 đường thẳng trên có thể chia mặt phẳng thành tối đa bao nhiêu miền? Hãy giải
thích.
Nguyễn Tiến Lâm - Trịnh Huy Vũ 9

1.2. Đề thi tuyển sinh vào lớp 10 trường THPT


chuyên Khoa học Tự nhiên năm 2018, vòng 2
Câu 1.
1. Giải hệ phương trình

iế n
(
xy(x + y) = 2
x3 + y 3 + x3 y 3 + 7(x + 1)(y + 1) = 31

K
2. Giải phương trình p √ √
9+3 x(3 − 2x) = 7 x + 5 3 − 2x.


Câu 2.

-T
1. Cho x, y là các số nguyên sao cho x2 − 2xy − y và xy − 2y 2 − x đều chia hết cho 5.
Chứng minh rằng 2x2 + y 2 + 2x + y cũng chia hết cho 5.

2. Cho a1 , a2 , · · · , a50 là các số nguyên thỏa mãn 1 ≤ a1 ≤ a2 ≤ · · · ≤ a50 ≤ 50 và


A1
a1 + a2 + · · · + a50 = 100. Chứng minh rằng từ các số đã cho có thể chọn được một
vài số có tổng bằng 50.

Câu 3. Cho ngũ giác lồi ABCDE nội tiếp đường tròn (O) có CD song song với BE.
Hai đường chéo CE và BD cắt nhau tại P . Điểm M thuộc đoạn thẳng BE sao cho
n

∠M AB = ∠P AE. Điểm K thuộc đường thẳng AC sao cho M K song song với AD, điểm
á

L thuộc đường thẳng AD sao cho M L song song với AC. Đường tròn ngoại tiếp tam giác
To

KBC lần lượt cắt BD và CE tại Q, S (Q khác B, S khác C)

1. Chứng minh rằng ba điểm K, M, Q thẳng hàng.

2. Đường tròn ngoại tiếp tam giác LDE lần lượt cắt BD, CE tại T, R (T khác D, R
bộ

khác E). Chứng minh rằng năm điểm M, S, Q, R, T cùng thuộc một đường tròn.

3. Chứng minh rằng đường tròn ngoại tiếp của tam giác P QR tiếp xúc với đường tròn
(O).
c
lạ

Câu 4. Cho a, b, c là các số thực dương. Chứng minh rằng


r r ! 
ab bc 1 1
+ √ +√ ≤ 2.
u

a+b b+c a+b b+c



10 Tuyển tập đề thi vào lớp 10 chuyên Toán

1.3. Đề thi tuyển sinh vào lớp 10 chuyên Toán Hà


Nội, năm 2018
Câu 1.

1. Giải phương trình: x2 + 3x + 8 = (x + 5) x2 + x + 2.
(

iế n
y 2 − 2xy = 8x2 − 6x + 1
2. Giải hệ phương trình
y 2 = x3 + 8x2 − x + 1
Câu 2.

K
1. Cho p, q là hai số nguyên tố lớn hơn 5. Chứng minh rằng p4 + 2019q 4 chia hết cho
20.


√ √
2. Cho các số nguyên dương a, b, c, d thỏa mãn a < b ≤ c < d, ad = bc và d − a ≤ 1.

-T
a. Chứng minh rằng a + d > b + c.
b. Chứng minh rằng a là một số chính phương.
Câu 3.
A1
1. Với x, y, z là các số thực dương thỏa mãn xyz = 1. Chứng minh rằng
1 1 1
+ + = 1.
xy + x + 1 yz + y + 1 zx + z + 1
n

1 1 1
2. Với x, y, z là các số thự dương thay đổi và thỏa mãn + + = 3, tìm giá trị lớn
á

x y z
To

nhất của biểu thức


1 1 1
P =p +p +√ .
2x2 + y 2 + 3 2y 2 + z 2 + 3 2z 2 + x2 + 3
bộ

Câu 4. Cho tứ giác ABCD (không có hai cạnh nào song song) nội tiếp đường tròn (O).
Các tia BA và CD cắt nhau tại điểm F . Gọi E là giao điểm của hai đường chéo AC và
BD. Vẽ hình bình hành AEDK.
c

1. Chứng minh rằng tam giác F KD đồng dạng với tam giác F EB.
lạ

2. Gọi M, N tương ứng là trung điểm của các cạnh AD, BC. Chứng minh rằng đường
thẳng M N đi qua trung điểm của đoạn thẳng EF .
u

3. Chứng minh rằng đường thẳng EF tiếp xúc với đường tròn ngoại tiếp của tam giác
EM N .

Câu 5. Cho tập hợp S = {x ∈ Z | 1 ≤ x ≤ 50}. Xét A là một tập con bất kỳ của tập
hợp S và có tính chất: Không có ba phần tử nào của tập hợp A là số đo độ dài ba cạnh
của một tam giác vuông.
1. Tìm một tập hợp A có đúng 40 phần tử và thỏa mãn điều kiện đề bài.
2. Có hay không có một tập hợp A có đúng 41 phần tử và thỏa mãn điều kiện của đề
bài? Hãy giải thích câu trả lời.
Nguyễn Tiến Lâm - Trịnh Huy Vũ 11

1.4. Đề thi tuyển sinh vào lớp 10 trường THPT


chuyên Khoa học Tự nhiên năm 2019, vòng 1
Câu 1.
1. Giải phương trình
26x + 5 √ √
√ + 2 26x + 5 = 3 x2 + 30.

iế n
x2 + 30

2. Giải hệ phương trình

K
(
x2 + y 2 = 2
(x + 2y)(2 + 3y 2 + 4xy) = 27


-T
Câu 2.
1. Tìm tất cả các cặp (x, y) nguyên thỏa mãn

(x2 − x + 1)(y 2 + xy) = 3x − 1.


A1
2. Với x, y là các số thực thỏa mãn 1 ≤ y ≤ 2, xy + 2 ≥ 2y, tìm giá trị nhỏ nhất của
biểu thức
x2 + 4
M= 2 .
y +1
á n

Câu 3. Cho hình vuông ABCD, đường tròn (O) nội tiếp hình vuông ABCD tiếp xúc
với các cạnh AB, AD lần lượt tại các điểm E, F . Gọi giao điểm của CE và BF là G.
To

1. Chứng minh rằng năm điểm A, F, O, G, E cùng nằm trên một đường tròn.

2. Gọi giao điểm của F B và đường tròn (O) là M (M 6= F ). Chứng minh rằng M là
bộ

trung điểm của đoạn thẳng BG.

3. Chứng minh rằng trực tâm của tam giác GAF nằm trên đường tròn (O).
c

Câu 4. Cho x, y, z là các số thực dương thỏa mãn xy + yz + xz = 1. Chứng minh rằng:
lạ

!3
1 1 1 2 x y z
+ + ≥ √ +p +√ .
1 + x2 1 + y 2 1 + z 2 3 1 + x2 1 + y2 1 + z2
u

12 Tuyển tập đề thi vào lớp 10 chuyên Toán

1.5. Đề thi tuyển sinh vào lớp 10 trường THPT


chuyên Khoa học Tự nhiên năm 2019, vòng 2
Câu 1.
1. Giải hệ phương trình (
3x2 + y 2 + 4xy = 8

iế n
(x + y)(x2 + xy + 2) = 8

2. Giải phương trình √ √

K
27 + x2 + x 27 + 2x
p = √ .
2 + 5 − (x2 + x) 2 + 5 − 2x


Câu 2.

-T
1. Chứng minh rằng với mọi số nguyên dương n, ta luôn có
7 7 7
(27n + 5)7 + 10 + (10n + 27)7 + 5 + (5n + 10)7 + 27
A1
chia hết cho 42.

2. Với x, y là các số thực dương thỏa mãn điều kiện 4x2 + 4y 2 + 17xy + 5x + 5y ≥ 1,
tìm giá trị nhỏ nhất của biểu thức
n

P = 17x2 + 17y 2 + 16xy.


á

Câu 3. Cho tam giác ABC cân tại A, có đường tròn nội tiếp (I). Các điểm E, F theo
To

thứ tự thuộc các cạnh CA, AB (E khác C và A; F khác B và A) sao cho EF tiếp xúc
với đường tròn (I) tại điểm P . Gọi K, L lần lượt là hình chiếu vuông góc của E, F trên
BC. Giả sử F K cắt EL tại điểm J. Gọi H là hình chiếu vuông góc của J trên BC.
bộ

1. Chứng minh rằng HJ là phân giác của ∠EHF .

2. Ký hiệu S1 và S2 lần lượt là diện tích của các tứ giác BF JL và CEJK. Chứng
minh rằng
c

S1 BF 2
lạ

= .
S2 CE 2

3. Gọi D là trung điểm của cạnh BC. Chứng minh rằng ba điểm P, J, D thẳng hàng.
u

Câu 4. Cho M là tập tất cả 4039 số nguyên liên tiếp từ −2019 đến 2019. Chứng minh

rằng trong 2021 số đôi một phân biệt được chọn bất kỳ từ tập M luôn tồn tại 3 số đôi
một phân biệt có tổng bằng 0.
Nguyễn Tiến Lâm - Trịnh Huy Vũ 13

1.6. Đề thi tuyển sinh vào lớp 10 trường THPT


chuyên Sư phạm Hà Nội năm 2019, vòng 2
Câu 1. Cho hai số thực phân biệt a, b thỏa mãn a3 + b3 = a2 b2 (ab − 3). Tính giá trị của
biểu thức T = a + b − ab.

Câu 2. Cho các đa thức P (x) = m1 x2 + n1 x + k1 , Q(x) = m2 x2 + n2 x + k2 , R(x) =

iế n
m3 x2 + n3 x + k3 với mi , ni , ki là các số thực và mi > 0, i = 1, 2, 3. Giả sử phương trình
P (x) = 0 có hai nghiệm phân biệt a1 , a2 , phương trình Q(x) = 0 có hai nghiệm phân biệt
b1 , b2 , phương trình R(x) = 0 có hai nghiệm phân biệt c1 , c2 thỏa mãn

K
P (c1 ) + Q(c1 ) = P (c2 ) + Q(c2 )
P (b1 ) + R(b1 ) = P (b2 ) + R(b2 )


Q(a1 ) + R(a1 ) = Q(a2 ) + R(a2 )

-T
Chứng minh rằng a1 + a2 = b1 + b2 = c1 + c2 .

Câu 3.
1. Tìm các số nguyên x, y thỏa mãn x2 y 2 − 4x2 y + y 3 + 4x2 − 3y 2 + 1 = 0.
A1
2. Cho ba số nguyên dương a, b, c thỏa mãn a3 + b3 + c3 chia hết cho 14. Chứng minh
rằng abc chia hết cho 14.

Câu 4. Cho tam giác ABC có ba góc nhọn nội tiếp đường tròn (O) và AB > AC. Gọi
n

D, E lần lượt là chân các đường cao của tam giác ABC hạ từ A, B. Gọi F là chân đường
á

vuông góc hạ từ B lên đường thẳng AO.


To

1. Chứng minh rằng B, D, E, F là bốn đỉnh của một hình thang cân.

2. Chứng minh rằng EF đi qua trung điểm của BC.


bộ

3. Gọi P là giao điểm thứ hai của đường thẳng AO và đường tròn (O), M và N lần
lượt là trung điểm của EF và CP . Tính góc ∠BM N .

Câu 5. Cho tập X thỏa mãn tính chất sau: tồn tại 2019 tập con A1 , A2 , · · · , A2019 của
c

X sao cho mỗi tập con A1 , A2 , · · · , A2019 có đúng ba phần tử và hai tập con Ai , Aj đều
lạ

có đúng một phần tử chung với mọi 1 ≤ i < j ≤ 2019.

1. Chứng minh rằng tồn tại 4 tập hợp trong các tập A1 , A2 , · · · , A2019 sao cho giao của
u

4 tập hợp này có đúng một phần tử.


2. Chứng minh rằng số phần tử của X phải lớn hơn hoặc bằng 4039.
14 Tuyển tập đề thi vào lớp 10 chuyên Toán

1.7. Đề thi tuyển sinh vào lớp 10 chuyên Toán Hà


Nội, năm 2019
Câu 1.
√ √ √
1. Giải phương trình ( x + 5 − x)(1 + x2 + 5x) = 5.
(

iế n
x2 + 7 = 4y 2 + 4y
2. Giải hệ phương trình
x2 + 3xy + 2y 2 + x + y = 0.

K
Câu 2.
1. Cho biểu thức P = abc(a − 1)(b + 4)(c + 6) với a, b, c là các số nguyên thỏa mãn
a + b + c = 2019. Chứng minh rằng giá trị của biểu thức P chia hết cho 6.


-T
2. Tìm tất cả các số tự nhiên n sao cho giá trị của biểu thức
√ √
q
Q= n+2+ n+ n+2

là số nguyên.
A1
Câu 3. Cho biểu thức K = ab + 4ac − 4bc, với a, b, c là các số thực không âm thỏa mãn
a + b + 2c = 1.
1
1. Chứng minh K ≥ − .
n

2
á

2. Tìm giá trị lớn nhất của biểu thức K.


To

Câu 4. Cho tam giác ABC có ba góc nhọn (AB < AC), nội tiếp đường tròn (O). Gọi
I là tâm đường tròn nội tiếp tam giác ABC. Tia AI cắt đoạn thẳng BC tại điểm J, cắt
đường tròn (O) tại điểm thứ hai M (M khác A).
bộ

1. Chứng minh rằng M I 2 = M J · M A.


2. Kẻ đường kính M N của đường tròn (O). Đường thẳng AN cắt các tia phân giác
trong của góc ABC và góc ACB lần lượt tại các điểm P và Q. Chứng minh rằng
c

N là trung điểm của đoạn thẳng P Q.


lạ

3. Lấy điểm E bất kỳ thuộc cung nhỏ M C của đường tròn (O) (E khác M ). Gọi F
là điểm đối xứng với điểm I qua điểm E. Gọi R là giao điểm của hai đường thẳng
u

P C và QB. Chứng minh rằng bốn điểm P, Q, R, F cùng nằm trên một đường tròn.

Câu 5. Mỗi điểm trong mặt phẳng được tô bởi một trong hai màu xanh hoặc đỏ.
1. Chứng minh trong mặt phẳng đó tồn tại hai điểm được tô bởi cùng một màu và có
khoảng cách bằng d.
2. Gọi tam giác có ba đỉnh được tô cùng một màu là tam giác đơn sắc. Chứng minh
trong mặt phẳng đó tồn tại hai tam giác đơn sắc là hai tam giác vuông đồng dạng
1
với nhau theo tỉ số k = .
2019
Nguyễn Tiến Lâm - Trịnh Huy Vũ 15

1.8. Đề thi tuyển sinh vào lớp 10 trường THPT


chuyên Khoa học Tự nhiên năm 2020, vòng 1
Câu 1.
1. Giải hệ phương trình (
x2 + y 2 + xy = 7

iế n
9x3 = xy 2 + 70(x − y).

2. Giải phương trình

K
√ √ p
11 5 − x + 8 2x − 1 = 24 + 3 (5 − x)(2x − 1).


Câu 2.

-T
1. Tìm x, y nguyên dương thỏa mãn

x2 y 2 − 16xy + 99 = 9x2 + 36y 2 + 13x + 26y.

2. Với a, b là các số thực dương thỏa mãn


A1
2 ≤ 2a + 3b ≤ 5, 8a + 12b ≤ 2a2 + 3b2 + 5ab + 10.

Chứng minh rằng


n

3a2 + 8b2 + 10ab ≤ 21.


á

Câu 3. Cho tam giác ABC có ∠BAC là góc nhỏ nhất trong ba góc của tam giác và nội
To

tiếp đường tròn (O). Điểm D thuộc cạnh BC sao cho AD là phân giác của ∠BAC. Lấy
các điểm M, N thuộc (O) sao cho các đường thẳng CM và BN cùng song song với đường
thẳng AD.
bộ

1. Chứng minh rằng AM = AN .

2. Gọi giao điểm của đường thẳng M N với các đường thẳng AC, AB lần lượt là E, F .
Chứng minh rằng bốn điểm B, C, E, F cùng thuộc một đường tròn.
c
lạ

3. Gọi P, Q theo thứ tự là trung điểm của các đoạn thẳng AM, AN . Chứng minh rằng
EQ, F P, AD đồng quy.
u

Câu 4. Cho a, b, c là các số thực dương thỏa mãn a + b + c = 3. Chứng minh rằng

a(a + bc)2 b(b + ca)2 c(c + ab)2


+ + ≥ 4.
b(ab + 2c2 ) c(bc + 2a2 ) a(ca + 2b2 )
16 Tuyển tập đề thi vào lớp 10 chuyên Toán

1.9. Đề thi tuyển sinh vào lớp 10 trường THPT


chuyên Khoa học Tự nhiên năm 2020, vòng 2
Câu 1.
1. Giải hệ phương trình

iế n
(
(x + y)(x + 1) = 4
.
(y 2 + xy + x + y + 5)(x3 + y 3 + 12y + 13) = 243.

K
2. Giải phương trình

(x − 12)7 + (2x − 12)7 + (24 − 3x)7 = 0.


-T
Câu 2.
1. Tìm tất cả các số nguyên dương a, b, c sao cho cả ba số 4a2 + 5b, 4b2 + 5c, 4c2 + 5a
đều là bình phương của số nguyên dương. A1
2. Từ một bộ bốn số thực (a, b, c, d) ta xây dựng bộ số mới (a + b, b + c, c + d, d + a)
và liên tiếp xây dựng các bộ số mới theo quy tắc trên. Chứng minh rằng nếu có hai
thời điểm khác nhau ta thu được cùng một bộ số (có thể khác thứ tự) thì bộ số ban
đầu phải có dạng (a, −a, a, −a).
n

Câu 3. Cho tam giác ABC cân tại A với ∠BAC < 90◦ . Điểm E thuộc cạnh AC sao cho
á

∠AEB > 90◦ . Gọi P là giao điểm của BE và trung trực của BC. Gọi K là hình chiếu
To

vuông góc của P lên AB. Gọi Q là hình chiếu vuông góc của E lên AP . Gọi giao điểm
của EQ và P K là F .

1. Chứng minh rằng bốn điểm A, E, P, F cùng thuộc một đường tròn.
bộ

2. Gọi giao điểm của KQ và P E là L. Chứng minh rằng LA vuông góc với LE.

3. Gọi giao điểm của F L và AB là S. Gọi giao điểm của KE và AL là T . Lấy R là


điểm đối xứng của A qua L. Chứng minh rằng đường tròn ngoại tiếp tam giác AST
c

và đường tròn ngoại tiếp tam giác BP R tiếp xúc với nhau.
lạ

Câu 4. Cho a, b, c là các số thực dương thỏa mãn a + b + c = 3. Chứng minh rằng
2
u

  
1 1 1 4 a b c
3 + + −1 +1≥ +3 + + .
a b c abc bc ca ab

Nguyễn Tiến Lâm - Trịnh Huy Vũ 17

1.10. Đề thi tuyển sinh vào lớp 10 trường THPT


chuyên Sư phạm Hà Nội năm 2020, vòng 2
Câu 1. Cho ba số thực x, y, z thỏa mãn các điều kiện sau

3
2x = 3y 3 = 4z 3
√ √

iế n
p3
2x2 + 3y 2 + 4z 2 = 2 + 3 12 + 3 16

xyz > 0.

1 1 1

K
Tính giá trị của biểu thức P = + + .
x y z

Câu 2. Xét phương trình bậc hai ax2 + bx + c = 0 (1), trong đó a, b, c là các số nguyên


dương. Biết rằng các điều kiện sau được thỏa mãn: phương trình (1) có nghiệm, số a2020b

-T
chia hết cho 12, số c2 + 3 chia hết cho c + 3. Hãy tìm giá trị lớn nhất của tổng a + b + c.

Câu 3. Tìm số nguyên a bé nhất sao cho x4 + 2x2 − 4x + a ≥ 0 với mọi số thực x.

Câu 4. Cho tam giác nhọn ABC nội tiếp đường tròn (O) có AB > BC. Một đường tròn
A1
đi qua hai đỉnh A, C của tam giác ABC lần lượt cắt các cạnh AB, BC tại hai điểm K, N
(K, N khác các đỉnh của tam giác ABC.) Giả sử đường tròn (O) và đường tròn ngoại
tiếp của tam giác BKN cắt nhau tại giao điểm thứ hai là M (M khác B). Chứng minh
rằng
á n

1. Ba đường thẳng BM, KN, AC đồng quy tại điểm P .


To

2. Tứ giác M N CP nội tiếp.

3. BM 2 − P M 2 = BK · BA − P C · P A.
bộ

Câu 5. Cho hai số A, B cùng có 2020 chữ số. Biết rằng số A có đúng 1945 chữ số khác 0,
bao gồm 1930 chữ số ngoài cùng về bên trái và 15 chữ số ngoài cùng về bên phải, số B có
đúng 1954 chữ số khác 0, bao gồm 1930 chữ số ngoài cùng về bên trái và 24 chữ số ngoài
cùng về bên phải. Chứng minh rằng U CLN (A, B) là một số có không quá 1954 chữ số.
c
lạ
u

18 Tuyển tập đề thi vào lớp 10 chuyên Toán

1.11. Đề thi tuyển sinh vào lớp 10 chuyên Toán


Hà Nội, năm 2020
Câu 1.

1. Giải phương trình x2 + 3x + 5 = (x + 3) x2 + 5.

iế n
2. Cho các số thực a, b, c thỏa mãn a + b − 2c = 0 và 2ab − bc − ca = 0. Chứng minh
rằng a = b = c.

Câu 2.

K
1. Chứng minh với mọi số nguyên dương n, số A = 11n + 7n − 2n − 1 chia hết cho 15.
√ m
2. Cho hai số nguyên dương m và n thỏa mãn 11 − > 0. Chứng minh rằng


n

-T

√ m 3( 11 − 3)
11 − ≥ .
n mn
Câu 3.
A1
1. Cho đa thức P (x) với hệ số thực thỏa mãn P (1) = 3 và P (3) = 7. Tìm đa thức dư
trong phép chia đa thức P (x) cho đa thức x2 − 4x + 3.

2. Với a, b, c là các số thực không âm thỏa mãn a + b + c + abc = 4, tìm giá trị lớn nhất
n

của biểu thức P = ab + bc + ca.


á

Câu 4. Cho tam giác ABC có ba góc nhọn và AB < AC. Gọi (I) là đường tròn nội
To

tiếp của tam giác ABC và K là tâm đường tròn bàng tiếp trong góc A của tam giác
ABC. Gọi D, E, F lần lượt là chân các đường vuông góc kẻ từ điểm I đến các đường
thẳng BC, CA, AB. Đường thẳng AD cắt đường tròn (I) tại hai điểm phân biệt D và M .
Đường thẳng qua K song song với đường thẳng AD cắt đường thẳng BC tại N .
bộ

1. Chứng minh rằng tam giác M F D đồng dạng với tam giác BN K.

2. Gọi P là giao điểm của BI và F D. Chứng minh rằng ∠BM F = ∠DM P .


c

3. Chứng minh rằng đường tròn ngoại tiếp của tam giác M BC đi qua trung điểm của
lạ

đoạn thẳng KN .

Câu 5. Cho một bảng ô vuông kích thước 6 × 7 (6 hàng, 7 cột) được tạo bởi các ô vuông
u

có kích thước 1 × 1. Mỗi ô vuông có kích thước 1 × 1 được tô bởi một trong hai màu đen

hoặc trắng sao cho trong mọi bảng ô vuông kích thước 2 × 3 hoặc 3 × 2, có ít nhất hai ô
vuông kích thước 1 × 1 được tô màu đen có chung cạnh. Gọi m là số ô vuông kích thước
1 × 1 được tô màu đen trong bảng

1. Chỉ ra một cách tô sao cho m = 20.

2. Tìm giá trị nhỏ nhất của m.


Nguyễn Tiến Lâm - Trịnh Huy Vũ 19

1.12. Đề thi tuyển sinh vào lớp 10 chuyên Tin Hà


Nội, năm 2020
Câu 1.

1. Giải phương trình (x + 2) x2 + 1 = x2 + 2x + 1.

iế n
2. Chứng minh rằng
1 1 1 1
√ √ + √ √ + ··· + √ √ =1− √ .
2 1+1 2 3 2+2 3 2021 2020 + 2020 2021 2021

K
Câu 2.
1. Chứng minh rằng với mọi số nguyên dương n, số A = 59n − 17n − 9n + 2n chia hết


cho 35.

-T
2. Tìm tất cả các số nguyên x, y thỏa mãn điều kiện x2 y − 3y − 4x − 1 = 0.

Câu 3. A1
1. Tìm tất cả các số thực a, b, c thỏa mãn đồng thời các điều kiện a2 + b2 + c2 =
38, a + b = 8 và b + c ≥ 7.

2. Với a, b, c là các số thực không√âm và luôn thỏa mãn a2 + b2 + c2 = 2ab + 2bc + 2ca,
chứng minh rằng a + b + c ≥ 3 2abc.
á n

Câu 4. Cho tam giác ABC có ba góc nhọn, AB < AC và ba đường cao AD, BE, CF
cùng đi qua điểm H. Gọi (S) là đường tròn ngoại tiếp của tam giác DEF .
To

1. Chứng minh rằng đường tròn (S) đi qua trung điểm của đoạn thẳng AH.

2. Gọi M và N lần lượt là giao điểm của đường tròn (S) với các đoạn thẳng BH và
bộ

CH. Tiếp tuyến tại D của đường tròn (S) cắt M N tại điểm T . Chứng minh rằng
đường thẳng HT song song với đường thẳng EF .

3. Gọi P là giao điểm của hai đường thẳng BH và DF , Q là giao điểm của hai đường
c

thẳng CH và DE. Chứng minh rằng ba điểm T, P, Q là ba điểm thẳng hàng.


lạ

Câu 5. Trên bàn có 6 hộp kẹo, mỗi hộp có 5 viên kẹo. An và Bình cùng chơi một trò chơi
như sau: mỗi lượt chơi, An sẽ chọn một hộp tùy ý và lấy ít nhất 1 viên kẹo trong hộp đó;
u

còn Bình thì chọn một số hộp và trong các hộp đã chọn, mỗi hộp lấy đúng 1 viên kẹo.
Hai bạn luân phiên thực hiện lượt chơi của mình. Bạn đầu tiên không thể thực hiện được

lượt chơi của mình là người thua cuộc. Nếu An là người lấy kẹo trước, hãy chỉ ra chiến
thuật chơi để Bình là người thắng cuộc.
20 Tuyển tập đề thi vào lớp 10 chuyên Toán

1.13. Đề thi tuyển sinh vào lớp 10 trường Phổ


thông Năng Khiếu ĐHQG TPHCM năm 2020,
vòng 2
Câu 1. Cho các phương trình x2 + ax + 3 = 0 và x2 + bx + 5 = 0 với a, b là các tham số
(a) Chứng minh rằng nếu ab ≥ 16 thì trong hai phương trình có ít nhất một phương

iế n
trình có nghiệm.
(b) Giả sử hai phương trình trên có nghiệm chung x0 . Tìm a, b sao cho |a| + |b| có giá trị

K
nhỏ nhất.
Câu 2. Cho phương trình 3x2 − y 2 = 23n với n là số tự nhiên.


(a) Chứng minh rằng nếu n chẵn thì phương trình đã cho không có nghiệm nguyên (x, y).

-T
(b) Chứng minh rằng nếu n lẻ thì phương trình đã cho có nghiệm nguyên (x, y).
Câu 3. Cho đường tròn (O), dây cung BC không chứa tâm O và điểm A thay đổi trên
cung lớn BC. Lấy các điểm E, F thỏa mãn ∠ABE = ∠CAE = ∠ACF = ∠BAF = 90◦ .
(a) Chứng minh rằng AE · AC = AF · AB và O là trung điểm của EF .
A1
(b) Hạ AD vuông góc với EF (D ∈ EF ). Chứng minh rằng các tam giác DAB và DCA
đồng dạng và D thuộc một đường tròn cố định.
(c) Gọi G là giao điểm của AD và đường tròn (O) (G 6= A). Chứng minh rằng AD đi
n

qua một điểm cố định và GB · AC = GC · AB.


á

(d) Gọi K là tâm đường tròn ngoại tiếp của tam giác AEF . Chứng minh rằng AK đi
To

qua một điểm cố định.


Câu 4. Cho số tự nhiên a = 313 .57 .720 .
(a) Gọi A là tập hợp các số nguyên dương k sao cho k là ước của a và k chia hết cho 105.
bộ

Hỏi tập A có bao nhiêu phần tử?


(b) Giả sử B là một tập con bất kỳ của A có 9 phần tử. Chứng minh ta luôn có thể tìm
được 2 phần tử của B sao cho tích của chúng là số chính phương.
c

Câu 5. Cho hệ phương trình với k là tham số:


lạ

 r r
x x x
√ + + =k




 yz y z
u







r r
y y y

√ + + =k
 zx
 z x




 r r

 z z z
√ + + =k


xy x y
(a) Giải hệ với k = 1.
(b) Chứng minh hệ vô nghiệm với k ≥ 2 và k 6= 3.
Nguyễn Tiến Lâm - Trịnh Huy Vũ 21

1.14. Đề thi tuyển sinh vào lớp 10 chuyên Toán


Thành phố Hồ Chí Minh, năm 2020
a b c
Câu 1. Cho ba số dương a, b, c thỏa mãn điều kiện + + = 2020.
 2 b +c c + a a + b
2 2
a b c
Tính giá trị của biểu thức P = + + : (a + b + c).

iế n
b+c c+a a+b

Câu 2.
√ √
a) Giải phương trình: 2x2 + x + 9 + 2x2 − x + 1 = x + 4.

K
(
y 2 − 2xy = 8x2 − 6x + 1
b) Giải hệ phương trình
y 2 = x3 + 8x2 − x + 1.


-T
Câu 3. Cho tam giác nhọn ABC (AB < BC < CA) nội tiếp đường tròn (O). Từ A kẻ
đường thẳng song song với BC cắt (O) tại A1 . Từ B kẻ đường thẳng song song với AC
cắt (O) tại B1 . Từ C kẻ đường thẳng song song với AB cắt (O) tại C1 . Chứng minh rằng
các đường thẳng qua A1 , B1 , C1 lần lượt vuông góc với BC, CA, AB đồng quy.
A1
Câu 4.
a2 + b 2 (a − b)2
a) Cho 2 số thực a, b. Chứng minh rằng: ≥ ab + 2 .
2 a + b2 + 2
n

b) Cho 2 số dương a, b thỏa mãn điều kiện a + b ≤ 3.


á

20 7
Tìm giá trị nhỏ nhất của biểu thức: Q = b − a + + .
To

a b
Câu 5. Đường tròn (I) nội tiếp tam giác ABC tiếp xúc với các cạnh AB, BC, CA lần
lượt tại D, E, F . Kẻ đường kính EJ của đường tròn (I). Gọi d là đường thẳng qua A song
bộ

song với BC. Đường thẳng JD cắt d, BC lần lượt tại L, H.

(a) Chứng minh rằng E, F, L thẳng hàng.

(b) JA, JF cắt BC lần lượt tại M, K. Chứng minh rằng M H = M K.


c
lạ

Câu 6. Tìm tất cả các số nguyên dương x, y thỏa mãn phương trình 3x − y 3 = 1.
u

22 Tuyển tập đề thi vào lớp 10 chuyên Toán

1.15. Đề thi tuyển sinh vào lớp 10 trường THPT


chuyên Khoa học Tự nhiên năm 2021, vòng 1
Câu 1. (2,0 điểm) Giải phương trình
√ √ p
13 5 − x + 18 x + 8 = 61 + x + 3 (5 − x)(x + 8)

iế n
Câu 2. (2,0 điểm) Giải hệ phương trình
 4
x + y 4 + 6x2 y 2 = 1

K
x(x + y)4 = x − y

Câu 3. (2,0 điểm) Tìm số nguyên dương n nhỏ nhất biết rằng khi chia n cho 7, 9, 11, 13


ta nhận được các số dư là 3, 4, 5, 6.

-T
Câu 4. (3,0 điểm) Cho tam giác nhọn ABC có điểm P nằm trong tam giác (P không
nằm trên các cạnh). Gọi J, K, L lần lượt là tâm đường tròn nội tiếp của các tam giác
P BC, P CA, P AB. A1
1. Chứng minh rằng BJC
[ + CKA [ = 90◦ .
\ + ALB

2. Giả sử P B = P C và P C < P A. Gọi X, Y, Z lần lượt là hình chiếu vuông góc của
J, K, L trên các cạnh BC, CA, AB. Dựng hình bình hành XY W Z. Chứng minh
n
rằng W nằm trên phân giác BAC.
[
á

Câu 5. (1,0 điểm) Cho tập A = {1, 2, ..., 2021}. Tìm số nguyên dương k lớn nhất (k ≥ 2)
To

sao cho ta có thể chọn được k số phân biệt từ tập A mà tổng của hai số phân biệt bất kỳ
trong k số được chọn không chia hết cho hiệu của chúng.
bộ
c
lạ
u

Nguyễn Tiến Lâm - Trịnh Huy Vũ 23

1.16. Đề thi tuyển sinh vào lớp 10 trường THPT


chuyên Khoa học Tự nhiên năm 2021, vòng 2
Câu 1. (4,0 điểm)
1. Với a, b, c là các số thực thoả mãn a + b + c 6= 0 và (a + b)(b + c)(c + a) = 1, chứng
minh rằng

iế n
a b 1 + abc + ab(a + b + c)
+ 2 = .
a2 (a + b + c) + 1 + abc b (a + b + c) + 1 + abc (a + b + c)2

K
2. Giải hệ phương trình
(
x2 + 4y 2 + 4xy + 2x2 y 2 = 11


.
3xy(x + 2y) + 31 = 9x + 18y + 13xy.

-T
Câu 2. (2,0 điểm)
1. Tìm x, y nguyên dương thoả mãn 3x + 29 = 2y .
A1
2. Với a, b, c là các số thực dương thoả mãn điều kiện 2(a + b + c) + ab + bc + ca = 9,
tìm giá trị lớn nhất của biểu thức
a+1 b+1 c+1
M= + 2 + 2 .
n

a2 + 10a + 21 b + 10b + 21 c + 10c + 21


á

Câu 3. (3,0 điểm) Cho hình thoi ABCD (BAD \ < 90◦ ) có đường tròn nội tiếp (O). Các
To

điểm M, N lần lượt thuộc các cạnh CB, CD sao cho M N tiếp xúc (O) tại P , và tam
giác CM N nhọn không cân. Đường thẳng M N lần lượt cắt các đường thẳng AB, AD tại
E, F . Gọi K, L theo thứ tự là trực tâm của các tam giác BM E và DN F .
bộ

1. Chứng minh rằng OP đi qua trung điểm I của KL.


2. Gọi H là trực tâm của tam giác CM N . Chứng minh rằng
OI EF 1
c

− =− .
CH 2M N 2
lạ

3. Gọi giao điểm của EK, F L với BD lần lượt là S, T . Gọi giao điểm của N S và M T
là Q. Đường tròn nội tiếp của tam giác CM N tiếp xúc với M N tại G. Chứng minh
u

rằng hai đường thẳng P Q và GH song song.


Câu 4. (1,0 điểm) Giả sử a1 , a2 , ..., a2021 là các số thực thoả mãn
a1 a2 a2021
+ 2 + ··· + 2 = 0.
a21+ 1 a2 + 1 a2021 + 1
Chứng minh rằng tồn tại số nguyên k (1 ≤ k ≤ 2021) sao cho

a1 2a 2 ka k 2k + 1
+
a2 + 1 a2 + 1 + · · · + 2
≤ .
1 2 a k + 1 8
24 Tuyển tập đề thi vào lớp 10 chuyên Toán

1.17. Đề thi tuyển sinh vào lớp 10 trường THPT


chuyên Sư phạm Hà Nội năm 2021, vòng 2

1+ 5
Câu 1. (2,5 điểm) Cho α = .
2
a) Tìm một đa thức bậc hai Q(x) với hệ số nguyên sao cho α là nghiệm của Q(x).

iế n
b) Cho đa thức P (x) = x5 − x4 − x + 1. Tính giá trị của P (α).

Câu 2. (3,0 điểm) Cho A, B là hai điểm cố định nằm trên đường tròn tâm O, bán kính

K
R. Giả sử C là điểm cố định trên tia đối của tia BA. Một cát tuyến thay đổi qua C cắt
đường tròn (O) tại D và E (D nằm giữa C và E). Các đường tròn ngoại tiếp các tam


giác BCD và ACE cắt nhau tại giao điểm thứ hai M . Biết rằng bốn điểm O, B, M, E
tạo thành tứ giác OBM E. Chứng minh rằng

-T
a) Tứ giác OBM E nội tiếp.

b) CD · CE = CO2 − R2 . A1
c) M luôn di chuyển trên một đường tròn cố định.

Câu 3. (2,0 điểm) Tìm tất cả các số nguyên dương N sa cho N có thể biểu diễn một
x2 + y
cách duy nhất ở dạng với x, y là hai số nguyên dương.
n
xy + 1
á

Câu 4. (2,5 điểm) Cho a, b, c là ba số nguyên dương sao cho mỗi số đó đều biểu diễn được ở
To

dạng luỹ thừa của 2 với số mũ tự nhiên. Biết rằng phương trình bậc hai ax2 −bx+c = 0 (1)
có cả hai nghiệm đều là số nguyên. Chứng minh rằng hai nghiệm của phương trình (1)
bằng nhau.
bộ
c
lạ
u

Nguyễn Tiến Lâm - Trịnh Huy Vũ 25

1.18. Đề thi tuyển sinh vào lớp 10 chuyên Toán


Hà Nội, năm 2021
Câu 1.

1. Giải phương trình x2 + x + 2 − 2 x + 1 = 0.

iế n
2. Cho ba số thực a, b và c thoả mãn ab + bc + ca = 1. Chứng minh
a−b b−c c−a
2
+ 2
+ = 0.
1+c 1+a 1 + b2

K
Câu 2.
1. Tìm tất cả các cặp số nguyên (x, y) thoả mãn x2 + 5xy + 6y 2 + x + 2y − 2 = 0.


-T
2. Chứng minh rằng với mỗi số nguyên n, số n2 + n + 16 không chia hết cho 49.

Câu 3.
2
1. Cho số thực x khác 0 thoả mãn x + và x3 là số hữu tỉ. Chứng minh x là số hữu
x
tỉ.
A1
2. Cho các số thực không âm a, b và c thoả mãn a + b + c = 5. Chứng minh rằng
2a + 2ab + abc ≤ 18.
n

[ = 60◦ và AB < AC.


Câu 4. Cho tam giác nhọn ABC nội tiếp đường tròn (O), với BAC
á

Các đường thẳng BO, CO lần lượt cắt các đoạn thẳng AC, AB tại M, N . Gọi F là điểm
To

chính giữa của cung BC lớn.

(a) Chứng minh năm điểm A, N, O, M và F cùng thuộc một đường tròn.

(b) Gọi P, Q lần lượt là các giao điểm thứ hai của hai tia F N, F M với đường tròn (O).
bộ

Gọi J là giao điểm của đường thẳng BC và đường thẳng P Q. Chứng minh tia AJ là
tia phân giác của góc BAC.
[
c

(c) Gọi K là giao điểm của đường thẳng OJ và đường thẳng CF . Chứng minh AB vuông
góc với AK.
lạ

Câu 5. Cho A là một con có 100 phần tử của tập hợp {1, 2, 3, ..., 178}.
u

1. Chứng minh A chứa hai số tự nhiên liên tiếp.


2. Chứng minh với mọi số tự nhiên n thuộc tập hợp {2, 3, ..., 22}, tồn tại hai phần tử
của A có hiệu bằng n.
26 Tuyển tập đề thi vào lớp 10 chuyên Toán

1.19. Đề thi tuyển sinh vào lớp 10 chuyên Tin Hà


Nội, năm 2021
Câu 1. (2,0 điểm)

1. Giải phương trình 4 + 2x − x2 = x − 2.

iế n
(
x3 + 2 = 3y
2. Giải hệ phương trình .
y 3 + 2 = 3x

K
Câu 2. (2,0 điểm)

1. Chứng minh rằng với mỗi số nguyên n, số n2 + 3n + 16 không chia hết cho 25.


2. Tìm tất cả các số nguyên x và y thoả mãn x2 − xy − 2y 2 + x + y − 5 = 0.

-T
Câu 3. (2,0 điểm)

1. Cho a, b, c thực đôi một phân biệt. Chứng minh:


A1
        
a+b b+c b+c c+a c+a a+b
+ + = −1.
a−b b−c b−c c−a c−a a−b

a b c
n
2. Cho biểu thức P = √ + √ + √ với a, b, c không âm và
1 + 2bc 1 + 2ca 1 + 2ab
á

a2 + b2 + c2 = 1. Tìm giá trị lớn nhất của P .


To

Câu 4. (3,0 điểm) Cho tam giác nhọn ABC nội tiếp đường tròn (O) và AB < AC. Gọi
I là tâm đường tròn nội tiếp của tam giác ABC. Đường thẳng AI cắt đường tròn (O) tại
điểm thứ hai M (M khác A). Gọi D, E và F lần lượt là các hình chiếu của điểm I trên
bộ

các cạnh BC, CA và AB.

(a) Chứng minh tam giác M BI là tam giác cân.

(b) Đường tròn ngoại tiếp tam giác AEF cắt đường tròn (O) tại điểm thứ hai P (P khác
c

A). Chứng minh P, M và D là ba điểm thẳng hàng.


lạ

(c) Gọi H là giao điểm của đường thẳng IP và đường thẳng EF . Chứng minh HD song
song với AM .
u

Câu 5. (1,0 điểm) Trên bàn có n viên kẹo. Hai bạn An và Bình cùng chơi một trò chơi

như sau: Hai bạn luân phiên lấy kẹo trên bàn, mỗi lần chỉ được lấy 1, 2, 3, 4 hoặc 5 viên
kẹo và phải lấy số viên kẹo khác với số viên kẹo của bạn còn lại vừa lấy ngay trước đó.
Bạn đầu tiên không thể thực hiện được lượt chơi của mình là người thua cuộc. Nếu An
là người lấy kẹo trước,

1. Với n = 7, hãy chỉ ra chiến thuận chơi của Bình khiến An là người thua cuộc.

2. Với n = 22, hãy chỉ ra chiến thuận chơi của An khiến là người thua cuộc.
Nguyễn Tiến Lâm - Trịnh Huy Vũ 27

1.20. Đề thi tuyển sinh vào lớp 10 trường THPT


chuyên Khoa học Tự nhiên năm 2022, vòng 1
Câu 1 (4 điểm).
1) Giải hệ phương trình

iế n
(
6(xy + 5) + x3 y + 5x2 = 42
.
x3 + 5x2 y + 6x + 30y = 42

K
2) Giải phương trình
√ √ p
( 3 x + 6 + 3 3 − x)(2 + 3 3 (x + 6)(3 − x)) = 24.


-T
Câu 2 (2 điểm).
1) Tìm tất cả các cặp số nguyên (x, y) thoả mãn đẳng thức

25y 2 + 354x + 60 = 36x2 + 305y + (5y − 6x)2022 .


A1
2) Trên bàn có 8 hộp rỗng (trong các hộp không có viên bi nào). Người ta thực hiện
các lần thêm bi vào các hộp theo quy tắc sau: mỗi lần ta chọn ra 4 bất kỳ và bỏ vào
một hộp 1 viên, một hộp 2 viên, hai hộp còn lại mỗi hộp 3 viên. Hỏi số lần thêm bi
n

ít nhất có thể nhận được số bi ở 8 hộp trên là 8 số tự nhiên liên tiếp?


á

Câu 3 (3 điểm).
To

Cho hình chữ nhật ABCD (AB < AD) nội tiếp trong đường (O). Trên cạnh AD lấy
hai điểm E và F (E, F không trùng với A, D) sao cho E nằm giữa A và F, đồng thời
ABE
[ + DCF \ = 1 BOC.
\
2
bộ

1) Chứng minh BE và CF cắt nhau tại một điểm nằm trên đường tròn (O).

2) Đường thẳng qua O song song với BC cắt BE, CF theo thứ tự tại M, N. Chứng
c

minh rằng DAM


\ + ADN\ + 1 AOD \ = 180◦ .
lạ

2
3) Dựng hình chữ nhật M N P Q sao cho N Q song song với BD, đồng thời M P song
u

song với AC. Chứng minh rằng đường tròn ngoại tiếp hình chữ nhật M N P Q tiếp
xúc với đường tròn (O).

Câu 4. (1 điểm)
Cho a, b, c là các số thực dương. Chứng minh rằng
2a a + b 6a + 2c 4a + 3b + c 32a
+ + + ≥ .
a+b a+c 3b + c b+c 2a + b + c
28 Tuyển tập đề thi vào lớp 10 chuyên Toán

1.21. Đề thi tuyển sinh vào lớp 10 trường THPT


chuyên Khoa học Tự nhiên năm 2022, vòng 2
Câu 1 (3.5 điểm).
1 1 1
1) Với a, b, c là những số thực dương thỏa mãn điều kiện + + = 1. Chứng minh
a b c

iế n
rằng s
 
1 1 1 1 abc
+ + = .
2 a + bc b + ca c + ab (a + bc)(b + ca)(c + ab)

K
2) Giải hệ phương trình
(
2x2 + 3xy + y 2 = 6



3x + 2y + 1 = 2 2x + y + 6.

-T
Câu 2 (2.5 điểm).
1) Tìm tất cả các cặp số nguyên dương (x, y) thỏa mãn đẳng thức
(x + y)(5x + y)3 + xy 3 = (5x + y)3 + x2 y 3 + xy 4 .
A1
2) Với a, b, c là những số thực dương thoả mãn các điều kiện sau
(
c ≤ b ≤ a ≤ 3, b2 + 2a ≤ 10, b2 + 2a + 2c ≤ 14,
n

(a2 + 1)(b2 + 1) + 4ab ≤ 2a3 + 2b3 + 2a + 2b.


á

Tìm giá trị lớn nhất của biểu thức


To

P = 4a2 + b4 + 2b2 + 4c2 .


Câu 3 (3 điểm).
bộ

Cho tam giác ABC nhọn, không cân, nội tiếp đường tròn (O). Điểm P nằm trong tam
giác ABC. Gọi E, F lần lượt là hình chiếu vuông góc của P trên các cạnh CA, AB. Giả
sử tứ giác BCEF nội tiếp trong đường tròn (K).
1) Chứng minh rằng AP vuông góc với BC.
c
lạ

2) Chứng minh rằng AP = 2OK.


3) Đường thẳng qua P vuông góc với AP cắt đường tròn (O) tại hai điểm Q và R. Chứng
u

minh rằng đường tròn tâm A bán kính AP tiếp xúc với đường tròn ngoại tiếp tam
giác KQR.

Câu 4. (1 điểm)
Cho các điểm A1 , A2 , ..., A30 theo thứ tự nằm trên một đường thẳng sao cho độ dài
các đoạn Ai Ai+1 bằng k (đơn vị dài), với k = 1, 2, ..., 29. Ta tô màu mỗi đoạn thẳng
A1 A2 , A2 A3 , ..., A29 A30 bởi một trong ba màu (mỗi đoạn được tô bởi đúng một màu).
Chứng minh rằng với mọi cách tô màu, ta luôn chọn được hai số nguyên dương 1 ≤ i ≤
j ≤ 29 sao cho hai đoạn Ai Ai+1 , Aj Aj+1 được tô cùng màu và i − j là bình phương của
một số nguyên dương.
Nguyễn Tiến Lâm - Trịnh Huy Vũ 29

1.22. Đề thi tuyển sinh vào lớp 10 trường THPT


chuyên Sư phạm Hà Nội năm 2022, vòng 2
Câu 1 (2,5 điểm).
p
3
√ p3

a) Không sử dụng máy tính, hãy tính giá trị của biểu thức P = 7 + 5 2+ 7 − 5 2.

iế n
b) Cho đa thức P (x) = ax2 + bx + c. Chứng minh rằng nếu P (x) nhận giá trị nguyên
với mỗi số nguyên x thì ba số 2a, a + b, c đều là những số nguyên. Sau đó, chứng tỏ
rằng nếu 2a, a + b, c là những số nguyên thì P (x) cũng nhận giá trị nguyên với mỗi

K
số nguyên x.

Câu 2 (3.0 điểm). Cho tam giác đều ABC ngoại tiếp đường tròn (O). Cung nhỏ OB
của đường tròn ngoại tiếp tam giác tam giác OBC cắt đường tròn (O) tại điểm E. Tia


BE cắt đường tròn (O) tại điểm thứ hai F .

-T
a) Chứng minh tia EO là tia phân giác của góc CEF.

b) Chứng minh tứ giác ABOF nội tiếp. A1


c) Gọi D là giao điểm thứ hai của CE với đường tròn (O). Chứng minh ba điểm A, F, D
thẳng hàng.

Câu 3 (2,0 điểm). Cho a, b, c, d là các số nguyên dương thoả mãn ab = cd. Chứng minh
n

rằng N = a2022 + b2022 + c2022 + d2022 là hợp số.


á

Câu 4 (2,0 điểm). Ta viết 10 số 0, 1, 2, . . . , 9 vào mười ô tròn trong hình bên, mỗi số
To

được viết đúng một lần. Sau đó ta tính tổng ba số trên mỗi đoạn thẳng để nhận được 6
tổng. Có hay không một cách viết 10 số như thế sao cho 6 tổng nhận được là bằng nhau?

Câu 5 (1,0 điểm).


bộ

a) Trong mặt phẳng cho 5 điểm sao cho không có ba điểm nào thẳng hàng. Chứng
minh rằng tồn tại ít nhất một tam giác tù có các đỉnh được lấy từ 5 điểm đã cho.

b) Trong mặt phẳng cho 2022 điểm sao cho không có ba điểm nào thẳng hàng. Chứng
c

minh rằng tồn tại ít nhất 2018 tam giác tù mà mỗi tam giác tù đó có các đỉnh được
lạ

lấy từ 2022 điểm đã cho.


u

30 Tuyển tập đề thi vào lớp 10 chuyên Toán

1.23. Đề thi tuyển sinh vào lớp 10 chuyên Toán


Hà Nội, năm 2022
Câu 1 (2.0 điểm).

1. Giải phương trình x2 − 4x + 2 2x − 1 + 1 = 0.

iế n
2. Cho các số thực a, b và c thỏa mãn điều kiện ab + bc + ca = 1. Tính giá trị của biểu
thức
a b c 2
P = 2
+ 2
+ 2

1+a 1+b 1+c a + b + c − abc

K
Câu 2 (2.0 điểm).
1) Chứng minh rằng nếu n là số tự nhiên lẻ thì 32n+1 − 7 chia hết cho 20.


-T
2) Tìm tất cả cặp số nguyên dương (x, y) sao cho y(x2 + x + 1) = (x + 1)(y 2 − 1)

Câu 3 (2.0 điểm).


m3 n3
1) Tìm hai số nguyên dương m, n sao cho và đều là các số nguyên tố.
A1
m+n m+n
2) Với a, b và c là các số thực không âm thoả mãn điều kiện a + b + c = 3, tìm giá trị
lớn nhất của biểu thức P = ab + 2bc + 3ca − 3abc.
n
Câu 4 (3.0 điểm).
á

Cho tam giác ABC nhọn với AB < AC. Đường tròn (I) nội tiếp tam giác ABC, tiếp xúc
với ba cạnh BC, CA và AB lần lượt tại ba điểm D, E, F.
To

1) Gọi M là giao điểm của hai đường thẳng AI và DF. Chứng minh đường thẳng CM
vuông góc với đường thẳng AI.
bộ

2) Gọi N là giao điểm của hai đường thẳng AI và DE. Gọi K là trung điểm của đoạn
thẳng BC. Chứng minh tam giác KM N là tam giác cân.

3) Các tiếp tuyến tại M và N của đường tròn (K; KM ) cắt nhau tại điểm S. Chứng
c

minh đường thẳng AS song song với đường thẳng ID.


lạ

Câu 5 (1.0 điểm).


Cho tập hợp A gồm 70 số nguyên dương không vượt quá 90. Gọi B là tập hợp gồm các
u

số có dạng x + y với x, y ∈ A và x, y không nhất thiết phân biệt,


1) Chứng minh 68 ∈ B.

2) Chứng minh B chứa 91 số nguyên liên tiếp.


Nguyễn Tiến Lâm - Trịnh Huy Vũ 31

1.24. Đề thi tuyển sinh vào lớp 10 chuyên Tin Hà


Nội, năm 2022
Câu 1 (2.0 điểm).
p
1) Giải phương trình x2 − 2x + 2 = (x2 + 4)(x + 1).

iế n
2) Với a, b và c là các số thực dương thỏa mãn abc = 3, tính giá trị của biểu thức
1 1 1
P = 2 + 2 + 2 .
a (b + c) + 3 b (c + a) + 3 c (a + b) + 3

K
Câu 2 (2.0 điểm).
1) Với p là số nguyên tố lớn hơn 3 , chứng minh số A = 5p + p2 chia hết cho 6.


2) Tìm tất cả cặp số nguyên (x, y) thỏa mãn x3 − x2 y + 2x = 5x2 − 2y − 1.

-T
Câu 3 (2.0 điểm).
1) Với a, b, c là các số thực dương thỏa mãn abc = 2, chứng minh
9
a2 + b2 + c2 − 3 (a + b − c) ≥ − .
A1 4
2) Tìm tất cả các số nguyên dương a, b và c sao cho các phương trình x2 − 2ax + b = 0,
x2 − 2bx + c = 0 và x2 − 2cx + a = 0 đều có nghiệm là các số nguyên dương.
Câu 4 (3.0 điểm).
n

Cho tam giác ABC với AB < AC nội tiếp đường tròn (O). Ba đường cao AD, BE và
á

CF của tam giác ABC cùng đi qua điểm H. Gọi I và K lần lượt là trung điểm của EF
To

và BC.
AI HI
1) Chứng minh = .
AK HK
bộ

2) Chứng minh đường thẳng AH là tiếp tuyến của đường tròn ngoại tiếp tam giác
HIK.
3) Gọi P là chân đường vuông góc kẻ từ điểm H đến đường thẳng EF. Chứng minh
c

rằng đường thẳng DP song song với đường thẳng AI.


lạ

Câu 5 (1.0 điểm).


Trên bảng có hai số tự nhiên m và n . An và Bình chơi một trò chơi như sau : Mỗi lượt
chơi, một bạn chọn một trong hai số trên bảng để xóa và viết lên bảng một số mới là hiệu
u

không âm của số vừa xóa với một ước số tự nhiên bất kỳ của số vừa xóa. Hai bạn luân

phiên thực hiện lượt chơi. Bạn đầu tiên không thể thực hiện được lượt chơi của mình là
người thua cuộc , người còn lại là người thắng cuộc . Biết rằng An là người thực hiện lượt
chơi đầu tiên.
1) Với m = 2022, n = 2023 hãy chỉ ra chiến thuật chơi của An để An là người thắng
cuộc.
2) Vơi m = 2022, n = 1981 hãy chỉ ra chiến thuật chơi của An để An là người thắng
cuộc.
32 Tuyển tập đề thi vào lớp 10 chuyên Toán

1.25. Đề thi tuyển sinh vào lớp 10 chuyên Toán


Thành phố Hồ Chí Minh, năm 2022
p
Câu 1. Cho x, y là hai số thực thỏa mãn xy + (1 + x2 ) (1 + y 2 ) = 1. Tính giá trị của
biểu thức  p  √ 
M = x+ 1+y 2 y+ 1+x . 2

iế n
Câu 2.

a) Giải phương trình x + 4 + |x| = x2 − x − 4.

K
x


 = 2x − 1
y + z


y
= 3y − 1


b) Giải hệ phương trình
z+x
z

-T


= 5z − 1


x+y

Câu 3. Cho hình vuông ABCD. Trên các cạnh BC, CD lần lượt lấy các điểm M và N
sao cho M
\ AN = 45◦ .
A1
a) Chứng minh rằng M N tiếp xúc với đường tròn tâm A bán kính AB.

b) Kẻ M P k AN (P ∈ AB) và kẻ N Q k AM (Q ∈ AD). Chứng minh rằng AP = AQ.


n

Câu 4. Cho ba số thực dương a, b, c thỏa a + b + c = 3.


á

a) Chứng minh rằng ab + bc + ca ≤ 3.


To

a b c
b) Tìm giá trị nhỏ nhất của biểu thức P = + 2 + 2 .
b2 +1 c +1 a +1
Câu 5. Cho tam giác ABC nhọn (AB < AC) có các đường cao AD, BE, CF cắt nhau
bộ

tại H. Đường thẳng EF cắt đường thẳng BC tại I. Đường thẳng qua A vuông góc với
IH tại K và cắt BC tại M .
BI CI
c

a) Chứng minh rằng tứ giác IKF C nội tiếp và BD


= CD
.
lạ

b) Chứng minh M là trung điểm của BC.

Câu 6. Số nguyên dương n được gọi là "số tốt" nếu n + 1 và 8n + 1 đều là các số chính
u

phương.

a) Hãy chỉ ra ví dụ ba "số tốt" lần lượt có 1, 2, 3 chữ số.

b) Tìm các số nguyên k thỏa mãn |k| ≤ 10 và 4n + k là hợp số với mọi n là "số tốt".

u

Lời giải
lạ
c Chương 2
bộ
To

33
án
A1
-T

K
iế n
34 Tuyển tập đề thi vào lớp 10 chuyên Toán

2.1. Đề thi tuyển sinh vào lớp 10 trường THPT


chuyên Khoa học Tự nhiên năm 2018, vòng 1

Câu 1
1. Giải phương trình √ √
x2 − x + 2 x3 + 1 = 2 x + 1.

iế n
2. Giải hệ phương trình

K
(
xy + y 2 = 1 + y
x2 + 2y 2 + 2xy = 4 + x


-T
Lời giải.
1. Điều kiện xác định: x ≥ −1.
Phương trình đã cho tương đương với

p
A1
⇐⇒ x2 − x + 1 + 2 (x + 1)(x2 − x + 1) + x + 1 = x + 1 + 2 x + 1 + 1
√ √ 2 √ 2
⇐⇒ 2
x −x+1+ x+1 = x+1+1 .
n
Từ đó có 2 trường hợp xảy ra.
√ √ √
á

Trường hợp 1. x2 − x + 1 + x + 1 = x + 1 + 1

To

Phương trình ⇐⇒ x2 − x + 1 = 1 ⇐⇒ x2 − x = 0 ⇐⇒ x(x − 1) = 0.


Như vậy, x = 0 ∨ x = 1.
√ √ √
Trường hợp 2. x2 − x + 1 + x + 1 = − x + 1 − 1
bộ

Trường hợp này vô nghiệm vì VT ≥ 0 và VP < 0.

Kết luận. S = {0; 1}.


c

2. Nhân phương trình đầu với 2 rồi cộng với phương trình thứ hai, ta được
lạ

(x + 2y)2 − (x + 2y) − 6 = 0.
u

Giải tìm được x + 2y = 3 hoặc x + 2y = −2. Xét hai trường hợp


Trường hợp 1. Với x + 2y = 3 thì x = 3 − 2y, thay vào phương trình đầu, ta được

y 2 − 2y + 1 = 0.

Giải tìm được (x; y) = (1; 1).

Trường hợp 2. Với x + 2y = −2 thì x = −2 − 2y, thay vào phương trình đầu, ta
được
y 2 + 3y + 1 = 0.
Nguyễn Tiến Lâm - Trịnh Huy Vũ 35

√ ! √ !
√ −3 − 5 √ −3 + 5
Giải tìm được (x; y) = 1+ 5; và (x; y) = 1− 5; .
2 2

Câu 2
1. Tìm tất cả các cặp số nguyên (x, y) thỏa mãn

iế n
(x + y)(3x + 2y)2 = 2x + y − 1.

K
r
√ b
2. Với a, b là các số thực dương thỏa mãn a + 2b = 2 + , tìm giá trị nhỏ
3
nhất của biểu thức


a b
M=√ +√ .

-T
a + 2b b + 2a

Lời giải. A1
1. Viết lại phương trình dưới dạng (x + y)(3x + 2y)2 + x + y = 3x + 2y − 1 hay

3x + 2y − 1
x+y = .
(3x + 2y)2 + 1
n

t−1
Đặt t = 3x + 2y thì x + y = . Ta phải có t − 1 chia hết cho t2 + 1, suy ra
á

t2 + 1
(t − 1)(t + 1) = t2 − 1 chia hết cho t2 + 1. Suy ra 2 chia hết cho t2 + 1, nghĩa là
To

t2 + 1 = 1 hoặc t2 + 1 = 2. Xét ba khả năng

Trường hợp 1. t = 0 thì x + y = −1, 3x + 2y = 0, tìm được x = 2, y = −3.


bộ

Trường hợp 2. t = 1 thì x + y = 0, 3x + 2y = 1, tìm được x = 1, y = −1.


Trường hợp 3. t = −1 thì x + y = −1, 3x + 2y = −1 tìm được x = 1, y = −2.
c
lạ

Kết luận. Phương trình đã cho có ba nghiệm nguyên (2; 3), (1; −1) và (1; −2).

2. Áp dụng bất đẳng thức cộng mẫu số, ta có


u

b a2 b2 b2 (a + b + b)2

M+√ = √ + √ + √ ≥ √ √ √ .
b + 2b a a + 2b b b + 2a b b + 2b a a + 2b + b b + 2a + b b + 2b

Lại có
 √ √ √ 2 √ √ √ √ √ √ 2
a a + 2b + b b + 2a + b b + 2b = 2 2 2
a a + 2ab + b b + 2ab + b b + 2b 2

≤ (a + b + b)(a2 + 2ab + b2 + 2ab + b2 + 2b2 )


= (a + 2b)3 .
36 Tuyển tập đề thi vào lớp 10 chuyên Toán

Do đó, r
b √
M+ ≥ a + 2b,
3
r
√ b
kéo theo M ≥ a + 2b − = 2. Dấu bằng xảy ra khi a = b = 3.
3

iế n
Nhận xét. Xét bài toán sau

Cho các số thực a, b, c thỏa mãn điều kiện

K
√ √ √
a + 2b + b + 2c + c + 2a = 9.


Tìm giá trị nhỏ nhất của biểu thức

-T
a b c
M=√ +√ +√ .
a + 2b b + 2c c + 2a
A1
Như vậy câu 2.2 trong đề là một trường hợp đặc biệt của bài toán trên khi
cho b = c.
n

Câu 3
á

Cho tam giác ABC có đường tròn nội tiếp (I) tiếp xúc với các cạnh BC, CA, AB
To

lần lượt tại các điểm D, E, F . Gọi K là hình chiếu vuông góc của B trên đường
thẳng DE, M là trung điểm của đoạn thẳng DF .

1. Chứng minh rằng hai tam giác BKM và DEF đồng dạng.
bộ

2. Gọi L là hình chiếu vuông góc của C trên đường thẳng DF , N là trung điểm
của đoạn thẳng DE. Chứng minh rằng hai đường thẳng M K và N L song
song.
c
lạ

3. Gọi J, X lần lượt là trung điểm của các đoạn thẳng KL và ID. Chứng minh
rằng đường thẳng JX vuông góc với đường thẳng EF .
u

Lời giải.
1. Do 4BF D cân tại B có M là trung điểm của đáy DF nên BM ⊥ DF . Từ đó suy
ra tứ giác BM DK nội tiếp. Như vậy, kết hợp với giả thiết BC tiếp xúc với (I) tại
D, ta được

∠BKM = ∠BDM = ∠DEF ; ∠BM K = ∠BDK = ∠CDE = ∠DF E

Vậy 4BKM ∼ 4DEF .


Nguyễn Tiến Lâm - Trịnh Huy Vũ 37

iế n
I
N

K
M P
X


-T
B D C

J
A1
L
n

2. Do M N là đường trung bình của tam giác DEF nên M N k EF . Từ đó ta có


á
To

∠DM K = ∠DBK = 90◦ −∠BDK = 90◦ −∠CDE = 90◦ −∠DF E = 90◦ −∠DM N
bộ

Suy ra ∠KM N = 90◦ , hay KM ⊥ M N . Chứng minh tương tự như trên ta cũng
được LN ⊥ N M . Như vậy, ta thu được M K k N L.
c
lạ

3. Gọi P là trung điểm của M N . Khi đó, ta được JP là đường trung bình của hình
thang M KLN . Mặt khác, do ∠KM N = ∠LN M = 90◦ nên JP ⊥ M N . Từ đó suy
ra JP là trung trực của M N và JM = JN .
u

Trong khi đó, do ∠IM D = ∠IN D = 90◦ nên tứ giác IM DN nội tiếp đường tròn
đường kính ID. Suy ra XM = XN . Kết hợp với JM = JN chứng minh ở trên ta
được JX là trung trực của M N . Vậy JX ⊥ M N , mà M N k EF nên JX ⊥ EF .
38 Tuyển tập đề thi vào lớp 10 chuyên Toán

Câu 4
Trên mặt phẳng cho hai điểm P, Q phân biệt. Xét 10 đường thẳng nằm trong mặt
phẳng trên thỏa mãn tính chất sau:

i) không có hai đường thẳng nào song song hoặc trùng nhau.

ii) mỗi đường thẳng đều đi qua P hoặc Q, không có đường thẳng nào đi qua cả

iế n
P và Q.

Hỏi 10 đường thẳng trên có thể chia mặt phẳng thành tối đa bao nhiêu miền? Hãy

K
giải thích.


Lời giải. Giả sử có x đường kẻ từ P (gọi là x đường gốc P ) và y đường kẻ từ Q (gọi là

-T
y đường gốc Q) với x + y = 10. Ta thấy x đường gốc P chia mặt phẳng thành 2x miền.
Nếu có thêm một đường gốc Q thì để số miễn nhiều nhất, đường này sẽ cắt x đường gốc
P, tạo thêm x + 1 miền mới. Số miền mới sẽ bằng là 2x + (x + 1). Nếu có thêm một đường
gốc Q nữa thì đường này sẽ cắt x đường gốc P và một đường gốc Q, tạo thêm x + 2 miền
mới, cứ thế, cứ thế...Thế thì số miền sẽ là
A1
2x + (x + 1) + (x + 2) + · · · + (x + 2) = 2x + (x + 2)y − 1 = xy + 19.

(x + y)2
Lại có xy ≤ = 25, nên số miền sẽ không vượt quá 44 miền. Để đạt được điều
n

4
này thì phải có 5 đường gốc P và 5 đường gốc Q, trong đó không có hai đường thẳng nào
á

song song.
To
bộ
c
lạ
u

Nguyễn Tiến Lâm - Trịnh Huy Vũ 39

2.2. Đề thi tuyển sinh vào lớp 10 trường THPT


chuyên Khoa học Tự nhiên năm 2018, vòng 2

Câu 1
1. Giải hệ phương trình

iế n
(
xy(x + y) = 2
x3 + y 3 + x3 y 3 + 7(x + 1)(y + 1) = 31

K
2. Giải phương trình
√ √


p
9+3 x(3 − 2x) = 7 x + 5 3 − 2x.

-T
Lời giải.
1. Đặt a = x+y, b = xy thì hệ có thể viết lại thành ab = 2 và a3 −3ab+b3 +7(a+b)+7 =
A1
31. Từ đó, ta có
(a + b)3 + (a + b) − 30 = 0.
Giải tìm được a + b = 3 và ab = 2. Suy ra (x; y) = (1; 1)
n

Hệ đã cho có một nghiệm (1, 1).


á

3 √ √
To

2. Điều kiện: 0 ≤ x ≤ . Đặt a = x, b = 3 − 2x với a, b ≥ 0 thì theo đề ta có


2
2a2 + b2 = 3 và 9 + 3ab = 7a + 5b. Cộng cả hai phương trình lại, ta được

2a2 + 3ab + b2 − 7a − 5b + 6 = 0.
bộ

Tới đây, phân tích vế trái thành nhân tử, ta được


c

(a + b − 2)(2a + b − 3) = 0.
lạ

Xét hai trường hợp

√ √
u

Trường hợp 1. Nếu a + b = 2 thì x + 3 − 2x = 2, tới đây bình phương hai lần

1
tìm được x = 1 và x = .
9

Trường hợp 2. Nếu 2a + b = 3, tương tự như trường hợp 1 thì trường hợp này có
duy nhất nghiệm x = 1.

1
Tóm lại, phương trình đã cho có hai nghiệm x = 1 và x = .
9
40 Tuyển tập đề thi vào lớp 10 chuyên Toán

Câu 2
1. Cho x, y là các số nguyên sao cho x2 − 2xy − y và xy − 2y 2 − x đều chia hết
cho 5. Chứng minh rằng 2x2 + y 2 + 2x + y cũng chia hết cho 5.

2. Cho a1 , a2 , · · · , a50 là các số nguyên thỏa mãn 1 ≤ a1 ≤ a2 ≤ · · · ≤ a50 ≤ 50


và a1 + a2 + · · · + a50 = 100. Chứng minh rằng từ các số đã cho có thể chọn
được một vài số có tổng bằng 50.

iế n
Lời giải.

K
1. Từ giả thiết suy ra x2 − 2xy − y − (xy − 2y 2 − x) = x(x − y) − 2y(x − y) + x − y =
(x − y)(x − 2y + 1) chia hết cho 5. Tới đây, có hai trường hợp


Trường hợp 1. Nếu x − y chia hết cho 5 thì từ x2 − 2xy − y = x(x − y) − y(x + 1)

-T
chia hết cho 5 ta suy ra hoặc x, y đều chia hết cho 5 hoặc x, y chia 5 dư 4. Trong
mọi tình huống, ta đều suy ra đpcm.

Trường hợp 2. Nếu x − 2y + 1 chia hết cho 5 thì từ x2 − 2xy − y = x(x − 2y + 1) −


A1
(x + y) chia hết cho 5. Từ đó x + y chia hết cho 5, kết hợp x − 2y + 1 chia hết cho 5,
ta suy ra x chia 5 dư 3, còn y chia 5 dư 2. Từ đây 2x2 + y 2 + 2x + y ≡ 3 + 4 + 1 + 2
(mod 5), tức là 2x2 + y 2 + 2x + y chia hết cho 5.
n

2. Xét 50 tổng Si = a1 + a2 + · · · + ai , với i = 1, 2, · · · , 50. Xét hai trường hợp


á
To

Trường hợp 1. Nếu trong 50 tổng nói trên có hai tổng có cùng dư khi chia cho 50,
giả sử là Si , Sj với i > j thì Si − Sj chia hết cho 50 và rõ ràng 0 < Si − Sj < 100,
nên Si − Sj = 50. Từ đó suy ra điều phải chứng minh.
bộ

Trường hợp 2. Nếu 50 tổng trên cho số dư đôi một khác nhau khi chia cho 50.
Thay vị trí của a50 và a49 trong dãy tổng, thì ta được hai tổng thứ 49 và 50 mới,
∗ ∗ ∗ ∗
giả sử là S49 , S50 . Xét dãy tổng mới S1 , S2 , · · · , S48 , S49 , S50 mà có hai tổng có cùng
∗ ∗
số dư khi chia cho 50 thì trở lại trường hợp 1, kể cả trường hợp S49 − S50 = a50 − a1
c

chia hết cho 50 khi đó ta phải có a50 = a49 = · · · = a1 = 2 và ta cũng có điều phải
lạ

chứng minh. Nếu ngược lại thì 50 tổng này có số dư đôi một khác nhau khi chia
cho 50, trong đó 48 tổng đầu tiên không đổi nên ta phải có tập số dư của hai tổng
∗ ∗
S49 , S50 khi chia cho 50 trùng với tập số dư của hai tổng S49 , S50 . Xét hai trường
u

hợp nhỏ


(i) Nếu S49 ≡ S49 (mod 50) thì ta có a50 ≡ a49 (mod 50). Lặp lại tương tự ta sẽ có
a50 ≡ a49 ≡ · · · ≡ a1 (mod 50) và từ đó ai = 2 với mọi i = 1, 2, · · · , 50. Từ đó ta có
đpcm.
∗ ∗
(ii) Nếu S49 ≡ S50 (mod 50) thì S50 − S49 = a49 chia hết cho 50, nghĩa là a49 phải
bằng 50. Trường hợp này không xảy ra được.

Trong mọi trường hợp, ta có điều phải chứng minh.


Nguyễn Tiến Lâm - Trịnh Huy Vũ 41

Nhận xét. Thực ra, ta có thể trình bày bài toán trên gọn hơn, nhưng chúng
tôi chọn cách trình bày thế này thể hiện mạch tư duy khi giải bài toán. Đề bài
có thể thay dữ kiện a50 ≤ 50 bằng dữ kiện a50 khác 51. Có thể thay 50 bằng n
và 100 bằng 2n.

iế n
Câu 3
Cho ngũ giác lồi ABCDE nội tiếp đường tròn (O) có CD song song với BE. Hai

K
đường chéo CE và BD cắt nhau tại P . Điểm M thuộc đoạn thẳng BE sao cho
∠M AB = ∠P AE. Điểm K thuộc đường thẳng AC sao cho M K song song với AD,
điểm L thuộc đường thẳng AD sao cho M L song song với AC. Đường tròn ngoại


tiếp tam giác KBC lần lượt cắt BD và CE tại Q, S (Q khác B, S khác C)

-T
1. Chứng minh rằng ba điểm K, M, Q thẳng hàng.

2. Đường tròn ngoại tiếp tam giác LDE lần lượt cắt BD, CE tại T, R (T khác
D, R khác E). Chứng minh rằng năm điểm M, S, Q, R, T cùng thuộc một
đường tròn.
A1
3. Chứng minh rằng đường tròn ngoại tiếp của tam giác P QR tiếp xúc với đường
tròn (O).
á n
To

L
bộ

B E
c

M
lạ

T
O
S
u

P
Q
R

C D
42 Tuyển tập đề thi vào lớp 10 chuyên Toán

Lời giải.
1. Do tứ giác KBCQ nội tiếp và M K k AD nên ta có

∠CKQ = ∠CBQ = ∠CBD = ∠CAD = ∠CKM

Từ đó suy ra K, M, Q thẳng hàng.

iế n
2. Do các tứ giác BCQS và DET R là các tứ giác nội tiếp nên ta có

∠RSQ = ∠CSQ = ∠CBD = ∠CED = ∠RED = ∠RT Q

K
Mặt khác, do M K k AD, M L k AC nên

∠RM Q = ∠CAD = ∠CED = ∠RED = ∠RT Q


-T
Như vậy, ∠RSQ = ∠RT Q = ∠RM Q. Vậy năm điểm M, S, Q, R, T cùng thuộc một
đường tròn.

Ghi chú. Hơn nữa, trong mô hình của ý 2, ta còn có thể thu được một kết
A1
quả nữa như sau:

Đường tròn đi qua năm điểm M, S, Q, R, T tiếp xúc với đường thẳng
n
BE tại M.
á
To

3. Để đơn giản hơn, ta có thể phát biểu lại bài 3.3 như sau:
bộ

Bài toán. Cho ngũ giác lồi ABCDE nội tiếp đường tròn (O) có CD song
song với BE. Hai đường chéo CE và BD cắt nhau tại P . Điểm M thuộc đoạn
thẳng BE sao cho ∠M AB = ∠P AE. Lấy điểm R, Q tương ứng nằm trên các
đường thẳng P C, P D sao cho M R k AC và M Q k AD. Chứng minh rằng
c

đường tròn ngoại tiếp của tam giác P QR tiếp xúc với đường tròn (O).
lạ
u

Đặt AM, AP lần lượt cắt đường tròn (O) tại X, Y (X, Y khác A). Vì ∠M AB =

∠P AE nên XY k BE k CD.
Trước hết, ta sẽ chứng minh X ∈ (P QR). Thật vậy, do M R k AC nên ∠RM X =
∠CAX = ∠CEX = ∠REX nên tứ giác RM EX nội tiếp, vì vậy ∠RXM = ∠REM .
Chứng minh tương tự ta cũng được ∠QXM = ∠QBM . Từ đó,

∠RXQ = ∠RXM +∠QXM = ∠REM +∠QBM = 180◦ −∠BP E = 180◦ −∠CP D

Vậy tứ giác P QXR nội tiếp, hay X ∈ (P QR).


Nguyễn Tiến Lâm - Trịnh Huy Vũ 43

B E
M
O

iế n
P
Q

K
H R
t
C D


-T
X Y

Dựng tia tiếp tuyến Xt của đường tròn (O). Khi đó ta có


A1
∠RXt = ∠AXt − ∠AXR = ∠AEX − ∠M EP = ∠AEB + ∠P EX

Mặt khác, do tính đối xứng qua trung trực của CD nên ta có
n
∠RP X = ∠CP X = ∠DP Y = ∠P DA+∠P AD = ∠ADB+∠CAX = ∠AEB+∠P EX
á

Như vậy, ∠RXt = ∠RP X, vì vậy nên đường tròn (P QR) tiếp xúc với Xt. Từ đó
To

suy ra đường tròn (P QR) tiếp xúc với (O).

Câu 4
bộ

Cho a, b, c là các số thực dương. Chứng minh rằng


r r ! 
ab bc 1 1
√ +√
c

+ ≤ 2.
a+b b+c a+b b+c
lạ

Lời giải. Áp dụng bất đẳng thức AM−GM và Cauchy−Schwarz, ta có


u

√ √ √ √ √ √ √ √

ab bc ab + bc ab bc ab + bc
VT = + +p ≤ √ + √ +√ √ = 2.
a+b b+c (a + b)(b + c) 2 ab 2 bc ab + bc

Dấu bằng xảy ra khi a = b = c.


44 Tuyển tập đề thi vào lớp 10 chuyên Toán

2.3. Đề thi tuyển sinh vào lớp 10 chuyên Toán Hà


Nội, năm 2018

Câu 1

1. Giải phương trình: x2 + 3x + 8 = (x + 5) x2 + x + 2.

iế n
(
y 2 − 2xy = 8x2 − 6x + 1
2. Giải hệ phương trình
y 2 = x3 + 8x2 − x + 1

K
Lời giải.


-T
1. Phương trình đã cho có thể viết lại

(x2 + x + 2) − (x + 5) x2 + x + 2 + 2(x + 3) = 0

hay tương dương với


A1
√  √ 
x2 + x + 2 − 2 x2 + x + 2 − (x + 3) = 0.
n

Tới đây, xét hai trường hợp


á


To

Trường hợp 1. x2 + x + 2 = 2, tìm được x = 1 hoặc x = −2.


√ 7
Trường hợp 2. x2 + x + 2 = x + 3, tìm được x = − .
5
bộ

2. Cộng hai vế của phương trình đầu với x2 , ta được (y − x)2 = (3x − 1)2 . Tới đây, xét
hai trường hợp
c

Trường hợp 1. y − x = 3x − 1 hay y = 4x − 1. Thay vào phương trình thứ hai,


lạ

tìm được các nghiệm là


(0; −1), (1; 3), (7; 27).
u

Trường hợp 2. x − y = 3x − 1 hay y = 1 − 2x. Thay vào phương trình thứ hai,
tìm được các nghiệm là
(0; 1), (−1; 3), (−3; 7).

Vậy, hệ đã cho có 6 nghiệm (đã liệt kê ở trên).


Nguyễn Tiến Lâm - Trịnh Huy Vũ 45

Câu 2
1. Cho p, q là hai số nguyên tố lớn hơn 5. Chứng minh rằng p4 + 2019q 4 chia hết
cho 20.

2. √ √ số nguyên dương a, b, c, d thỏa mãn a < b ≤ c < d, ad = bc và


Cho các
d − a ≤ 1.

iế n
a. Chứng minh rằng a + d > b + c.
b. Chứng minh rằng a là một số chính phương.

K
Lời giải.


1. Ta có p4 + 2019q 4 ≡ p4 − q 4 (mod 20). Ta có p > 5 nên p4 không chia hết cho 5, dẫn

-T
tới p4 chia 5 dư 1, suy ra p4 − 1 chia hết cho 5. Mặt khác, p4 − 1 = (p2 − 1)(p2 + 1)
chia hết cho 4. Từ đó, ta có p4 − 1 chia hết cho 20. Chứng minh tương tự q 4 − 1
chia hết cho 20. Từ đó, suy ra điều phải chứng minh.

2.
A1
a. Từ giả thiết suy ra d − a > b − c ≥ 0, kéo theo (d − a)2 > (b − c)2 . Khai triển

d2 − 2da + a2 > b2 − 2bc + c2 .


n

Sử dụng giả thiết da = bc, nên từ bất đẳng thức trên, cộng vế trái với 4ad,
á

cộng vế phải với 4bc, ta được


To

(d + a)2 > (b + c)2 ,

kéo theo d + a > b + c.


bộ

√ √ √
b. Mặt khác ( b − c)2 ≥ 0, suy ra b + c ≥ 2 bc. Từ câu a), ta suy ra
√ √
d + a ≥ b + c + 1 ≥ 2 bc + 1 = 2 da + 1.
c

√ √ √ √
lạ

Suy ra ( d − √a)2 ≥ 1. Theo giả thiết thì ta phải có d − a = 1. Suy ra


d = a + 1 + 2 a. Do vế phải là số nguyên dương nên suy ra a là số chính
phương.
u

Nhận xét. Từ chứng minh trên, ta cũng suy ra được d là số chính phương.
a c x

Ngoài cách giải trên, ta còn có thể giải theo hướng = = với (x, y) = 1.
b d y
Từ đó, suy ra a = mx, b = my, c = nx, d = ny. Theo đề, ta có x < y, m < n
nên x + 1 ≤ y, m + 1 ≥ n. Từ đó
√ √
d ≥ (x + 1)(m + 1) ≥ ( xm + 1)2 = ( a + 1)2 ,
√ √
kéo theo d≥ a + 1. Từ đó, ta cũng có điều phải chứng minh.
46 Tuyển tập đề thi vào lớp 10 chuyên Toán

Câu 3
1. Với x, y, z là các số thực dương thỏa mãn xyz = 1. Chứng minh rằng
1 1 1
+ + = 1.
xy + x + 1 yz + y + 1 zx + z + 1

1 1 1

iế n
2. Với x, y, z là các số thự dương thay đổi và thỏa mãn + + = 3, tìm giá
x y z
trị lớn nhất của biểu thức

K
1 1 1
P =p +p +√ .
2x2 + y 2 + 3 2y 2 + z 2 + 3 2z 2 + x2 + 3


-T
Lời giải.
xyz 1 y
1. Từ giả thiết, ta có V T = + + = 1.
xy + x + xyz yz + y + 1 xyz + yz + 1

a b c
A1
Nhận xét. Ngoài cách giải trên, ta còn có thể đặt x = , y = , z = .
b c a

2. Theo bất đẳng thức Cauchy-Schwarz, ta có (2x2 + y 2 + 3)(2 + 1 + 3) ≥ (2x + y + 3)2 ,


n
suy ra √
6
á

X
VT ≤ .
2x + y + 3
To

(2 + 1 + 3)2 4 1 9
Áp dụng bất đẳng thức ≤ + + , từ trên, suy ra
2x + y + z 2x y 3
√   √
6 3 3 3 6
bộ

VT ≤ + + +9 = .
36 x y z 2
Dấu bằng xảy ra khi và chỉ khi x = y = z = 1.
c
lạ

Câu 4
Cho tứ giác ABCD (không có hai cạnh nào song song) nội tiếp đường tròn (O).
Các tia BA và CD cắt nhau tại điểm F . Gọi E là giao điểm của hai đường chéo
u

AC và BD. Vẽ hình bình hành AEDK.


1. Chứng minh rằng tam giác F KD đồng dạng với tam giác F EB.

2. Gọi M, N tương ứng là trung điểm của các cạnh AD, BC. Chứng minh rằng
đường thẳng M N đi qua trung điểm của đoạn thẳng EF .

3. Chứng minh rằng đường thẳng EF tiếp xúc với đường tròn ngoại tiếp của
tam giác EM N .
Nguyễn Tiến Lâm - Trịnh Huy Vũ 47

E
P

iế n
M O N
F K L

K
D


C

-T
Lời giải.
1. Ta có phép biến đổi góc như sau:
A1
∠F DK = ∠F DA − ∠KDA = ∠F BC − ∠EAD = ∠F BC − ∠EBC = ∠F BE
FD AD
Do 4F DA ∼ 4F BC nên = . Mặt khác, cũng do 4EAD ∼ 4EBC
FB BC
n

EA AD
nên = . Từ đó, kết hợp với giả thiết AEDK là hình bình hành, suy ra
á

EB BC
FD EA DK
= = . Vì vậy nên 4F KD ∼ 4F EB (cgc).
To

FB EB BE
2. Do 4F KD ∼ 4F EB nên ∠DF K = ∠BF E.
Mặt khác, dựng hình bình hành EBLC, chứng minh tương tự như câu a ta sẽ thu
bộ

được 4F LC ∼ 4F EA. Từ đó ta cũng thu được ∠CF L = ∠AF E.


Như vậy, ∠DF K = ∠CF L = ∠DF L. Vì vậy, F, K, L thẳng hàng. Chú ý rằng M, N
cũng là trung điểm của EK, EL. Từ đó, M, N và trung điểm P của EF thẳng hàng
c

trên đường trung bình của 4EKL.


lạ

FK FD
3. Do 4F KD ∼ 4F EB nên = . Mặt khác, do 4F LC ∼ 4F EA nên
FE FB
FE FA
= . Hơn nữa, vì tứ giác ABCD nội tiếp nên F A · F B = F C · F D nên
u

FL FC
FD FA

= .
FB FC
FK FE
Kết hợp những điều trên ta thu được = . Suy ra F E 2 = F K · F L. Từ đó,
FE FL
1 1
P E2 = · F E2 = · F K · F L = P M · P N
4 4
Từ đó suy ra EF tiếp xúc với đường tròn (EM N ).
48 Tuyển tập đề thi vào lớp 10 chuyên Toán

Câu 5
Cho tập hợp S = {x ∈ Z | 1 ≤ x ≤ 50}. Xét A là một tập con bất kỳ của tập hợp
S và có tính chất: Không có ba phần tử nào của tập hợp A là số đo độ dài ba cạnh
của một tam giác vuông.

1. Tìm một tập hợp A có đúng 40 phần tử và thỏa mãn điều kiện đề bài.

iế n
2. Có hay không có một tập hợp A có đúng 41 phần tử và thỏa mãn điều kiện
của đề bài? Hãy giải thích câu trả lời.

K
Lời giải. Nếu a, b, c lập thành độ dài ba cạnh của một tam giác vuông thì ta gọi bộ đó
là bộ Pythagore. Trước hết, ta có thể liệt kê được đúng 20 bộ Pythagore thỏa mãn yêu


cầu bài toán.

-T
1. Bỏ đi tất cả các số là bội của 5 trong tập A thì ta được tập gồm 40 số thỏa mãn đề
bài. Chú ý là nếu bộ (a, b, c) là bộ Pythagore thì abc chia hết cho 5, nghĩa là trong
ba số a, b, c phải có một số là bội của 5, nên nếu bỏ đi tất cả các bội của 5 thì sẽ
thu được tập hợp thỏa mãn.
A1
2. Từ tập hợp A tìm được ở câu a) ta bỏ khỏi A các số 8, 9, 24, 36 và thêm vào
10, 15, 25, 40, 45 ta sẽ thu được một tập hợp gồm 41 phần tử thỏa mãn. Vì tập này
không chứa các phần tử 8, 9, 24, 36 nên ta không thể tìm được một tam giác vuông
với cạnh là 10, 15, 25, 40, 45. Điều này có nghĩa là A chứa đúng 41 phần tử thỏa mãn
n

yêu cầu bài toán.


á
To

Nhận xét. Bài toán này sẽ hay hơn nếu hỏi như sau
bộ

Tìm số nguyên dương k nhỏ nhất để mọi tập con k phần tử của S luôn chứa
ít nhất một bộ Pythagore.
c

Đáp số: kmin = 42.


lạ
u

Nguyễn Tiến Lâm - Trịnh Huy Vũ 49

2.4. Đề thi tuyển sinh vào lớp 10 trường THPT


chuyên Khoa học Tự nhiên năm 2019, vòng 1

Câu 1
1. Giải phương trình

iế n
26x + 5 √ √
√ + 2 26x + 5 = 3 x2 + 30.
x2 + 30

K
2. Giải hệ phương trình
(
x2 + y 2 = 2


-T
(x + 2y)(2 + 3y 2 + 4xy) = 27

Lời giải. .
A1
1. Điều kiện xác định: 26x + 5 ≥ 0
Phương trình đã cho tương đương với
á n

√ √  √ √ 
26x + 5 + 3 x2 + 30 26x + 5 − x2 + 30 = 0
To

√ √
⇐⇒ 26x + 5 = x2 + 30 ⇐⇒ x2 − 26x + 25 = 0

Từ đây ta tìm được x ∈ {1, 25}.


bộ

2. Ta có
c

( (
2 2
x +y =2 x2 + y 2 = 2
⇐⇒
lạ

(x + 2y)(2 + 3y 2 + 4xy) = 27 (x + 2y)(x2 + 4y 2 + 4xy) = 27


( (
x2 + y 2 = 2 x2 + y 2 = 2
⇐⇒ ⇐⇒
u

(x + 2y)3 = 27 x + 2y = 3

Từ đây ta tìm được


  
1 7
(x, y) ∈ (1, 1); ,
5 5
50 Tuyển tập đề thi vào lớp 10 chuyên Toán

Câu 2
1. Tìm tất cả các cặp (x, y) nguyên thỏa mãn

(x2 − x + 1)(y 2 + xy) = 3x − 1.

2. Với x, y là các số thực thỏa mãn 1 ≤ y ≤ 2, xy + 2 ≥ 2y, tìm giá trị nhỏ nhất

iế n
của biểu thức
x2 + 4
M= 2 .
y +1

K
Lời giải.


 2
1 3

-T
2 2
1. Ta có x − x + 1 | 3x − 1, chú ý rằng x − x + 1 = x − + > 0 với mọi x, suy
2 4
ra
x2 − x + 1 ≤ |3x − 1|
A1 (2.1)

Ta xét hai trường hợp

Trường hợp 1. 3x − 1 > 0


√ √
Từ (2.6) suy ra x2 − 4x + 2 ≤ 0, do đó 2 − 2<x<2+ 2, từ đó x ∈ {1, 2, 3}.
n

Trường hợp 2. 3x − 1 ≤ 0
á
To

Từ (2.6) suy ra x2 + 2x ≤ 0, do đó −2 ≤ x ≤ 0 hay x ∈ {−2, −1, 0}

Bằng cách thử trực tiếp, ta tìm được các nghiệm là


bộ

(x, y) ∈ {(−2, 1); (1, 1); (1, −2)}

2y − 2
c

2. Từ xy + 2 ≥ 2y =⇒ x ≥ > 0. Do đó
y
lạ

 2
2y − 2
+4
u

y 8y 2 − 8y + 4
M≥ =
y2 + 1 y4 + y2

Mà 8y 2 − 8y + 4 ≥ y 4 + y 2 ⇐⇒ (2 − y)(y 3 + 2y 2 − 3y + 2) ≥ 0 (đúng do 1 ≤ y ≤ 2).


Vì vậy nên M ≥ 1.

Vậy minM = 1, dấu bằng xảy ra ⇐⇒ x = 1, y = 2


Nguyễn Tiến Lâm - Trịnh Huy Vũ 51

Câu 3
Cho hình vuông ABCD, đường tròn (O) nội tiếp hình vuông ABCD tiếp xúc với
các cạnh AB, AD lần lượt tại các điểm E, F . Gọi giao điểm của CE và BF là G.

1. Chứng minh rằng năm điểm A, F, O, G, E cùng nằm trên một đường tròn.

2. Gọi giao điểm của F B và đường tròn (O) là M (M 6= F ). Chứng minh rằng

iế n
M là trung điểm của đoạn thẳng BG.

3. Chứng minh rằng trực tâm của tam giác GAF nằm trên đường tròn (O).

K

A E B

-T
H G M

A1
F O
á n
To

D C
bộ

Lời giải. 1. Ta có E, F là trung điểm của AB, AD. Từ đó 4AF B = 4BEC(cgc), suy
ra ∠EBG = ∠ABF = ∠BCE = 90◦ − ∠BEG. Vậy ∠BGE = 90◦ hay BF ⊥ CE.
Từ đó, do ∠EAF = ∠EGF = 90◦ nên tứ giác AEGF nội tiếp. Hơn nữa, ∠OEA =
∠OF A = 90◦ nên tứ giác AEOF nội tiếp. Từ đó suy ra năm điểm A, F, O, G, E
c

cùng nằm trên một đường tròn.


lạ

2. Do EB là tiếp tuyến của đường tròn (O) nên ∠BEM = ∠EF M = ∠EF G. Mặt
khác, ta có tứ giác AEGF nội tiếp nên ∠EF G = ∠EAG. Như vậy, ∠BEM =
u

∠EAG suy ra EM k AG. Kết hợp với E là trung điểm của AB suy ra M là trung

điểm của BG (tính chất đường trung bình).

3. Gọi H là trực tâm của tam giác GAF . Ta cần chứng minh rằng H ∈ (O). Thật
vậy, ta có AE k GH(⊥ AD) và AH k GE(⊥ BF ) nên AEGH là hình bình hành.
Do đó, GH = AE = BE, từ đó BEHG là hình bình hành, suy ra HE k BF .
Mặt khác, ta có ∠AGF = ∠AOF = 45◦ , từ đó ∠EM F = ∠AGF = 45◦ và
∠HF M = 90◦ − ∠AGF = 45◦ =⇒ ∠EM F = ∠HF M . Suy ra EHF M là hình
thang cân, do đó cũng là một tứ giác nội tiếp. Vì vậy, H ∈ (EF M ) ≡ (O).
52 Tuyển tập đề thi vào lớp 10 chuyên Toán

Câu 4
Cho x, y, z là các số thực dương thỏa mãn xy + yz + xz = 1. Chứng minh rằng:
!3
1 1 1 2 x y z
2
+ 2
+ 2
≥ √ +p +√ .
1+x 1+y 1+z 3 1+x 2 1+y 2 1 + z2

iế n
Lời giải (Cách 1). Ta có:
√ √ √
2(x + y + z)(xy + yz + xz) (x y + z + y x + z + z x + y)3

K
VT = ; VP = √ √ √
(x + y)(y + z)(z + x) (x + y)(y + z)(z + x) x + y y + z z + x

Ta cần chứng minh:


-T
√ √ p
(x y + z + y x + z + z x + y)3 ≤ 3(x + y + x) (x + y)(y + z)(z + x) (2.2)

Thật vậy, ta có:


√ √ √ √ √ √ √ √ √
(x y + z + y x + z + z x + y)2 = ( x xy + xz + y yx + yz + z zx + zy)2
A1
≤ 2(x + y + z)(xy + yz + zx) = 2(x + y + z)

Từ đó suy ra
n

√ √ √ p 3 √ √
(x y + z + y x + z + z x + y)3 ≤ 2(x + y + z) = 2 2(x + y + z) x + y + z
á
To

Mặt khác,
r
p 8
3(x + y + x) (x + y)(y + z)(z + x) ≥ 3(x + y + z) · (x + y + z)(xy + yz + xz)
9
bộ

√ √
= 2 2(x + y + z) x + y + z

Từ đó suy ra bất đẳng thức (2.2) đúng. Vậy ta có điều phải chứng minh.
c
lạ

Lời giải (Cách 2). Sử dụng giả thiết 1 = xy + yz + zx ta có 1 + x2 = (x + y)(x + z).


Áp dụng bất đẳng thức AM-GM ta có
u

r  
x x x 1 x x

√ = · ≤ +
1 + x2 x+y x+z 2 x+y x+z

Tương tự, ta có  
y 1 y y
p ≤ +
1+y 2 2 y+x y+z
và  
z 1 z z
√ ≤ +
1 + z2 2 z+x z+y
Nguyễn Tiến Lâm - Trịnh Huy Vũ 53

Cộng các bất đẳng thức trên suy ra


X x 3
√ ≤
1 + x2 2

Từ đó, !2
X x
VT ≤ (2.3)

iế n
p
(x + y)(x + z)
Sử dụng bất đẳng thức Cauchy-Schwarz, ta có

K
!2  2
X x X√ r x
X
x

p = x ≤ (x+y+z)
(x + y)(x + z) (x + y)(x + z) (x + y)(x + z)


(2.4)

-T
Quy đồng mẫu số biểu thức trong ngoặc và sử dụng giả thiết thì
X 
x
(x + y + z) =VP (2.5)
(x + y)(x + z)
A1
Từ (2.3), (2.4), (2.5) ta có điều phải chứng minh.
á n
To
bộ
c
lạ
u

54 Tuyển tập đề thi vào lớp 10 chuyên Toán

2.5. Đề thi tuyển sinh vào lớp 10 trường THPT


chuyên Khoa học Tự nhiên năm 2019, vòng 2

Câu 1
1. Giải hệ phương trình

iế n
(
3x2 + y 2 + 4xy = 8
(x + y)(x2 + xy + 2) = 8

K
2. Giải phương trình
√ √


27 + x2 + x 27 + 2x
= √

-T
p
2 + 5 − (x2 + x) 2 + 5 − 2x

Lời giải. 1. Phương trình thứ nhất có thể viết lại (3x + y)(x + y) = 8. Thay vào
phương trình thứ hai, ta được
A1
(x + y)(x2 + xy + 2 − 3x − y) = 0.
n
hay tương đương
(x + y)(x − 1)(x + y − 2) = 0.
á
To

Từ đây, ta có ba khả năng. Giải, ta thu được các nghiệm

(x, y) ∈ {(1; 1), (1; −5)}.


bộ

27 5
2. Điều kiên xác định: x2 + x ≤ 5 và − ≤ x ≤ .
2 2

27 + t
Xét biểu thức f (t) = √ với −27 ≤ t ≤ 5 thì dễ thấy khi t tăng thì tử số
c

2+ 5−t
lạ

tăng, còn mẫu số giảm, do đó phân số sẽ tăng. Sử dụng điều này, ta thấy

(a) Nếu x2 + x > 2x thì vế trái lớn hơn vế phải.


u

(b) Nếu x2 + x < 2x thì vế trái nhỏ hơn vế phải.


Do đó, ta phải có x2 + x = 2x. Giải ta tìm được x = 0 và x = 1. Các nghiệm này


đều thỏa mãn.
Vậy, phương trình đã cho có nghiệm x ∈ {0; 1}.

Nhận xét. Ý thứ 2 có nhiều cách trình bày, nhưng chúng tôi chọn cách giải trên cho
ngắn gọn.
Nguyễn Tiến Lâm - Trịnh Huy Vũ 55

Câu 2
1. Chứng minh rằng với mọi số nguyên dương n, ta luôn có
7 7 7
(27n + 5)7 + 10 + (10n + 27)7 + 5 + (5n + 10)7 + 27

chia hết cho 42.

iế n
2. Với x, y là các số thực dương thỏa mãn điều kiện 4x2 +4y 2 +17xy+5x+5y ≥ 1,
tìm giá trị nhỏ nhất của biểu thức

K
P = 17x2 + 17y 2 + 16xy


-T
Lời giải.

1. Ta có bổ đề sau A1
Bổ đề. Ta có a7 ≡ a (mod 42) với mọi số nguyên a.
n

Bổ đề này quen thuộc, ta không chứng minh.


á

Áp dụng bổ đề thì ta có ngay A ≡ 27n + 5 + 10 + 10n + 27 + 5 + 5n + 10 + 27


To

(mod 42) hay A ≡ 42n + 84 (mod 42). Do đó, A chia hết cho 42.

2. Đặt S = x + y, T = xy thì áp dụng bất đẳng thức (x + y)2 ≥ 4xy ta có S 2 ≥ 4T.


bộ

Từ giả thiết, suy ra 4S 2 + 5S + 9T ≥ 1, kết hợp với S 2 ≥ 4T, ta suy ra

9
4S 2 + 5S + S 2 ≥ 1.
c

4
lạ

2 √
Chú ý là S > 0 nên từ bất đẳng thức trên, ta suy được S ≥ ( 2 − 1). Do đó,
5
u

18 2 25 2 √
P = 17S 2 − 18T ≥ 17S 2 − S = S ≥ 6 − 4 2.
4 2

2 √ S2 2−1
Dấu bằng đạt được khi và chỉ khi S = ( 2−1), T = , giải được x = y = .
5 4 5
56 Tuyển tập đề thi vào lớp 10 chuyên Toán

Câu 3
Cho tam giác ABC cân tại A, có đường tròn nội tiếp (I). Các điểm E, F theo thứ
tự thuộc các cạnh CA, AB (E khác C và A; F khác B và A) sao cho EF tiếp xúc
với đường tròn (I) tại điểm P . Gọi K, L lần lượt là hình chiếu vuông góc của E, F
trên BC. Giả sử F K cắt EL tại điểm J. Gọi H là hình chiếu vuông góc của J trên
BC.

iế n
1. Chứng minh rằng HJ là phân giác của ∠EHF .

2. Ký hiệu S1 và S2 lần lượt là diện tích của các tứ giác BF JL và CEJK. Chứng

K
minh rằng
S1 BF 2
=
S2 CE 2


-T
3. Gọi D là trung điểm của cạnh BC. Chứng minh rằng ba điểm P, J, D thẳng
hàng.

A
A1
n

Y E
á
To

N M
P
F X

R
bộ

J I
c
lạ
u

B L H D K C

Lời giải. 1. Áp dụng định lý Thales ta có


FL FJ LH
= =
EK JK KH
Từ suy ra 4F LH ∼ 4EKH(cgc), vì vậy nên ta được ∠F HJ = ∠LF H =
∠KEH = ∠EHJ và do đó HJ là phân giác của ∠EHF .
Nguyễn Tiến Lâm - Trịnh Huy Vũ 57

2. Ta có 4F LB ∼ 4EKC(gg) vì vậy nên

SF LB BF 2 F L2
= =
SEKC CE 2 EK 2

Hơn nữa ta cũng có 4F JL ∼ 4KJE(gg) nên

iế n
SF JL F L2 BF 2
= =
SKJE EK 2 CE 2

Từ đó suy ra

K
BF 2 SF LB SF JL SF LB + SF JL S1
2
= = = =
CE SEKC SKJE SEKC + SKJE S2


-T
3. Ta có D là tiếp điểm của đường tròn (I) với BC và A, I, D thẳng hàng. Gọi Q, R là
tiếp điểm của đường tròn (I) với CA, AB. X, Y lần lượt là giao điểm của P Q, P R
với AD.
Ta có IE, IF tương ứng là phân giác của các góc ∠P IQ, ∠P IR nên ∠EIF =
1
A1
∠QIR = ∠AIQ = ∠AIR, từ đó thu được ∠AIE = ∠F IR và ∠AIF = ∠EIQ.
2
Do đó, ∠XP E = ∠QP E = ∠EIQ = ∠AIF suy ra tứ giác XP F I nội tiếp và
∠F XI = ∠F P I = 90◦ hay F X ⊥ AD. Chứng minh tương tự ta cũng được tứ giác
Y P IE nội tiếp và EY ⊥ AD. Từ đó, các tứ giác F LDX, EKDY là các hình chữ
n

nhật, do đó DX = F L và DY = EK.
á

Mặt khác, ta có ∠IP X = ∠IEP = ∠IY P suy ra 4IXP ∼ 4IP Y =⇒ IP 2 =


To

IX · IY . Dựng đường kính DM của đường tròn (I). Tiếp tuyến tại M của (I) cắt
EF tại N . Từ đó, ta được ID2 = IM 2 = IX · IM = IX · IY và do đó

IX ID ID + IX DX FL FJ
bộ

= = = = =
ID IY ID + IY DY EK JK

IX IM IM − IX MX FN
= = = =
c

IM IY IY − IM MY NE
lạ

FJ FN
Từ đó suy ra = =⇒ JN k EK. Theo tiên đề Euclid suy ra N, J, H thẳng
JK NE
JN FJ LJ
u

hàng. Từ đó suy ra M N HD là hình chữ nhật. Hơn nữa, ta có = = =


EK FK LE
JH

suy ra J là trung điểm của N H. Kết hợp với I là trung điểm M D, ta suy ra
EK
các tứ giác M N JI và IJHD là hình chữ nhật. Từ đó ta cũng có JN = IM = ID
và JN k ID nên N JDI là hình bình hành và ta được JD k IN . Mặt khác, chú
ý rằng phân giác góc ngoài ở đỉnh của tam giác cân song song với đáy, ta suy ra
DP k IN (dễ thấy IN là phân giác của ∠M IP ). Từ đó, áp dụng tiên đề Euclid
suy ra P, J, D thẳng hàng.
58 Tuyển tập đề thi vào lớp 10 chuyên Toán

Câu 4
Cho M là tập tất cả 4039 số nguyên liên tiếp từ −2019 đến 2019. Chứng minh rằng
trong 2021 số đôi một phân biệt được chọn bất kỳ từ tập M luôn tồn tại 3 số đôi
một phân biệt có tổng bằng 0.

Lời giải. Ta giải bài toán tổng quát như sau: Trong 2n + 1, n ∈ Z+ phần tử bất kỳ của

iế n
tập
M = {−(2n − 1), −(2n − 2), ..., 0, ..., 2n − 2, 2n − 1}
luôn có ba phần tử có tổng bằng 0.

K
Ta chứng minh bằng phương pháp quy nạp. Thật vậy, dễ dàng kiểm tra với n = 1 thì
M = {−1; 0; 1} thì hiển nhiên khẳng định được chứng minh. Giả sử khẳng định đúng với


n. Với n + 1 xét tập

-T
M = {−(2n + 1), −2n, ..., 0, ..., 2n, 2n + 1}.
Đặt N = {−(2n + 1), −2n, 2n, 2n + 1}, xét các khả năng sau
Trường hợp 1. Nếu trong 2n + 3 phần tử có tối đa hai phần tử thuộc N thì sẽ có ít
nhất 2n + 1 phần tử thuộc tập {−(2n − 1), −(2n − 2), ..., 0, ..., 2n − 2, 2n − 1}. Sử dụng
A1
giả thiết quy nạp, ta có ngay điều phải chứng minh.
Trường hợp 2. Nếu 2n + 3 phần tử đều chứa cả bố phần tử của N, thì bỏ đi hai phần
tử −2n − 1, 2n + 1, xét 2n + 1 phần tử còn lại, thuộc vào trong 2n nhóm.
n

(1, 2n), (2, 2n − 1), ..., (n, n + 1), (0, −2n − 1), (−1, −2n), ..., (−n, −n − 1).
á

Theo nguyên tắc Dirichlet, tồn tại hai phần tử thuộc cùng một nhóm, tổng của chúng là
To

−2n − 1 hoặc 2n + 1. Trong tình huống nào ta cũng có điều phải chứng minh.
Trường hợp 3. Nếu 2n + 3 phần tử chứa đúng ba phần tử của N. Xét các khả năng sau
1. Nếu 3 phần tử đó là −2n − 1, 2n + 1, 2n thì nếu trong 2n phần tử còn lại chứa 0
bộ

hoặc 1 thì xong, còn nếu không chứa 0 và 1 thì 2n phần tử còn lại sẽ thuộc 2n − 1
nhóm
(−1, −2n), (−2, −2n + 1), ..., (−n, −n − 1), (2, 2n − 1), (3, 2n − 2), ..., (n, n + 1)
c

nên theo nguyên tắc Dirichlet, tồn tại hai phần tử thuộc cùng một nhóm, tổng hai
lạ

phần tử này là −(2n + 1) hoặc 2n + 1. Trong tình huống nào ta cũng có điều phải
chứng minh.
u

2. Nếu 3 phần tử đó là −2n, 2n, 2n + 1 thì lập luận tương tự như trên bằng cách thay

0,1 bằng 0, −1. Ta bỏ đi phần tử n, thì 2n − 1 phần tử còn lại sẽ thuộc 2n − 2 nhóm
(1, 2n − 1), (2, 2n − 2), ..., (n − 1, n + 1), (−2, −2n + 1), ..., (−n, −n − 1),
nên theo nguyên lý Dirichlet, tồn tại hai phần tử thuộc cùng một nhóm, hai phần tử
này có tổng là 2n hoặc −2n − 1. Trong tình huống nào ta cũng có điều phải chứng
minh.
Các trường hợp còn lại được xét tương tự bằng cách đổi dấu lại.
Nguyễn Tiến Lâm - Trịnh Huy Vũ 59

2.6. Đề thi tuyển sinh vào lớp 10 trường THPT


chuyên Sư phạm Hà Nội năm 2019, vòng 2

Câu 1
Cho hai số thực phân biệt a, b thỏa mãn a3 + b3 = a2 b2 (ab − 3). Tính giá trị của

iế n
biểu thức T = a + b − ab.

Lời giải. Giả thiết có thể viết lại thành

K
(a + b)3 − a3 b3 + 3a2 b2 − 3ab(a + b) = 0


hay tương đương

-T
(a + b − ab).M = 0
 2
2 2 2 a+b 3
với M = (a + b) + ab(a + b) + a b − 3ab = ab + + (a − b)2 > 0 do a, b phân
2 4
biệt. Từ đó suy ra T = a + b − ab = 0.
A1
Câu 2
n

Cho các đa thức P (x) = m1 x2 + n1 x + k1 , Q(x) = m2 x2 + n2 x + k2 , R(x) = m3 x2 +


á

n3 x + k3 với mi , ni , ki là các số thực và mi > 0, i = 1, 2, 3. Giả sử phương trình


To

P (x) = 0 có hai nghiệm phân biệt a1 , a2 , phương trình Q(x) = 0 có hai nghiệm
phân biệt b1 , b2 , phương trình R(x) = 0 có hai nghiệm phân biệt c1 , c2 thỏa mãn

P (c1 ) + Q(c1 ) = P (c2 ) + Q(c2 )


bộ

P (b1 ) + R(b1 ) = P (b2 ) + R(b2 )


Q(a1 ) + R(a1 ) = Q(a2 ) + R(a2 )

Chứng minh rằng a1 + a2 = b1 + b2 = c1 + c2 .


c
lạ

Lời giải. Ta có P (c1 ) + Q(c1 ) = P (c2 ) + Q(c2 ) tương đương với


u

(m1 + m2 )(c1 + c2 ) + (n1 + n2 ) = 0.


n1 + n2 n3
Suy ra c1 + c2 = − . Áp dụng định lý Viete cho R(x), ta có c1 + c2 = − . Suy
m1 + m2 m3
ra
n3 n1 + n2 n1 + n2 + n3
= = .
m3 m1 + m2 m1 + m2 + m3
n3 n2 n1
Tương tự với các đẳng thức còn lại, suy ra = = . Áp dụng định lý Viete, ta
m3 m2 m1
có ngay a1 + a2 = b1 + b2 = c1 + c2 .
60 Tuyển tập đề thi vào lớp 10 chuyên Toán

Câu 3
1. Tìm các số nguyên x, y thỏa mãn x2 y 2 − 4x2 y + y 3 + 4x2 − 3y 2 + 1 = 0.

2. Cho ba số nguyên dương a, b, c thỏa mãn a3 + b3 + c3 chia hết cho 14. Chứng
minh rằng abc chia hết cho 14.

iế n
Lời giải.
1. Phương trình đã cho có thể viết lại thành

K
(y 2 − 4y + 4)x2 = −(y 3 − 3y 2 + 1).


Dễ thấy y = 2 không thỏa mãn nên ta có y −2 | y 3 −3y 2 +1 = y 2 (y −2)−(y 2 −4)−3.

-T
Suy ra y − 2 | 3. Từ đây, suy ra y ∈ {−1; 1; 3; 5}. Bằng cách thử trực tiếp, ta tìm
được 2 cặp (x, y) thỏa mãn là (1; 1), (−1; 1).

2. Từ giả thiết suy ra trong ba số a, b, c phải có ít nhất một số chẵn, kéo theo abc chia
hết cho 2.
A1
Mặt khác, dễ dàng kiểm tra x3 ≡ 0, −1, 1 (mod 7) với mọi số nguyên x nên từ điều
kiện 7 | a3 + b3 + c3 ta suy ra trong ba số a, b, c phải có ít nhất một số chia hết cho
7, vì nếu không có số nào chia hết cho 7 thì a3 + b3 + c3 ≡ −3, 3, −1, 1 (mod 7), vô
lí. Từ đó suy ra abc chia hết cho 7.
n

Từ các điều vừa chứng minh ở trên, với chú ý (2, 7) = 1 nên abc chia hết cho 14.
á
To

Câu 4
Cho tam giác ABC có ba góc nhọn nội tiếp đường tròn (O) và AB > AC. Gọi
bộ

D, E lần lượt là chân các đường cao của tam giác ABC hạ từ A, B. Gọi F là chân
đường vuông góc hạ từ B lên đường thẳng AO.

1. Chứng minh rằng B, D, E, F là bốn đỉnh của một hình thang cân.
c
lạ

2. Chứng minh rằng EF đi qua trung điểm của BC.

3. Gọi P là giao điểm thứ hai của đường thẳng AO và đường tròn (O), M và N
u

lần lượt là trung điểm của EF và CP . Tính góc ∠BM N .


Lời giải.
1. Ta có ∠ADB = ∠AEB = ∠AF B = 90◦ nên ba điểm A, B, D, E, F cùng nằm trên
đường tròn đường kính AB. Như vậy, tứ giác BEDF nội tiếp. Chú ý rằng ta có kết
quả quen thuộc ∠OAB = ∠DAC nên ∠F DB = ∠F AB = ∠DAC = ∠DBE suy
ra DF k BE. Từ đó suy ra BEDF là hình thang cân.
Nguyễn Tiến Lâm - Trịnh Huy Vũ 61

O E

iế n
M
D

K
B C
K
F
N


-T
P

2. Gọi K là giao điểm của EF và BC. Do BEDF là hình thang cân nên KE = KB và
từ đó ∠KEB = ∠KBE. Do đó, ∠KCE = 90◦ − ∠KBE = 90◦ − ∠KEB = ∠KEC
A1
suy ra tam giác KEC cân tại K. Như vậy, KC = KE = KB và vì vậy K là trung
điểm của BC.

3. Do các tứ giác ABF E và ABP C nội tiếp nên ∠BF E = 180◦ − ∠BAE = 180◦ −
n
∠BAC = ∠BP C và ∠BEF = ∠BAF = ∠BAP = ∠BCP . Suy ra 4BF E ∼
4BP C(gg). Hai tam giác đồng dạng này có cặp trung tuyến tương ứng là BM và
á

BM BE
BN , suy ra = và ∠M BN = ∠M BC + ∠N BC = ∠M BC + ∠M BE =
To

BN BC
∠EBC. Từ đó 4BM N ∼ 4BEC, vì vậy nên ∠BM N = ∠BEC = 90◦ .
bộ

Câu 5
Cho tập X thỏa mãn tính chất sau: tồn tại 2019 tập con A1 , A2 , · · · , A2019 của X
c

sao cho mỗi tập con A1 , A2 , · · · , A2019 có đúng ba phần tử và hai tập con Ai , Aj đều
có đúng một phần tử chung với mọi 1 ≤ i < j ≤ 2019.
lạ

1. Chứng minh rằng tồn tại 4 tập hợp trong các tập A1 , A2 , · · · , A2019 sao cho
giao của 4 tập hợp này có đúng một phần tử.
u

2. Chứng minh rằng số phần tử của X phải lớn hơn hoặc bằng 4039.

Lời giải. 1. Xét tập A1 = {a, b, c}, do giao của hai tập có đúng một phần tử nên 2008
tập con còn lại phải chứa đúng 1 trong 3 phần tử a, b, c. Do đó tồn tại ba tập con
cùng chứa một phần tử, giả sử là a và ba tập con đó cùng với A tạo ra bốn tập con
thỏa mãn bài toán.
(Thực ra ta có thể chứng minh có ít nhất 674 tập có đúng một phần tử chung).
62 Tuyển tập đề thi vào lớp 10 chuyên Toán

2. Gọi a là phần tử thuộc vào ít nhất 4 tập con. Ta chứng minh a thuộc vào tất cả các
tập hợp. Thật vậy, giả sử phản chứng tồn a ∈ A1 , A2 , A3 , A4 nhưng a ∈ / A5 . Vì A5
đều có chung với bốn tập A1 , A2 , A3 , A4 một phần tử nên A5 chứa ít nhất 4 phần
tử, vô lý. Do vậy, a thuộc vào tất cả các tập. Gọi Bi là tập sau khi bỏ khỏi Ai phần
tử a. Thế thì các tập Bi đôi một giao nhau bằng rỗng. Suy ra số phần tử trong
hợp các tập B1 , B2 , · · · , B2019 là 2 × 2019. Suy ra nếu gọi n là số phần tử của X thì
n ≥ 2.2019 + 1 = 4039 vì có thêm phần tử a nữa.

iế n
K

-T
A1
á n
To
bộ
c
lạ
u

Nguyễn Tiến Lâm - Trịnh Huy Vũ 63

2.7. Đề thi tuyển sinh vào lớp 10 chuyên Toán Hà


Nội, năm 2019

Câu 1
√ √ √
1. Giải phương trình ( x + 5 − x)(1 + x2 + 5x) = 5.

iế n
(
x2 + 7 = 4y 2 + 4y
2. Giải hệ phương trình
x2 + 3xy + 2y 2 + x + y = 0.

K
Lời giải.


√ √

-T
1. Điều kiện: x ≥ 0. Với điều kiện này thì x + x + 5 > 0, nên phương trình đã cho
có thể viết lại dưới dạng
p √ √
1 + x(x + 5) = x + x + 5
A1
hay tương đương
√ √
( x − 1)( x + 5 − 1) = 0.
Giải phương trình trên và so sánh điều kiên, ta tìm được đúng một nghiệm là x = 1.
n

2. Phương trình thứ hai có thể viết lại (x + y)(x + 2y + 1) = 0. Từ đó ta có hai trường
á

hợp x = −y hoặc x = −2y − 1. Thay từng trường hợp vào phương trình đầu, ta tìm
To

được hai nghiệm của hệ phương trình là


 
7 7
(−1; 1), ;−
3 3
bộ
c

Câu 2
lạ

1. Cho biểu thức P = abc(a − 1)(b + 4)(c + 6) với a, b, c là các số nguyên thỏa
mãn a + b + c = 2019. Chứng minh rằng giá trị của biểu thức P chia hết cho
u

6.

2. Tìm tất cả các số tự nhiên n sao cho giá trị của biểu thức
√ √
q
Q= n+2+ n+ n+2

là số nguyên.

Lời giải.
64 Tuyển tập đề thi vào lớp 10 chuyên Toán

1. a và a − 1 là hai số nguyên liên tiếp nên trong hai số này luôn có một số chẵn, nghĩa
là P luôn chia hết cho 2.
Nếu trong ba số a, b, c có một số chia hết cho 3 thì P chia hết cho 3. Nếu cả ba số
a, b, c đều không chia hết cho 3 thì vì a + b + c chia hết cho 3 nên a, b, c phải có cùng
dư khi chia cho 3. Nếu a, b, c cùng dư 1 khi chia cho 3 thì a − 1 chia hết cho 3, nếu
a, b, c cùng dư 2 khi chia cho 3 thì b + 4 chia hết cho 3. Trong mọi tình huống thì P
luôn chia hết cho 3.

iế n
Mà (2, 3) = 1 nên P chia hết cho 2.3 = 6.

2. Vì Q nhận giá trị tự nhiên và dễ thấy Q > 0 nên ta viết

K
√ √
n + n + 2 = Q2 − 2Q n + 2 + n + 2,


kéo theo

-T
Q2 + 2
n+2= .
2Q + 1
(Q2 + 2)2
Suy ra n + 2 = . Do đó 2Q + 1 phải là ước của Q2 + 2. Từ đây, ta có
(2Q + 1)2
A1
4(Q2 + 2) = 4Q2 − 1 + 9 chia hết cho 2Q + 1, nên 9 chia hết cho 2Q + 1. Tìm được
Q = 1 hoặc Q = 4. Nhưng nếu Q = 1 thì n = −1 không thỏa mãn, vậy Q = 4,
tương ứng với n = 2.
Vậy, n = 2 là số tự nhiên duy nhất thỏa mãn bài toán.
á n
To

Câu 3
Cho biểu thức K = ab + 4ac − 4bc, với a, b, c là các số thực không âm thỏa mãn
a + b + 2c = 1.
bộ

1
1. Chứng minh K ≥ − .
2
2. Tìm giá trị lớn nhất của biểu thức K.
c
lạ

Lời giải.
1. Ta có 1 = a + b + 2c ≥ b + 2c nên suy ra 1 ≥ (b + 2c)2 ≥ 4 · b · 2c = 8bc. Suy ra
u

1 1
−4bc ≥ − . Từ đây với chú ý a, b, c ≥ 0 nên suy ra ngay K ≥ − .

2 2
2. Ta có

K = a(b + 2c) + 2ac − 4bc = a(1 − a) + 2ac − 4bc ≤ a(1 − a) + 2ca.

Từ điều trên, áp dụng bất đẳng thức AM − GM, ta suy ra

(a + 1 − a)2 (2c + a)2 1 (2c + a + b)2 1


K≤ + ≤ + = .
4 4 4 4 2
Nguyễn Tiến Lâm - Trịnh Huy Vũ 65

1 1
Dấu bằng xảy ra khi và chỉ khi b = 0, 2c = a, a + b + 2c = 1, hay a = , b = 0, c = .
2 4
1
Vậy giá trị lớn nhất của K là .
2

Câu 4
Cho tam giác ABC có ba góc nhọn (AB < AC), nội tiếp đường tròn (O). Gọi I

iế n
là tâm đường tròn nội tiếp tam giác ABC. Tia AI cắt đoạn thẳng BC tại điểm J,
cắt đường tròn (O) tại điểm thứ hai M (M khác A).

K
1. Chứng minh rằng M I 2 = M J · M A.

2. Kẻ đường kính M N của đường tròn (O). Đường thẳng AN cắt các tia phân


giác trong của góc ABC và góc ACB lần lượt tại các điểm P và Q. Chứng

-T
minh rằng N là trung điểm của đoạn thẳng P Q.

3. Lấy điểm E bất kỳ thuộc cung nhỏ M C của đường tròn (O) (E khác M ).
Gọi F là điểm đối xứng với điểm I qua điểm E. Gọi R là giao điểm của hai
đường thẳng P C và QB. Chứng minh rằng bốn điểm P, Q, R, F cùng nằm
trên một đường tròn.
A1
P
á n

A N
To

X
Q

Y L
bộ

I O

B
c

J C
lạ

E
M
u

F

Lời giải. 1. Ta có ∠M BJ = ∠M BC = ∠M AC = ∠M AB suy ra 4M JB ∼ 4M BA.


Từ đó ta được M B 2 = M J · M A. Mặt khác, ta lại có ∠M IB = ∠IAB + ∠IBA =
∠M BJ + ∠IBC = ∠IBM , vì vậy tam giác M IB cân tại M và M I = M B. Do đó,
M I 2 = M B 2 = M J · M A.
66 Tuyển tập đề thi vào lớp 10 chuyên Toán

2. Ta có P, Q lần lượt là tâm đường tròn bàng tiếp ứng với các đỉnh B, C của tam giác
ABC. Từ đó ta có CP là phân giác ngoài của 4ABC, ∠P CQ = 90◦ và tứ giác
AIBQ nội tiếp. Như vậy, ∠CN P = ∠ABC = 2∠ABI = 2∠AQI = 2∠N QC. Suy
ra N Q = N C, chú ý rằng tam giác CP Q cân tại C, ta thu được N là trung điểm
của P Q.

3. Ta có R là tâm đường tròn bàng tiếp ứng với đỉnh A của tam giác ABC. Từ

iế n
đó tam giác IBR vuông tại B và kết hợp với M B = M I (chứng minh ở ý 1)
thì ta được M là trung điểm của IR. Tương tự các trung điểm X, Y của các
cung nhỏ CA, AB của đường tròn (O) tương ứng là trung điểm của IP, IQ. Lấy

K
điểm L đối xứng I qua O. Khi đó, OE, OM, OX, OY là các đường trung bình
của các tam giác ILF, ILR, ILP, ILQ. Từ đó suy ra LF = LP = LQ = LR
(= 2OE = 2OX = 2OY = 2OM ). Vậy P, Q, R, F cùng thuộc đường tròn tâm L.


-T
Câu 5
Mỗi điểm trong mặt phẳng được tô bởi một trong hai màu xanh hoặc đỏ.
A1
1. Chứng minh trong mặt phẳng đó tồn tại hai điểm được tô bởi cùng một màu
và có khoảng cách bằng d.

2. Gọi tam giác có ba đỉnh được tô cùng một màu là tam giác đơn sắc. Chứng
n
minh trong mặt phẳng đó tồn tại hai tam giác đơn sắc là hai tam giác vuông
1
á

đồng dạng với nhau theo tỉ số k = .


2019
To

F E
bộ
c
lạ

A B
u

C D

Lời giải. 1. Câu này quen thuộc, có thể tìm thấy ở bất kỳ tài liệu nào. Chúng tôi
không trình bày lời giải ở đây.
Nguyễn Tiến Lâm - Trịnh Huy Vũ 67

2. Theo ý trên thì với mọi số thực dương d luôn tồn tại hai điểm được tô cùng màu có
khoảng cách bằng d. Thế thì tồn tại hai điểm A, B có khoảng cách là 2019 được tô
cùng màu, giả sử là màu đỏ. Dựng đường tròn đường kính AB. Dựng lục giác đều
ACDBEF. Trong các điểm C, D, E, F nếu có một điểm được tô đỏ, giả sử là C thì
tam giác ABC là tam giác đơn sắc nửa đều có ba đỉnh được tô đỏ. Ngược lại, nếu
cả 4 điểm C, D, E, F được tô xanh thì tam giác CDE là tam giác đơn sắc nửa đều
có ba đỉnh được tô xanh. Điều này có nghĩa là ta chứng minh được tồn tại một tam

iế n
giác đơn sắc nửa đều nội tiếp đường tròn đường kính 2019, giả sử là P QR.
Tương tự, cũng tồn tại hai điểm M, N được tô cùng màu và M N = 1. Chứng minh
tương tự như trên, ta cũng chỉ ra được tồn tại một tam giác đơn sắc nửa đều nội

K
tiếp đường tròn đường kính 1, giả sử là XY Z.
1
Rõ ràng XY Z và P QR là hai tam giác đơn sắc đồng dạng với tỉ số k = .
2019


-T
A1
á n
To
bộ
c
lạ
u

68 Tuyển tập đề thi vào lớp 10 chuyên Toán

2.8. Đề thi tuyển sinh vào lớp 10 trường THPT


chuyên Khoa học Tự nhiên năm 2020, vòng 1

Câu 1
1. Giải hệ phương trình

iế n
(
x2 + y 2 + xy = 7
9x3 = xy 2 + 70(x − y).

K
2. Giải phương trình
√ √


p
11 5 − x + 8 2x − 1 = 24 + 3 (5 − x)(2x − 1).

-T
Lời giải.
1. Từ hệ phương trình ta suy ra A1
9x3 = xy 2 + 10(x2 + y 2 + xy)(x − y) = xy 2 + 10(x3 − y 3 ).

Từ đó ta được x3 + xy 2 − 10y 3 = 0 ⇐⇒ (x − 2y)(x2 + 2xy + 5y 2 ) = 0. Chú ý rằng


x2 + 2xy + 5y 2 = (x + y)2 + 4y 2 ≥ 0. Dấu bằng xảy ra khi x = y = 0, tuy nhiên
n
điều này mâu thuẫn với phương trình thứ nhất của hệ phương trình đã cho. Như
vậy, x2 + 2xy + 5y 2 > 0 và do đó x = 2y. Thay x = 2y vào phương trình thứ nhất
á

ta suy ra 7y 2 = 7 hay y = ±1. Vậy hệ phương trình đã cho có hai cặp nghiệm (x, y)
To

là (2, 1) và (−2, −1).


1 √ √
2. Điều kiện xác định: ≤ x ≤ 5. Đặt ẩn phụ a = 5 − x, b = 2x − 1 với a, b ≥ 0. Từ
2
bộ

đó ta có 2a2 +b2 = 9. Khi đó, ta có 24+ab = 15+2a2 +b2 +3ab = 15+(2a+b)(a+b).


Hơn nữa, cũng chú ý rằng 11a + 8b = 3(2a + b) + 5(a + b). Như vậy phương trình
ban đầu sẽ tương đương với
c

3(2a + b) + 5(a + b) = 15 + (2a + b)(a + b)


lạ

⇐⇒ (2a + b − 5)(a + b − 3) = 0.

Từ đó suy ra 2a + b = 5 hoặc a + b = 3.
u

Trường hợp 1. 2a + b = 5. Từ 2a2 + b2 = 9 suy ra 2a2 + (5 − 2a)2 = 9 ⇐⇒


(a − 2)(3a − 4) = 0. Dẫn đến a = 2 hoặc a = 34 . Giải ra ta thu được hai nghiệm


x = 1 và x = 29
9
.

Trường hợp 2. a+b = 3. Từ 2a2 +b2 = 9 suy ra 2a2 +(3−a)2 = 9 ⇐⇒ 3a(a−2) = 0.


Dẫn đến a = 0 hoặc a = 2. Giải ra ta thu được hai nghiệm x = 5 và x = 29
9
.
 
29
Kết luận. S = 1, , 5 .
9
Nguyễn Tiến Lâm - Trịnh Huy Vũ 69

Câu 2
1. Tìm x, y nguyên dương thỏa mãn

x2 y 2 − 16xy + 99 = 9x2 + 36y 2 + 13x + 26y.

2. Với a, b là các số thực dương thỏa mãn

iế n
2 ≤ 2a + 3b ≤ 5, 8a + 12b ≤ 2a2 + 3b2 + 5ab + 10.

Chứng minh rằng

K
3a2 + 8b2 + 10ab ≤ 21.


Lời giải.

-T
1. Phương trình tương đương với

(xy + 10)2 = (3x + 6y)2 + 13(x + 2y) + 1. (2.6)

Do x, y là các số nguyên dương nên ta có x + 2y ≥ 3. Vì thế nên ta có


A1
(3x + 6y)2 + 13(x + 2y) + 1 ≥ (3x + 6y + 2)2


(3x + 6y)2 + 13(x + 2y) + 1 < (3x + 6y + 3)2 .
n

Do vế phải của (2.6) là số chính phương nên 9(x+2y)2 +13(x+2y)+1 = (3x+6y+2)2 .


á

Từ đó suy ra x + 2y = 3. Vì x, y là các số nguyên dương nên x = y = 1. Như vậy


To

phương trình có một cặp nghiệm là (x, y) = (1, 1).


2. Cách 1. Đặt 2a + 3b = x và a + b = y. Từ giả thiết ta thu được 2 ≤ x ≤ 5 và
4x − 10
4x ≤ xy + 10. Vì vậy nên y ≥ . Trong khi đó ta có
bộ

x
P = 3a2 + 8b2 + 10ab = (a + 2b)(3a + 4b) = (x − y)(x + y) = x2 − y 2 .

Ta cần chứng minh rằng x2 − y 2 ≤ 21.


c
lạ

5
Trường hợp 1. Nếu x < 2
thì x2 < 21 và vì thế nên x2 − y 2 < 21.
 2
4x − 10
u

Trường hợp 2. Nếu x > 52 thì 4x−10 ≥ 0. Do đó, ta suy ra được y 2 ≥


x

 2
4x − 10
=⇒ x2 − y 2 ≤ x2 − . Như vậy ta chỉ cần chứng minh bất đẳng thức sau
x
 2
2 4x − 10
x − ≤ 21 ⇐⇒ x4 − 37x2 + 80x − 100 ≤ 0.
x

Bất đẳng thức này tương đương với

(x − 5)(x3 + 5x2 − 12x + 20) ≤ 0.


70 Tuyển tập đề thi vào lớp 10 chuyên Toán

6 2
 64
Bất đẳng thức này luôn đúng do x ≤ 5 và x3 +5x2 −12x+20 = x3 +5 x − 5
+5 >
0. Như vậy ta có điều phải chứng minh.

Cách 2. Từ giả thiết ta suy ra

8a + 12b ≤ (2a + 3b)(a + b) + 10 ≤ 5(a + b) + 10 =⇒ 3a + 7b ≤ 10.

iế n
Từ đó ta có
1
3a2 + 8b2 + 10ab = (3a + 4b)(a + 2b) = (9a + 12b)(7a + 14b)
21

K
 2
1 9a + 12b + 7a + 14b 1
≤ = (16a + 26b)2
21 2 84
 2  2


1 34 4 1 34 4
= (2a + 3b) + (3a + 7b) ≤ · 5 + · 10 = 21.

-T
84 5 5 84 5 5

Câu 3
A1
Cho tam giác ABC có ∠BAC là góc nhỏ nhất trong ba góc của tam giác và nội
tiếp đường tròn (O). Điểm D thuộc cạnh BC sao cho AD là phân giác của ∠BAC.
Lấy các điểm M, N thuộc (O) sao cho các đường thẳng CM và BN cùng song song
với đường thẳng AD.
á n

1. Chứng minh rằng AM = AN .


To

2. Gọi giao điểm của đường thẳng M N với các đường thẳng AC, AB lần lượt là
E, F . Chứng minh rằng bốn điểm B, C, E, F cùng thuộc một đường tròn.

3. Gọi P, Q theo thứ tự là trung điểm của các đoạn thẳng AM, AN . Chứng minh
bộ

rằng EQ, F P, AD đồng quy.


c

Lời giải.
lạ

1. Do AD là phân giác ∠BAC và BN, CM cùng song song với AD nên ta có ∠ABN =
∠BAD = ∠CAD = ∠ACM . Từ đó suy ra số đo các cung nhỏ AM và AN của
đường tròn (O) là bằng nhau, vì vậy AM = AN .
u

2. Ta có ∠AEF = ∠EAM + ∠AM N = ∠M BC + ∠AN M = ∠M BC + ∠ABM =


∠ABC. Do đó, tứ giác BCEF là một tứ giác nội tiếp.

3. Đặt AD cắt đường tròn (O) lần thứ hai tại điểm X. Khi đó ta có X là trung điểm của
cung BC không chứa A của (O). Gọi J là trung điểm của AX. Ta có JQ k XN (tính
chất đường trung bình). Trong khi đó, vì ∠N XA = ∠ABN = ∠BAD = ∠CAD,
nên XN k AC. Như vậy ta thu được JQ k AC. Chứng minh tương tự ta cũng được
XM k AB và JP k AB. Mặt khác, từ XN k AE và XM k AF , ta cũng suy ra
được 4XM N ∼ 4AF E (gg) và XN AE
= XMAF
.
Nguyễn Tiến Lâm - Trịnh Huy Vũ 71

A
M
P
G
Q E
N F

iế n
O
J

K

-T
D
B C
A1
X

GJ JQ XN XM JP
Đặt EQ ∩ AD ≡ G. Từ AE k JQ ta suy ra GA = AE = 2AE = 2AF = AF . Từ đó
n

ta thu được 4GJP ∼ 4GAF (cgc). Vì vậy, ∠AGF = ∠JGP = 180 − ∠AGP hay
∠F GP = 180◦ . Như vậy, F, G, P thẳng hàng và do đó ba đường thẳng EQ, F P, AD
á

đồng quy tại G.


To

Câu 4
bộ

Cho a, b, c là các số thực dương thỏa mãn a + b + c = 3. Chứng minh rằng

a(a + bc)2 b(b + ca)2 c(c + ab)2


+ + ≥ 4.
b(ab + 2c2 ) c(bc + 2a2 ) a(ca + 2b2 )
c
lạ

Lời giải. Đặt


a(a + bc)2 b(b + ca)2 c(c + ab)2
P = + + .
u

b(ab + 2c2 ) c(bc + 2a2 ) a(ca + 2b2 )


Áp dụng bất đẳng thức Cauchy - Schwarz dạng phân thức ta có


(a2 + abc)2 (b2 + abc)2 (c2 + abc)2
P = + +
ab(ab + 2c2 ) bc(bc + 2a2 ) ca(ca + 2b2 )
(a2 + b2 + c2 + 3abc)2
≥ 2 2
a b + b2 c2 + c2 a2 + 2abc2 + 2a2 bc + 2ab2 c
(a2 + b2 + c2 + 3abc)2
= .
(ab + bc + ca)2
72 Tuyển tập đề thi vào lớp 10 chuyên Toán

a2 + b2 + c2 + 3abc
Tiếp theo ta cần chứng minh ≥ 2. Ta có hai cách cho bước này.
ab + bc + ca

Cách 1: Ta trước hết nêu ra bất đẳng thức sau

(a + b + c)3 + 9abc ≥ 4(a + b + c)(ab + bc + ca). (2.7)

Bất đẳng thức trên tương đương với

iế n
X
a(a − b)(a − c) ≥ 0.
cyc

K
Đây chính là bất đẳng thức Schur.
12q − 27 4q − 9
Đặt q = ab + bc + ca, r = abc. Từ bất đẳng thức (2.7) ta suy ra r ≥ = .


9 3
Khi đó

-T
a2 + b2 + c2 + 3abc 9 − 2q + 3r
=
ab + bc + ca q
9 − 2q + 4q − 9
≥ = 2.
A1 q

Ta có điều phải chứng minh.

Cách 2: Trong ba số a − 1, b − 1, c − 1 phải có hai số cùng dấu (theo nguyên lý Dirichlet),


n

không mất tính tổng quát giả sử hai số dương đó là b − 1, c − 1. Khi đó


á

a(b − 1)(c − 1) ≥ 0
To

dẫn đến abc ≥ ab + ac − a.


Do đó ta có
bộ

a2 + b2 + c2 + 3abc ≥ a2 + b2 + c2 + 3ab + 3ac − 3a


= a2 + b2 + c2 + 3ab + 3ac − a(a + b + c)
= b2 + c2 + 2ab + 2ac
c

= (b − c)2 + 2(ab + bc + ca) ≥ 2(ab + bc + ca).


lạ

Ta có điều phải chứng minh.


u

Nguyễn Tiến Lâm - Trịnh Huy Vũ 73

2.9. Đề thi tuyển sinh vào lớp 10 trường THPT


chuyên Khoa học Tự nhiên năm 2020, vòng 1

Câu 1
1. Giải hệ phương trình

iế n
(
(x + y)(x + 1) = 4
.
(y 2 + xy + x + y + 5)(x3 + y 3 + 12y + 13) = 243.

K
2. Giải phương trình


(x − 12)7 + (2x − 12)7 + (24 − 3x)7 = 0.

-T
Lời giải.
1. Từ đề bài suy ra x2 + xy + x + y = 4. Suy ra
A1
(y 2 + xy + x + y + 5)(x3 + y 3 + 12y + 13)

= (y 2 +xy+x+y+x2 +xy+x+y+1)(x3 +y 3 +3y(x2 +xy+x+y)+3(x2 +xy+x+y)+1)


n

= (x + y + 1)5 .
á

Từ đó, ta phải có x + y = 2. Giải tìm được x = y = 1.


To

2. Đặt a = x − 12, b = 2x − 12, c = 24 − 3x thế thì a + b + c = 0. Từ đây, ta có

• a7 + b7 = (a3 + b3 )(a4 + b4 ) + a3 b3 c,
bộ

• a3 + b3 = (a + b)3 − 3ab(a + b) = −c3 + 3abc,


• a4 + b4 = (a2 + b2 )2 − 2a2 b2 = c4 − 4abc2 + 2a2 b2 .
c

Từ các đẳng thức trên, suy ra


lạ

a7 + b7 = (−c3 + 3abc)(c4 − 4abc2 + 2a2 b2 ) + 3a3 b3 c.


u

Khai triển, ta thu được


a7 +b7 +c7 = 7abc(c2 (c2 −2ab)+a2 b2 ) = 7abc(c2 (a2 +b2 )+a2 b2 ) = 7abc(a2 b2 +b2 c2 +c2 a2 ).

Theo giả thiết vì a7 + b7 + c7 = 0 nên từ đẳng thức trên ta phải có ít nhất một trong
ba thừa số a, b, c bằng 0 hoặc ab = bc = ca = 0. Tuy nhiên trường hợp hai không
thể xảy ra, do đó ta phải có ít nhất một trong ba thừa số a, b, c bằng 0. Từ đây tìm
được x ∈ {12, 6, 8} và các giá trị này thỏa mãn. Tóm lại, phương trình đã cho có ba
nghiệm là 6, 8, 12.
74 Tuyển tập đề thi vào lớp 10 chuyên Toán

Câu 2
1. Tìm tất cả các số nguyên dương a, b, c sao cho cả ba số 4a2 +5b, 4b2 +5c, 4c2 +5a
đều là bình phương của số nguyên dương.

2. Từ một bộ bốn số thực (a, b, c, d) ta xây dựng bộ số mới (a+b, b+c, c+d, d+a)
và liên tiếp xây dựng các bộ số mới theo quy tắc trên. Chứng minh rằng nếu
có hai thời điểm khác nhau ta thu được cùng một bộ số (có thể khác thứ tự)

iế n
thì bộ số ban đầu phải có dạng (a, −a, a, −a).

K
Lời giải.
1. Không mất tổng quát, giả sử a = max{a, b, c}. Thế thì vì (2a)2 < 4a2 +5b < (2a+2)2
16a + 9


nên 4a2 + 5b = (2a + 1)2 . Suy ra 5b = 4a + 1. Tương tự thì 5c ≥ 4b + 1 =
5

-T
25c − 9
hay a ≤ < 2c. Từ đó
16
(2c)2 < 4c2 + 5a < (2c + 3)2
A1
nên ta phải có 4c2 + 5a ∈ {(2c + 1)2 , (2c + 2)2 }. Từ đây, xét hai trường hợp

• Nếu 4c2 + 5a = (2c + 1)2 thì 5a = 4c + 1 và do a ≥ c nên trong trường hợp này
thì a = b = c = 1.
128 64
n

• Nếu 4c2 +5a = (2c+2)2 thì 5a = 8c+4. Khi đó 16a = c+ > 25c ≥ 16a+9
5 5
á

vô lí.
To

Tóm lại a = b = c = 1.

2. Giả sử ở thời điểm thứ n ta thu được bộ số (an , bn , cn , dn ) và nếu đặt Sn = an +


bn + cn + dn thì Sn = 2Sn−1 nên suy ra Sn = 2n S0 với S0 = a + b + c + d. Vì tồn tại
bộ

hai thời điểm ta thu được cùng một bộ số nên S0 = 0, kéo Sn = 0 với mọi n ∈ N.
Nếu đặt Pn = a2n + b2n + c2n + d2n thì
2
Pn+1 = 2Pn + 2(an + cn )(bn + dn ) = 2Pn + 2Sn−1 = 2Pn ∀n ≥ 1.
c
lạ

Suy ra Pn = 2n−1 P1 với mọi n ≥ 2. Cũng vì tồn tại hai thời điểm thu được hai bộ
số giống nhau nên P1 = 0, suy ra a1 = b1 = c1 = d1 = 0. Điều đó có nghĩa là bộ số
ban đầu phải là (a, −a, a, −a).
u

Nguyễn Tiến Lâm - Trịnh Huy Vũ 75

Câu 3
Cho tam giác ABC cân tại A với ∠BAC < 90◦ . Điểm E thuộc cạnh AC sao cho
∠AEB > 90◦ . Gọi P là giao điểm của BE và trung trực của BC. Gọi K là hình
chiếu vuông góc của P lên AB. Gọi Q là hình chiếu vuông góc của E lên AP . Gọi
giao điểm của EQ và P K là F .

1. Chứng minh rằng bốn điểm A, E, P, F cùng thuộc một đường tròn.

iế n
2. Gọi giao điểm của KQ và P E là L. Chứng minh rằng LA vuông góc với LE.

3. Gọi giao điểm của F L và AB là S. Gọi giao điểm của KE và AL là T . Lấy R

K
là điểm đối xứng của A qua L. Chứng minh rằng đường tròn ngoại tiếp tam
giác AST và đường tròn ngoại tiếp tam giác BP R tiếp xúc với nhau.


-T
A

A1
L
n
S Q T
F
á

E
G
To

K
x X R
P
bộ

C
c

B
lạ
u

Lời giải. 1. Do ∠AQF = ∠AKF = 90◦ nên tứ giác AQKF là tứ giác nội tiếp. Vì vậy

∠P F E = ∠KF Q = ∠KAQ = ∠EAQ. Do đó, tứ giác AEP F là tứ giác nội tiếp.

2. Do hai tứ giác AEP F và AQKF là các tứ giác nội tiếp nên ∠AEL = ∠AF K =
∠AQL. Như vậy tứ giác AQEL là tứ giác nội tiếp và vì thế nên ∠ALE = 90◦ .

3. Do các tứ giác AQKF và AQEL nội tiếp nên ta có P K · P F = P Q · P A = P E · P L.


Như vậy, tứ giác EKF L nội tiếp và ∠P KE = ∠P LS. Chú ý rằng ∠AKP =
∠ALE = 90◦ nên ta suy ra ∠ALS = ∠AKE. Do đó, tứ giác KSLT nội tiếp. Từ
đó suy ra ∠AST = ∠ALK = ∠AP K = ∠ABC, dẫn đến ST k BC.
76 Tuyển tập đề thi vào lớp 10 chuyên Toán

Gọi G là hình chiếu của P trên AC. Ta sẽ chứng minh ba điểm B, G, T thẳng hàng.
LE
Thật vậy, từ 4ELA ∼ 4EGP và 4BKP ∼ 4BLA, ta suy ra GE = PLA
G
= PLA
K
=
LB
KB
. Chú ý rằng KA = GA nên ta có

KA GE LB GE LB
· · = · = 1.
KB GA LE LE KB

Sử dụng định lý Ceva cho tam giác EAB ta suy ra EK, AL, BG đồng quy. Mà T

iế n
là giao điểm KE và AL, nên dẫn đến ba điểm B, G, T thẳng hàng.
Gọi X là giao điểm thứ hai của BG và đường tròn đường kính AP . Khi đó, từ

K
tứ giác nội tiếp AP XG và A, R đối xứng qua đường thẳng BP , ta có ∠P XB =
∠P AC = ∠P AB = ∠P RB và vì vậy X ∈ (BP R).
Trong khi đó, chú ý rằng ST k KG (cùng k BC) và B, X, G, T thẳng hàng, ta cũng


có ∠AXT = ∠AXG = ∠AKG = ∠AST nên X ∈ (AST ).

-T
Như vậy, điểm X nằm trên cả hai đường tròn (AST ) và (BP R). Khi đó, do ∠BLT =
∠90◦ nên ∠P BX + ∠XT A = 90◦ = ∠P XA. Kẻ tia tiếp tuyến Xx của đường tròn
(AST ) thì ta được ∠xXA = ∠XT A, dẫn đến ∠xXP = ∠P BX. Vì vậy Xx cũng
tiếp xúc với (BP R). Do đó, (AST ) tiếp xúc với đường tròn (BP R).
A1
Nhận xét. Ý 3 của bài toán này có thể phát biểu gọn lại thành bài toán sau:
n

Bài toán. Cho tam giác ABC cân tại A với ∠BAC < 90◦ . P là một điểm
á

bất kỳ nằm trên trung trực của BC. L, G lần lượt là hình chiếu của A, P trên
To

các đường thẳng BP, AC. Gọi T là giao điểm AL và BG. R là điểm đối xứng
với A qua L. Dựng điểm S trên cạnh AB sao cho ST k BC. Chứng minh
rằng hai đường tròn (AST ) và (BP R) tiếp xúc với nhau.
bộ

Câu 4
Cho a, b, c là các số thực dương thỏa mãn a + b + c = 3. Chứng minh rằng
c
lạ

 2  
1 1 1 4 a b c
3 + + −1 +1≥ +3 + + .
a b c abc bc ca ab
u

Lời giải. Đặt x = a1 , y = 1b , z = 1c , ta có

1 1 1
+ + = 3. (2.8)
x y z

Hơn nữa bất đẳng thức trở thành


 
2 xy yz zx
3(x + y + z − 1) + 1 ≥ 4xyz + 3 + + (2.9)
z x y
Nguyễn Tiến Lâm - Trịnh Huy Vũ 77

Đặt p = x + y + z, q = xy + yz + zx, r = xyz. Khi đó điều kiện (2.8) trở thành

q = 3r.

Hơn nữa ta có
 
xy yz zx 1 1 1
+ + = xyz + +
z x y x2 y 2 z 2

iế n
" 2 #
1 1 1 2 2 2
= xyz + + − − −
x y z xy yz zx

K
 
2p
=r 9−
r
= 9r − 2p


-T
cho nên bất đẳng thức (2.9) trở thành

3(p − 1)2 + 1 ≥ 4r + 3(9r − 2p).

Khai triển ra thì ta có bất đẳng thức tương đương với


A1
3p2 + 4 ≥ 31r.

Áp dụng bất đẳng thức Schur


n

(x + y + z)3 + 9xyz ≥ 4(x + y + z)(xy + yz + zx)


á
To

ta có
p3 + 9r ≥ 4pq = 12pr.
 
1 1 1 1 1 1
Vì (x + y + z) + + ≥ 9 và + + = 3 nên p = x + y + z ≥ 3. Thế thì từ
bộ

x y z x y z
p3 + 9r ≥ 12pr ta suy ra
31p3
≥ 31r.
3(4p − 3)
c

31p3
lạ

Ta sẽ chứng minh 3p2 + 4 ≥ , từ đây sẽ hoàn tất chứng minh. Bất đẳng thức
3(4p − 3)
này tương đương với
(p − 3)(5p2 − 12p + 12) ≥ 0.
u

Bất đẳng thức cuối cùng này luôn đúng vì p ≥ 3 và 5p2 − 12p + 12 > 0.

78 Tuyển tập đề thi vào lớp 10 chuyên Toán

2.10. Đề thi tuyển sinh vào lớp 10 trường THPT


chuyên Sư phạm Hà Nội năm 2020, vòng 2

Câu 1
Cho ba số thực x, y, z thỏa mãn các điều kiện sau

iế n

3
2x = 3y 3 = 4z 3
√ √

p3
2x2 + 3y 2 + 4z 2 = 2 + 3 12 + 3 16

K
xyz > 0.

1 1 1
Tính giá trị của biểu thức P = + + .


x y z

-T
Lời giải. Đặt 2x3 = 3y 3 = 4z 3 = t. Khi đó
s  
p
3 3 1 1 1
2x2 + 3y 2 + 4z 2 =
A1t + +
x y z
.

√ √
√ 3
t √ 3
t √ √
3
t
Mặt khác 3 2 = 3
, 3= , 34= z
nên
x y
n

√ √ √ √ √ √  √ √ √ √

á

3 3 3
  
3 3 3 3 3 3 3 t t t 3 1 1 1
2+ 12 + 16 = 4 2+ 3+ 4 = 4 + + = 4t + +
x y z x y z
To

Từ đó ta suy ra s 

  
3 1 1 1 3 1 1 1
t + + = 4t + + .
bộ

x y z x y z
1 1 1 1
Từ đó ta tìm được + + = .
x y z 2
c
lạ

Câu 2
Xét phương trình bậc hai ax2 + bx + c = 0 (1), trong đó a, b, c là các số nguyên
u

dương. Biết rằng các điều kiện sau được thỏa mãn: phương trình (1) có nghiệm, số
a2020b chia hết cho 12, số c2 + 3 chia hết cho c + 3. Hãy tìm giá trị lớn nhất của

tổng a + b + c.

(
3 | a + b + 4,
Lời giải. Vì a2020b chia hết cho 12 nên
4 | b.
Từ đó suy ra b ∈ {0, 4, 8}.
Lại có b2 − 4ac ≥ 0 nên b2 > 0 và ac ≤ 16. Suy ra b ∈ {4, 8} và c ≤ 16. Hơn nữa c + 3 là
ước của c3 + 3 = c3 + 27 − 24 = (c + 3)(c2 − 3c + 9) − 24 nên c + 3 là ước của 24. Kết hợp
Nguyễn Tiến Lâm - Trịnh Huy Vũ 79

với c + 3 ≥ 4 ta có c + 3 ∈ {4, 6, 8, 12, 24}, hay c ∈ {1, 3, 5, 9, 21}. Kết hợp với c ≤ 16 ta
có c ∈ {1, 3, 5, 9}.
Xét các trường hợp

ac ≤ 16,

• b = 8. Khi đó 3 | a,

c ∈ {1, 3, 5, 9}.

iế n

Thử tất cả các bộ số ta thấy a + c max khi (a, c) = (9, 1). Lúc này a + b + c = 18.

K

ac ≤ 4,

• b = 4. Khi đó a ≡ 1 (mod 3),



c ∈ {1, 3, 5, 9}.

-T
Thử tất cả các bộ số ta thấy a + c max khi (a, c) = (4, 1). Lúc này a + b + c = 9.

Từ hai trường hợp trên ta suy ra giá trị lớn nhất của a + b + c bằng 18, xảy ra tai
(a, b, c) = (9, 8, 1).
A1
Câu 3
n
Tìm số nguyên a bé nhất sao cho x4 + 2x2 − 4x + a ≥ 0 với mọi số thực x.
á
To

Lời giải. Ta chứng minh giá trị nguyên nhỏ nhất của a là 2.

Trước hết, ta chứng minh a = 2 thõa mãn điều kiện đề bài, nghĩa là x4 + 2x2 − 4x + 2 ≥ 0
với mọi x. Thật vậy, ta có
bộ

x4 + 2x2 − 4x + 2 = x4 + 2(x − 1)2 ≥ 0 ∀x.


c

Bây giờ ta chứng minh a ≤ 1 không thỏa mãn đề bài, nghĩa là tồn tại x để
lạ

x4 + 2x2 − 4x + a < 0 với a ≤ 1.


u

Ta chỉ cần chứng minh tồn tại x để


x4 + 2x2 − 4x + 1 < 0

1 8
Chọn x = thì x4 + 2x2 − 4x + 1 = − < 0.
3 81
Kết luận. amin = 2.
80 Tuyển tập đề thi vào lớp 10 chuyên Toán

Câu 4
Cho tam giác nhọn ABC nội tiếp đường tròn (O) có AB > BC. Một đường tròn
đi qua hai đỉnh A, C của tam giác ABC lần lượt cắt các cạnh AB, BC tại hai điểm
K, N (K, N khác các đỉnh của tam giác ABC.) Giả sử đường tròn (O) và đường
tròn ngoại tiếp của tam giác BKN cắt nhau tại giao điểm thứ hai là M (M khác
B). Chứng minh rằng

iế n
1. Ba đường thẳng BM, KN, AC đồng quy tại điểm P .

2. Tứ giác M N CP nội tiếp.

K
3. BM 2 − P M 2 = BK · BA − P C · P A.


-T
B

M
A1 K

N
O
n

J
á

P C A
To
bộ

Lời giải.
1. Gọi P là giao điểm của N K và BC, M 0 là giao điểm thứ hai (khác B) của P B và
c

đường tròn (O). Do các tứ giác ACN K và ABM 0 C nội tiếp nên ta được P M 0 ·P B =
lạ

P A · P C = P N · P K. Từ đó suy ra tứ giác BKN M 0 nội tiếp. Nói cách khác M 0 là


giao điểm khác B của hai đường tròn (O) và đường tròn (BKN ). Do đó, M ≡ M 0
dẫn đến ba đường thẳng BM, KN, AC đồng quy tại điểm P .
u

2. Từ các tứ giác nội tiếp ACN K và BKN M ta có ∠N CP = ∠N KA = ∠N M B. Do


vậy, tứ giác M N CP nội tiếp.
3. Từ các tứ giác nội tiếp ACN K, N M CP , ta được BK · BA = BN · BC = BM · BP .
Từ tứ giác nội tiếp ABM C ta suy ra P C · P A = P M · P B. Vì vậy ta thu được
BK · BA − P C · P A = BM · BP − P M · P B = (BM − P M )BP
= (BM − P M )(BM + P M ) = BM 2 − P M 2 .
Nguyễn Tiến Lâm - Trịnh Huy Vũ 81

Câu 5
Cho hai số A, B cùng có 2020 chữ số. Biết rằng số A có đúng 1945 chữ số khác 0,
bao gồm 1930 chữ số ngoài cùng về bên trái và 15 chữ số ngoài cùng về bên phải,
số B có đúng 1954 chữ số khác 0, bao gồm 1930 chữ số ngoài cùng về bên trái và 24
chữ số ngoài cùng về bên phải. Chứng minh rằng U CLN (A, B) là một số có không
quá 1954 chữ số.

iế n
Lời giải. Ta viết dạng tổng quát của hai số A và B như sau:

K
A = x0 . . . 0y, B = z0 . . . 0t,

trong đó x, z có 1930 chữ số, y có 15 chữ số, t có 24 chữ số, A và B đều có tổng cộng 2020


chữ số. Phân tích cấu tạo số của hai số A, B ta được

-T
A = 1090 x + y, B = 1090 z + t.

Đặt d = (A, B). Khi đó d là ước của xB − zA, tức là d là ước của xt − yz. Không khó để
thấy xt > yz, nên xt − yz > 0. Do đó d ≤ xt − yz < xt. Lại có x < 101930 , t < 1024 (do x
có 1930 chữ số và t có 24 chữ số) nên
A1
d < xt < 101930 .1024 = 101954 .
n
Từ đó ta suy ra d có không quá 1954 chữ số.
á
To
bộ
c
lạ
u

82 Tuyển tập đề thi vào lớp 10 chuyên Toán

2.11. Đề thi tuyển sinh vào lớp 10 chuyên Toán


Hà Nội, năm 2020

Câu 1

1. Giải phương trình x2 + 3x + 5 = (x + 3) x2 + 5.

iế n
2. Cho các số thực a, b, c thỏa mãn a + b − 2c = 0 và 2ab − bc − ca = 0. Chứng
minh rằng a = b = c.

K
Lời giải.


1. Phương trình ban đầu tương đương với

-T
√ √
x2 + 5 − x x2 + 5 + 3x − 3 x2 + 5 = 0.
√ √
⇐⇒ ( x2 + 5 − x)( x2 + 5 − 3) = 0.
√ √ √
A1
Từ đó ta suy ra x2 + 5 = x hoặc √x2 + 5 = 3. Giải phương trình x2 + 5 = x
suy ra vô nghiệm. Giải phương trình x2 + 5 = 3 suy ra x = ±2.
Vậy S = {±2}.
n

2. Chú ý rằng a + b = 2c thì ta có 2ab = bc + ca = c(a + b) = 2c2 hay ab = c2 . Từ


á

đó ta suy ra (a − b)2 = (a + b)2 − 4ab = (2c)2 − 4c2 = 0. Như vậy a = b. Thay vào
To

a + b = 2c ta suy ra a = b = c.

Câu 2
bộ

1. Chứng minh với mọi số nguyên dương n, số A = 11n + 7n − 2n − 1 chia hết


cho 15.
√ m
c

2. Cho hai số nguyên dương m và n thỏa mãn 11 − > 0. Chứng minh rằng
n
lạ


√ m 3( 11 − 3)
11 − ≥ .
n mn
u

.
Lời giải. 1. Chú ý rằng (an − bn ) .. (a − b) với a, b, n ∈ Z+ nên ta có

.
A = (11n − 2n ) + (7n − 1n ) .. 3,

.
A = (11n − 1n ) + (7n − 2n ) .. 5.
Từ đó ta có điều phải chứng minh.
Nguyễn Tiến Lâm - Trịnh Huy Vũ 83

2. Ta cần chứng minh bất đẳng thức sau



√ 3( 11 − 3)
11n − m ≥ .
m

Bất đẳng thức này tương đương với



√ 9( 11 − 3)2

iế n
2 2
11n ≥ m + 6( 11 − 3) + .
m2

Ta xét ba trường hợp như sau

K
√ √
• Nếu m = 1 thì V P = 1 + 6( 11 − 3) + 9( 11 − 3)2 < 11 ≤ 11n2 .

√ 9( 11 − 3)2


• Nếu m = 2 thì V P = 4 + 6( 11 − 3) + < 11 ≤ 11n2 .

-T
4

√ 9( 11 − 3)2
• Nếu m ≥ 3 thì V P ≤ m + 6( 11 − 3) +
2
= m2 + 2.
9
Ta sẽ chứng minh 11n2 ≥ m2 + 2 ⇐⇒ 11n2 − m2 ≥ 2.

Thật vậy, từ giả thiết 11 − mn
A1
> 0 ta suy ra 11n2 − m2 > 0 và do đó
11n2 − m2 ≥ 1. Nếu 11n2 − m2 = 1 thì m2 chia 11 dư 10, tuy nhiên điều này
không thể xảy ra (Kiểm tra bằng đồng dư). Từ đó suy ra 11n2 − m2 ≥ 2.
Chứng minh của bài toán hoàn tất.
á n
To

Câu 3
1. Cho đa thức P (x) với hệ số thực thỏa mãn P (1) = 3 và P (3) = 7. Tìm đa
thức dư trong phép chia đa thức P (x) cho đa thức x2 − 4x + 3.
bộ

2. Với a, b, c là các số thực không âm thỏa mãn a + b + c + abc = 4, tìm giá trị
lớn nhất của biểu thức P = ab + bc + ca.
c

Lời giải.
lạ

1. Hiển nhiên deg P ≥ 1. Khi đó ta có thể viết


u

P (x) = (x − 1)(x − 3)Q(x) + ax + b


với a, b ∈ R. Thay x = 1 và x = 3 vào phương trình trên ta có


(
a + b = P (1) = 3
3a + b = P (3) = 7.

Giải hệ phương trình ta được a = 2, b = 1. Vậy dư của P (x) trong phép chia cho
x2 − 4x + 3 = (x − 1)(x − 3) là 2x + 1.
84 Tuyển tập đề thi vào lớp 10 chuyên Toán

4 − (a + b)
2. Cách 1. Ta có a + b + c + abc = 4 nên c = (1).
ab + 1
Dự đoán: Dấu đẳng thức xảy ra khi a = b = 2, c = 0. Khi đó P = 4.
Ta cần chứng minh P ≤ 4, tương đương với ab + c(a + b) ≤ 4. Thay (1) vào và biến
đổi thì bất đẳng thức tương đương với

(ab + 1)(ab − 4) ≤ (a + b)(a + b − 4).

iế n
Giả sử (a − 1)(b − 1) ≥ 0 (Dirichlet) thì ab + 1 ≥ a + b ≥ 0.

Do 4 = a + b + c + abc ≥ a + b nên a + b ≤ 4. Mặt khác 4 ≥ a + b ≥ 2 ab, nên

K

ab ≤ 4. Khi đó a + b ≥ 2 ab ≥ ab, cho nên 4 − ab ≥ 4 − a − b.
Khi đó (ab + 1)(4 − ab) ≥ (a + b)(4 − a − b), bất đẳng thức đã được chứng minh


xong. Dấu bằng xảy ra, ví dụ tại a = b = 2, c = 0.

-T
Cách 2. Không mất tính tổng quát, giả sử a = max{a, b, c}. Từ giả thiết ta suy ra
a3 + 3a ≥ 4. Bất đẳng thức này tương đương với (a − 1)(a2 + a + 4) ≥ 0. Lại do
a2 + a + 4 > 0 nên ta suy ra a ≥ 1 hay a2 ≥ 1. Từ đó suy ra a2 bc ≥ bc. Như vậy,
A1
P = ab + bc + ca = a(b + c) + bc ≤ a(b + c) + a2 bc
(a + b + c + abc)2
= a(b + c + abc) ≤ = 4.
4
n

Dấu bằng xảy ra khi (a, b, c) là hoán vị của (2, 2, 0).


á
To

Câu 4
Cho tam giác ABC có ba góc nhọn và AB < AC. Gọi (I) là đường tròn nội tiếp
của tam giác ABC và K là tâm đường tròn bàng tiếp trong góc A của tam giác
bộ

ABC. Gọi D, E, F lần lượt là chân các đường vuông góc kẻ từ điểm I đến các
đường thẳng BC, CA, AB. Đường thẳng AD cắt đường tròn (I) tại hai điểm phân
biệt D và M . Đường thẳng qua K song song với đường thẳng AD cắt đường thẳng
c

BC tại N .
lạ

1. Chứng minh rằng tam giác M F D đồng dạng với tam giác BN K.

2. Gọi P là giao điểm của BI và F D. Chứng minh rằng ∠BM F = ∠DM P .


u

3. Chứng minh rằng đường tròn ngoại tiếp của tam giác M BC đi qua trung

điểm của đoạn thẳng KN .

Lời giải.
1. Trước hết để ý rằng D, E, F chính là ba tiếp điểm của (I) với BC, CA, AB. Ta có
F D k BK (cùng ⊥ BI) nên ∠F M D = ∠F DB = ∠N BK. Chú ý rằng N K k AD
nên ta được ∠BN K = ∠ADC = ∠M F D. Từ đó suy ra 4M F D ∼ 4BN K (gg).
Nguyễn Tiến Lâm - Trịnh Huy Vũ 85

M
E

iế n
G
I
F

K
P

B D N C


-T
X
A1
K
n

2. Bổ đề. Cho tam giác ABC với đường cao AH (H ∈ BC) và tâm ngoại tiếp O. Khi
á

đó, ∠HAB = ∠OAC.


To

Gọi G là hình chiếu của M trên F D. Áp dụng hệ thức lượng trong tam giác vuông
thì ta có IM 2 = ID2 = IP · IB. Từ đó suy ra IM
IP
IB
= IM , kéo theo 4IM P ∼ 4IBM
(cgc). Như vậy, ∠IM P = ∠IBM = ∠GM B. Mặt khác, áp dụng bổ đề trên cho
bộ

tam giác M F D ta suy ra ∠GM F = ∠IM D. Vì vậy, ta được

∠BM F = ∠GM B − ∠GM F = ∠IM P − ∠IM D = ∠DM P.


c
lạ

3. Để ý rằng từ câu 2 ta suy ra được ∠F M P = ∠DM B. Gọi X là trung điểm của


KN . Chú ý rằng P là trung điểm của F D và từ câu 1 ta có 4M F D ∼ 4BN K
u

với hai trung tuyến tương ứng là M P và BX. Từ đó suy ra 4BN X ∼ 4M F P


nên ta thu được ∠XBC = ∠F M P = ∠DM B. Chứng minh tương tự câu 1 ta


có 4M ED ∼ 4CN K và tiếp tục chứng minh tương tự như trên ta thu được
∠XCB = ∠DM C. Như vậy, ∠BM C + ∠BXC = ∠DM B + ∠DM C + ∠BXC =
∠XBC + ∠XCB + ∠BXC = 180◦ . Do đó, tứ giác M BXC nội tiếp hay đường tròn
(M BC) luôn đi qua trung điểm X của đoạn thẳng KN .
86 Tuyển tập đề thi vào lớp 10 chuyên Toán

Câu 5
Cho một bảng ô vuông kích thước 6 × 7 (6 hàng, 7 cột) được tạo bởi các ô vuông
có kích thước 1 × 1. Mỗi ô vuông có kích thước 1 × 1 được tô bởi một trong hai
màu đen hoặc trắng sao cho trong mọi bảng ô vuông kích thước 2 × 3 hoặc 3 × 2,
có ít nhất hai ô vuông kích thước 1 × 1 được tô màu đen có chung cạnh. Gọi m là
số ô vuông kích thước 1 × 1 được tô màu đen trong bảng

iế n
1. Chỉ ra một cách tô sao cho m = 20.

2. Tìm giá trị nhỏ nhất của m.

K
Lời giải. 1. Ta chỉ ra một cách tô như sau


-T
A1
á n
To
bộ

2. Xét ba ô như sau


c
lạ
u

thì khi đó theo giả thiết, trong ba ô này có ít nhất một ô được tô màu đen.
Xét bốn ô như sau

Trong ba ô 1, 2, 3 có ít nhất một ô được tô màu đen. Giả sử ô được tô là ô 1. Trong


ba ô 2, 3, 4 có ít nhất một ô được tô màu đen. Như vậy, trong bốn ô này có ít nhất
2 ô được tô màu đen.
Trong bảng 6 × 7, ta đánh số các ô như hình vẽ sau
Nguyễn Tiến Lâm - Trịnh Huy Vũ 87

2 4

iế n
7 1 7 8 2 8

1 7 1 3 2 8 2

K
1 3 5 3 2


4 5 3 5 6

-T
4 9 4 5 6 10 6

9 4 9 10
A1 6 10

• Trong nhóm các ô được đánh số từ 1 đến 6, mỗi nhóm ô có ít nhất hai ô được
tô màu đen.
• Trong nhóm các ô được đánh số từ 7 đến 10, mỗi nhóm ô có ít nhất một ô
n

được tô màu đen.


á
To

Do đó, số ô được tô màu tối thiểu là 2 · 6 + 1 · 4 = 16 ô.


bộ
c
lạ
u

88 Tuyển tập đề thi vào lớp 10 chuyên Toán

Sau đây là một cách để tô được 16 ô đen.

iế n
K

-T
A1
á n
To
bộ
c
lạ
u

Nguyễn Tiến Lâm - Trịnh Huy Vũ 89

2.12. Đề thi tuyển sinh vào lớp 10 chuyên Tin Hà


Nội, năm 2020

Câu 1

1. Giải phương trình (x + 2) x2 + 1 = x2 + 2x + 1.

iế n
2. Chứng minh rằng
1 1 1 1
√ √ + √ √ + ··· + √ √ =1− √ .

K
2 1+1 2 3 2+2 3 2021 2020 + 2020 2021 2021

Lời giải.


-T
1. Phương trình ban đầu tương đương với
√ √
x2 + 1 − x x2 + 1 + 2x − 2 x2 + 1 = 0.
√ √
⇐⇒ ( x2 + 1 − x)( x2 + 1 − 2) = 0.
√ √ √
Từ đó ta suy ra x2 + 1 = x hoặc √x2 + 1 = 2. Giải phương √ trình x2 + 1 = x
A1
suy ra vô nghiệm. Giải phương trình x2 + 1 = 2 suy ra x = ± 3.

Vậy S = {± 3}.
2. Xét số hạng tổng quát
n

1 (n + 1) − n
á

√ √ =p √ √
(n + 1) n + n n + 1 n(n + 1)( n + n + 1)
To

√ √
n+1− n 1 1
= √ √ =√ −√ .
n n+1 n n+1
Vì vậy,
bộ

1 1 1 1 1 1 1
V T = √ − √ + √ − √ + ··· + √ −√ =1− √ .
1 2 2 3 2020 2021 2021
c
lạ

Câu 2
1. Chứng minh rằng với mọi số nguyên dương n, số A = 59n − 17n − 9n + 2n chia
u

hết cho 35.


2. Tìm tất cả các số nguyên x, y thỏa mãn điều kiện x2 y − 3y − 4x − 1 = 0.

.
Lời giải. 1. Chú ý rằng (an − bn ) .. (a − b) với a, b, n ∈ Z+ nên ta có
.
A = (59n − 9n ) − (17n − 2n ) .. 5,
.
A = (59n − 17n ) − (9n − 2n ) .. 7.
Từ đó ta có điều phải chứng minh.
90 Tuyển tập đề thi vào lớp 10 chuyên Toán

2. Phương trình ban đâu tương đương với y(x2 − 3) = 4x + 1. Chú ý rằng x2 − 3 6= 0
với mọi x ∈ Z. Do đó, ta được
4x + 1
y= 2 .
x −3
16x2 − 1 47
Ta có y ∈ Z =⇒ = 16 + ∈ Z =⇒ x2 − 3 | 47. Do đó, chú ý rằng
x2 − 3 x2 − 3
x2 − 3 ≥ −3, ta được x2 − 3 ∈ {±1, 47}. Vì x ∈ Z, ta thu được x = ±2.

iế n
Đáp số. (x, y) = (2, 9); (−2, −7).

K
Câu 3
1. Tìm tất cả các số thực a, b, c thỏa mãn đồng thời các điều kiện a2 + b2 + c2 =


38, a + b = 8 và b + c ≥ 7.

-T
2. Với a, b, c là các số thực không âm và
√ luôn thỏa mãn a2 + b2 + c2 = 2ab + 2bc +
3
2ca, chứng minh rằng a + b + c ≥ 2abc.

Lời giải.
A1
1. 
2 2 2
a + b + c = 38
 (1)
a+b=8 (2)
n

b+c≥7 (3)

á

Từ (1) và (2) ta suy ra a = 8 − b và c ≥ 7 − b. Thay vào (1) ta có


To

38 ≥ (8 − b)2 + b2 + (7 − b)2 = 3b2 − 30b + 113 = 3(b − 5)2 + 38 ≥ 38.

Suy ra b = 5. Từ (2) ta suy ra a = 3 và từ (1) ta suy ra c = 2. Vậy (a, b, c) = (3, 5, 2).


bộ

2. Từ a2 + b2 + c2 = 2ab + 2bc + 2ca ta suy ra

c2 − 2c(a + b) + (a2 + b2 − 2ab) = 0.


c
lạ

√ √ √ 2
Ta có ∆0 = (a + b)2 − (a2 + b2 − 2ab) = 4ab. Suy ra c = (a + b) + 2 ab = a+ b
√ √ √ 2
hoặc c = (a + b) − 2 ab = a− b .
u

√ 2

√
• c= a+ b . Bất đẳng thức tương đương với

√ 2 √ 2
√ r √
3
a+b+ a + b ≥ 3 2ab a+ b .

√ √ √
Đặt s = a+ b, p = ab. Bất đẳng thức trở thành
p
2s2 − 2p ≥ 3 3 2s2 p2
Nguyễn Tiến Lâm - Trịnh Huy Vũ 91

Lập phương hai vế, chuyển vế và phân tích nhân tử, bất đẳng thức trở thành

2(s2 − 4p)(2s2 + p)2 ≥ 0.

Bất đẳng thức trên hiển nhiên đúng do s2 ≥ 4p (AM-GM) và (2s2 + p2 )2 ≥ 0.


√ √ 2
• c= a − b . Bất đẳng thức tương đương với

iế n
√ 2 √ 2
√ r √
3
a+b+ a − b ≥ 3 2ab a− b .

√ √ √

K
Đặt s = a+ b, p = ab. Bất đẳng thức trở thành
p
2s2 − 6p ≥ 3 3 2(s2 − 4p)p2 .


-T
Lập phương hai vế, chuyển vế và phân tích nhân tử, bất đẳng thức trở thành

2s2 (2s2 − 9p)2 ≥ 0.

Bất đẳng thức trên luôn đúng


A1
Vậy ta đã chứng minh được bất đẳng thức. Dấu bằng xảy ra tại ví dụ a = b = 1 và
c = 4.
n

Câu 4
á
To

Cho tam giác ABC có ba góc nhọn, AB < AC và ba đường cao AD, BE, CF cùng
đi qua điểm H. Gọi (S) là đường tròn ngoại tiếp của tam giác DEF .

1. Chứng minh rằng đường tròn (S) đi qua trung điểm của đoạn thẳng AH.
bộ

2. Gọi M và N lần lượt là giao điểm của đường tròn (S) với các đoạn thẳng BH
và CH. Tiếp tuyến tại D của đường tròn (S) cắt M N tại điểm T . Chứng
minh rằng đường thẳng HT song song với đường thẳng EF .
c

3. Gọi P là giao điểm của hai đường thẳng BH và DF , Q là giao điểm của hai
lạ

đường thẳng CH và DE. Chứng minh rằng ba điểm T, P, Q là ba điểm thẳng


hàng.
u

Lời giải.
1. Gọi X là trung điểm của AH. Xét tam giác AF H vuông tại F với F X là trung
tuyến nên ta được XF = XA = XH và ∠F XH = 2∠F AH. Mặt khác, từ các
tứ giác nội tiếp AEHF , CDHE và AF DC ta có ∠BED = ∠BCH = ∠F AH =
∠F EB, dẫn tới EB là phân giác của ∠DEF . Vì vậy nên ∠F ED = 2∠F EB =
2∠F AH = ∠F XD. Do đó, tứ giác DEXF nội tiếp. Hay nói cách khác, đường tròn
(S) ≡ (DEF ) luôn đi qua trung điểm X của đoạn thẳng AH.
92 Tuyển tập đề thi vào lớp 10 chuyên Toán

X
E
F

iế n
H
S
P Q
T

K
M N

B D C


-T
2. Chứng minh tương tự câu 1, ta suy ra M, N tương ứng là trung điểm của BH và
CH. Do M N là đường trung bình của tam giác HBC và HD ⊥ BC nên ta được
M N chính là trung trực của HD. Như vậy, các điểm T, M nằm trên trung trực
của đoạn thẳng HD, kết hợp với DT là tiếp tuyến của (S), ta suy ra ∠T HM =
∠T DM = ∠BED = ∠F EB. Vì vậy HT k EF .
A1
3. Từ ∠M DF = ∠M EF = ∠M ED, ta suy ra 4M P D ∼ 4M DE (gg), dẫn tới
MP
MD
=M D
ME
MP
. Chú ý rằng M D = M H nên ta được M H
=M H
ME
=⇒ M PH
P
=M H
HE
. Chứng
NQ NH
minh hoàn toàn tương tự ta cũng được QH = HF . Mặt khác, chú ý rằng tứ giác
n

HF
EF M N nội tiếp nên 4HEF ∼ 4HN M (gg) và kéo theo HE = HM . Do đó, chú ý
á

HN
rằng HM = DM và HN = DN ta được
To

 2  2
P M QH HM HF HM DM
· = · = = .
P H QN HE HN HN DN
Trong khi đó, do DT là tiếp tuyến tại D của (S) nên 4T M D ∼ 4T DN , kéo theo
bộ

TM
TD
= TT N
D
= DM
DN
. Do đó,
 2
TM TM TD DM
= · = .
TN TD TN DN
c
lạ

Như vậy, PP M · QH = TT M
H QN N
hay viết theo cách khác, PP M · QH · T N = 1. Áp dụng định
H QN T M
lý Menelaus cho tam giác HM N ta suy ra ba điểm T, P, Q thẳng hàng.
u

Câu 5
Trên bàn có 6 hộp kẹo, mỗi hộp có 5 viên kẹo. An và Bình cùng chơi một trò chơi
như sau: mỗi lượt chơi, An sẽ chọn một hộp tùy ý và lấy ít nhất 1 viên kẹo trong
hộp đó; còn Bình thì chọn một số hộp và trong các hộp đã chọn, mỗi hộp lấy đúng
1 viên kẹo. Hai bạn luân phiên thực hiện lượt chơi của mình. Bạn đầu tiên không
thể thực hiện được lượt chơi của mình là người thua cuộc. Nếu An là người lấy kẹo
trước, hãy chỉ ra chiến thuật chơi để Bình là người thắng cuộc.
Nguyễn Tiến Lâm - Trịnh Huy Vũ 93

Lời giải. Ta cho Bình bốc như sau: Sau mỗi lần An bốc, Bình sẽ bốc 1 viên từ những
hộp còn có nhiều hơn 1 viên kẹo. Sau 4 lượt, An chỉ có thể làm rỗng tối đa 4 hộp kẹo, tức
là vẫn còn ít nhất 2 hộp kẹo còn kẹo. Điều này cũng đảm bảo cho Bình có thể bốc theo
cách nói trên trong vòng 4 lượt này. Hơn nữa trong mỗi hộp kẹo còn có kẹo thì chỉ còn
đúng 1 viên do cách bốc của Bình. Khi đó Bình sẽ thắng ở lượt thứ 5 (sau khi Bình bốc
1 viên ở một hộp bất kỳ thì Bình sẽ bốc nốt chỗ kẹo còn lại)

iế n
K

-T
A1
á n
To
bộ
c
lạ
u

94 Tuyển tập đề thi vào lớp 10 chuyên Toán

2.13. Đề thi tuyển sinh vào lớp 10 trường Phổ


thông Năng Khiếu ĐHQG TPHCM năm 2020,
vòng 2

Câu 1

iế n
Cho các phương trình x2 + ax + 3 = 0 và x2 + bx + 5 = 0 với a, b là các tham số

(a) Chứng minh rằng nếu ab ≥ 16 thì trong hai phương trình có ít nhất một phương

K
trình có nghiệm.

(b) Giả sử hai phương trình trên có nghiệm chung x0 . Tìm a, b sao cho |a| + |b| có
giá trị nhỏ nhất.


-T
Lời giải.
(a) Với phương trình x2 + ax + 3 = 0 ta có ∆1 = a2 − 12. Với phương trình x2 + bx + 5 = 0
ta có ∆2 = b2 − 20. Vì ab ≥ 16 nên ta có ∆1 + ∆2 = a2 + b2 − 32 ≥ 2ab − 32 ≥ 0.
A1
Từ đó suy ra ∆1 ≥ 0 hoặc ∆2 ≥ 0, dẫn đến trong hai phương trình có ít nhất một
phương trình có nghiệm.
x20 + 3
(b) Chú ý rằng x0 6= 0. Ta có x20 + ax0 + 3 = 0 và x20 + bx0 + 5 = 0, dẫn đến a = −
n
x0
x20 + 5
á

và b = − . Khi đó ta được
x0
To

2
x0 + 3 x20 + 5 2x20 + 8 2x20 + 8

8|x0 |
|a| + |b| = + ≥ = ≥ = 8.
x0 x0 x0 |x0 | |x0 |
bộ

Dấu bằng xảy ra chẳng hạn khi x0 = 2 hoặc x0 = −2. Thay x0 = 2 vào hai phương
7 9
trình thì ta thu được a = − và b = − . Thay x0 = −2 vào hai phương trình thì ta
2 2
7 9
thu được a = và b = .
c

2 2
lạ

Kết luận. Giá trị nhỏ nhất của |a| + |b| là 8.


u

Câu 2

Cho phương trình 3x2 − y 2 = 23n với n là số tự nhiên.

(a) Chứng minh rằng nếu n chẵn thì phương trình đã cho không có nghiệm nguyên
(x, y).

(b) Chứng minh rằng nếu n lẻ thì phương trình đã cho có nghiệm nguyên (x, y).

Lời giải.
Nguyễn Tiến Lâm - Trịnh Huy Vũ 95

(a) Chú ý rằng y 2 chia 3 dư 0 hoặc 1, nên 3x2 − y 2 chia 3 dư 0 hoặc 2. Tuy nhiên nếu n
chẵn thì 23n ≡ 2n ≡ 1 (mod 3), khi đó phương trình vô nghiệm.

(b) Ta dùng quy nạp ở câu này. Với n = 1 phương trình 3x2 − y 2 = 23 có một nghiệm
(x, y) = (3, 2) là thỏa mãn. Giả sử bài toán đúng đến n = k lẻ, tức là tồn tại cặp số
nguyên (xk , yk ) thỏa mãn 3x2k − yk2 = 23k . Khi đó ta có

23k+2 = (3x2k − yk2 ) · 232

iế n
= (3x2k − yk2 )(262 − 3.72 )
= 3 · 262 · x2k − 32 · 72 x2k − 262 yk2 + 3 · 72 · yk2

K
= 3(26xk − 7yk )2 − (21xk − 26yk )2

Khi đó phương trình 3x2 −y 2 = 23k+2 có một cặp nghiệm là (x, y) = (26xk −7yk , 21xk −


26yk ), tức là bài toán cũng đúng với n = k + 2. Quy nạp hoàn tất.

-T
Câu 3 A1
Cho đường tròn (O), dây cung BC không chứa tâm O và điểm A thay đổi trên cung
lớn BC. Lấy các điểm E, F thỏa mãn ∠ABE = ∠CAE = ∠ACF = ∠BAF = 90◦ .

(a) Chứng minh rằng AE · AC = AF · AB và O là trung điểm của EF .


n

(b) Hạ AD vuông góc với EF (D ∈ EF ). Chứng minh rằng các tam giác DAB và
DCA đồng dạng và D thuộc một đường tròn cố định.
á
To

(c) Gọi G là giao điểm của AD và đường tròn (O) (G 6= A). Chứng minh rằng AD
đi qua một điểm cố định và GB · AC = GC · AB.

(d) Gọi K là tâm đường tròn ngoại tiếp của tam giác AEF . Chứng minh rằng AK
bộ

đi qua một điểm cố định.


c

Lời giải.
lạ

(a) Ta có ∠EAB = ∠F AC(= 90◦ −∠BAC). Từ đó suy ra 4EAB ∼ 4F AC và AB AE AF


= AC ,
vậy nên AE · AC = AF · AB. Dựng đường kính AX của đường tròn (O). Vì XE k AF
u

và XF k AE nên AEXF là hình bình hành. Do đó, EF đi qua trung điểm O của

AX.

(b) Từ các tứ giác nội tiếp ADBE và ADCF ta suy ra ∠DAB = ∠BED = ∠AF D =
∠DCA. Chứng minh tương tự ta cũng có ∠DBA = ∠DAC. Như vậy 4DAB ∼
4DCA. Hơn nữa ta cũng có

∠BDC = ∠BAC+∠DBA+∠DCA = ∠BAC+∠DAC+∠DAB = 2∠BAC = ∠BOC.

Do đó, tứ giác BDOC nội tiếp, dẫn tới D luôn thuộc đường tròn (BOC) cố định.
96 Tuyển tập đề thi vào lớp 10 chuyên Toán

Q F

iế n
D
J
E I

K
H M
B C


X
G

-T
K

A1
T

(c) Dựng đường kính OT của đường tròn (BOC). Khi đó ta có T là điểm cố định. Mặt
n

khác, do ∠ODT = ∠ADO = 90◦ nên ba điểm A, D, T thẳng hàng. Như vậy đường
á

thẳng AD luôn đi qua điểm T cố định.


To

Chú ý rằng T chính là giao điểm hai tiếp tuyến tại B, C của đường tròn (O). Từ đó
ta thu được hai cặp tam giác đồng dạng 4T GB ∼ 4T BA và 4T GC ∼ 4T CA. Như
vậy, GB
AB
= TT B
G
= TT G
C
= GC
AC
, vì vậy GB · AC = GC · AB.
bộ

(d) Bổ đề. Cho tam giác ABC với đường cao AH (H ∈ BC) và tâm ngoại tiếp O. Khi
đó, ∠HAB = ∠OAC.
Cách 1. Xét tam giác AEF có AD là đường cao và K là tâm ngoại tiếp. Khi đó
ta có ∠DAE = ∠KAF (theo bổ đề), lại chú ý rằng ∠EAB = ∠F AC, ta thu được
c

∠DAB = ∠KAC (1).


lạ

Mặt khác, gọi M là trung điểm của BC. Khi đó ta có OA2 = OB 2 = OC 2 = OM ·OT ,
dẫn đến 4OAM ∼ 4OT A. Gọi H là hình chiếu của A trên BC. Từ đó ta thu được
u

∠OAM = ∠OT A = ∠HAT . Kết hợp với ∠HAB = ∠OAC (theo bổ đề) suy ra
∠T AB = ∠M AC (2).

Từ (1) và (2) suy ra ∠KAC = ∠M AC, dẫn tới A, K, M thẳng hàng. Vậy đường
thẳng AK luôn đi qua điểm M cố định.
Cách 2. Gọi Q, R tương ứng là trung điểm của AE, AF . KQ, KR tương ứng cắt
AB, AC tại các điểm I, J. Khi đó ta có AIKJ là hình bình hành và AK đi qua trung
điểm của IJ. Trong khi đó, từ 4AQI ∼ 4ARJ ta được AJ AI
= AQ
AR
= AE
AF
= AB
AC
. Do
đó, IJ k BC. Kết hợp với việc AK đi qua trung điểm của IJ, ta suy ra AK luôn đi
qua trung điểm M cố định của BC.
Nguyễn Tiến Lâm - Trịnh Huy Vũ 97

Câu 4
Cho số tự nhiên a = 313 .57 .720 .

iế n
(a) Gọi A là tập hợp các số nguyên dương k sao cho k là ước của a và k chia hết
cho 105. Hỏi tập A có bao nhiêu phần tử?

K
(b) Giả sử B là một tập con bất kỳ của A có 9 phần tử. Chứng minh ta luôn có
thể tìm được 2 phần tử của B sao cho tích của chúng là số chính phương.


-T
Lời giải. (a) Ta có 105 = 3.5.7. Do đó ta có thể viết lại tập A dưới dạng sau:
A1
A = {3x .5y .7z , 1 ≤ x ≤ 13, 1 ≤ y ≤ 7, 1 ≤ z ≤ 20}.

Do đó số phần tử của tập A là số các bộ (x, y, z) thỏa mãn điều kiện trên, mà theo
n
quy tắc nhân thì chính là: 13.7.20 = 1820.
á

Kết luận: |A| = 1820.


To

(b) 9 phần tử của B có dạng 3xi .5yi .7zi , với 1 ≤ i ≤ 9. Trước hết, trong 9 số x1 , . . . , x9 ,
theo Dirichlet tổn tại ít nhất 5 số có cùng tính chẵn lẻ, không mất tính tổng quát giả
bộ

sử đó là x1 , . . . , x5 .

Tiếp theo, trong 5 số y1 , . . . , y5 tồn tại ít nhất 3 số có cùng tính chẵn lẻ, không mất
tính tổng quát giả sử đó là y1 , y2 , y3 .
c
lạ

Cuối cùng, trong 3 số z1 , z2 , z3 tồn tại ít nhất 2 số có cùng tính chẵn lẻ, giả sử đó là
z1 , z2
u

Khi đó x1 + x2 , y1 + y2 , z1 + z2 là các số chẵn, nên tích


(3x1 .5y1 .7z1 ) . (3x2 .5y2 .7z2 )

là số chính phương.
98 Tuyển tập đề thi vào lớp 10 chuyên Toán

Câu 5
Cho hệ phương trình với k là tham số:
 r r
x x x
√ + + =k






 yz y z



iế n
r r
y y y

√ + + =k

 zx z x




 r r
z z z

K


√ + + =k


xy x y


(a) Giải hệ với k = 1.

-T
(b) Chứng minh hệ vô nghiệm với k ≥ 2 và k 6= 3.

Lời giải. (a) Điều kiện của nghiệm là x, y, z là các số cùng dương hoặc cùng âm.
A1
• Trường hợp 1: x, y, z cùng dương. Hệ phương trình tương đương với
r  r 
x x
+1 +1 =2


n




 y z

á




 r  r 
y y
To

+1 +1 =2

 x z




 r  r 

 z z
+1 +1 =2


bộ

y x

Nhân cả hai vế của ba phương trình lại với nhau ta được


c

Y r x  r 
y
+1 + 1 = 8.
lạ

cyc
y x
u

Tuy nhiên
r  r  r r
x y x y

+1 +1 =2+ + >2
y x y x
và tương tự với các tích còn lại. Khi đó
Y r x  r 
y
+1 + 1 > 2.2.2 = 8.
cyc
y x

Suy ra hệ vô nghiệm trong trường hợp này.


Nguyễn Tiến Lâm - Trịnh Huy Vũ 99

• Trường hợp 2: x, y, z cùng âm. Hệ phương trình tương đương với


r  r 
x x
−1 −1 =0






 y z




 r  r 
y y
−1 −1 =0
 x z

iế n





 r  r 

 z z
−1 −1 =0


y x

K
Khi đó x = y = z.

Vậy các nghiêm của hệ phương trình là x = y = z = a với a < 0.


-T
(b) Điều kiện của nghiệm là x, y, z là các số cùng dương hoặc cùng âm.

• Trường hợp 1: x, y, z cùng dương. Hệ phương trình tương đương với


r  r 
x x
+1 +1 =k+1





 y
A1 z





 r  r 
y y
+1 +1 =k+1
x z
n






á

 r  r 

 z z
+1 +1 =k+1


To

y x

x y x y
Không mất tính tổng quát giả sử x = min(x, y, z). Ta có ≤ và ≤ . Dấu
y x z z
bằng xảy ra khi và chỉ khi x = y. Khi đó
bộ

r  r  r  r 
x x y y
k+1= +1 +1 ≤ +1 + 1 = k + 1.
y z x z
c

Dấu bằng ở giữa buộc phải xảy ra, dẫn đến x = y. Tương tự ta cũng có x = z,
lạ

dẫn đến x = y = z. Khi đó thay vào phương trình thứ nhất của hệ ta thu được
k = 3, loại do k 6= 3. Vậy hệ vô nghiệm trong trường hợp này.
u

• Trường hợp 2: x, y, z cùng âm. Hệ phương trình tương đương với


 r  r 
x x
−1 −1 =1−k






 y z




 r  r 
y y
−1 −1 =1−k

 x z




 r  r 

 z z
−1 −1 =1−k


y x

100 Tuyển tập đề thi vào lớp 10 chuyên Toán

Không mất tính tổng quát, giả sử x = min(x, y, z). Khi đó −x ≥ −y và −x ≥ −z,
khi đó r  r 
x x
−1 − 1 ≥ 0.
y z
Tuy nhiên 1 − k < 0, cho nên hệ phương trình vô nghiệm trong trường hợp này.

Vậy hệ phương trình vô nghiệm với k ≥ 2, k 6= 3.

iế n
K

-T
A1
á n
To
bộ
c
lạ
u

Nguyễn Tiến Lâm - Trịnh Huy Vũ 101

2.14. Đề thi tuyển sinh vào lớp 10 chuyên Toán


Thành phố Hồ Chí Minh, năm 2020

Câu 1
a b c
Cho ba số dương a, b, c thỏa mãn điều kiện + + = 2020.

iế n
 2 b + c c +a a + b
a b2 c2
Tính giá trị của biểu thức P = + + : (a + b + c).
b+c c+a a+b

K
Lời giải. Ta có


-T
 
a b c
+ + (a + b + c) = 2020(a + b + c)
b+c c+a a+b
a2 b2 c2
=⇒ +a+ +b+ + c = 2020(a + b + c)
b+c c+a a+b
a2 b2 c2
=⇒ + +
A1
= 2019(a + b + c).
b+c c+a a+b

Từ đó ta suy ra P = 2019.
á n

Câu 2
√ √
To

a) Giải phương trình: 2x2 + x + 9 + 2x2 − x + 1 = x + 4.


(
y 2 − 2xy = 8x2 − 6x + 1
b) Giải hệ phương trình
y 2 = x3 + 8x2 − x + 1.
bộ

Lời giải.
c

√ √
lạ

a) Đặt a = 2x2 + x + 9 và b = 2x2 − x + 1. Khi đó a2 − b2 = 2(x + 4). Phương trình


trở thành
1 1
a + b = (a2 − b2 ) = (a − b)(a + b)
u

2 2

hay
(a + b)(a − b − 2) = 0.

√ a + b > 0, dẫn đến a − b = 2. Kết hợp với a + b = x + 4 dẫn


Lại có a > 0, b > 0 nên
đến 2a = x + 6, hay 2 2x 2
 + x+ 9 = x + 6. Bình phương hai vế và giải phương trình
8
bậc hai, ta thu được x ∈ 0, . Thử lại vào phương trình ban đầu thì cả hai nghiệm
7  
8
đều thỏa mãn. Vậy phương trình có hai nghiệm 0, .
7
102 Tuyển tập đề thi vào lớp 10 chuyên Toán

b)
(
y 2 − 2xy = 8x2 − 6x + 1, (1)
y 2 = x3 + 8x2 − x + 1. (2)

Phương trình (1) tương đương với (3x − 1)2 = (x − y)2 .

• Trường hợp 1: x − y = 3x − 1. Suy ra y = 1 − 2x. Thay vào phương trình (2)

iế n
ta có
(1 − 2x)2 = x3 + 8x2 − x + 1.

K
Giải phương trình trên ta thu được x ∈ {−3, −1, 0}. Kết hợp với y = 1 − 2x ta
thu được ba bộ nghiệm (x, y) = (−3, 7), (−1, 3), (0, 1).
• Trường hợp 2: x − y = 1 − 3x. Suy ra y = 4x − 1. Thay vào phương trình (2)


ta có

-T
(4x − 1)2 = x3 + 8x2 − x + 1.
Giải phương trình trên ta thu được x ∈ {0, 1, 7}. Kết hợp với y = 4x − 1 ta thu
được ba bộ nghiệm (x, y) = (0, −1), (1, 3), (7, 27).
A1
Vậy hệ phương trình có 6 nghiệm (x, y) = (−3, 7), (−1, 3), (0, 1), (0, −1), (1, 3), (7, 27).
n
Câu 3
á

Cho tam giác nhọn ABC (AB < BC < CA) nội tiếp đường tròn (O). Từ A kẻ
To

đường thẳng song song với BC cắt (O) tại A1 . Từ B kẻ đường thẳng song song với
AC cắt (O) tại B1 . Từ C kẻ đường thẳng song song với AB cắt (O) tại C1 . Chứng
minh rằng các đường thẳng qua A1 , B1 , C1 lần lượt vuông góc với BC, CA, AB đồng
quy.
bộ

A
c

A1
lạ
u

K

O
H

B C

X
Nguyễn Tiến Lâm - Trịnh Huy Vũ 103

Lời giải. Gọi H là trực tâm của tam giác ABC và K là điểm đối xứng với H qua O.
Dựng đường kính AX của đường tròn (O). Khi đó, ta có ∠AA1 X = 90◦ , hay A1 X ⊥ AA1 ,
kết hợp AA1 k BC dẫn đến A1 X ⊥ BC (1). Mặt khác, ta cũng có AHXK là hình bình
hành nên XK k AH, kết hợp với AH ⊥ BC dẫn đến XK ⊥ BC (2). Từ (1) và (2) suy
ra A1 , X, K thẳng hàng và A1 K ⊥ BC. Chứng minh tương tự B1 K ⊥ CA, C1 K ⊥ AB.
Như vậy, các đường thẳng qua A1 , B1 , C1 lần lượt vuông góc với BC, CA, AB đồng quy
tại K.

iế n
Câu 4

K
a2 + b 2 (a − b)2
a) Cho 2 số thực a, b. Chứng minh rằng: ≥ ab + 2 .
2 a + b2 + 2


b) Cho 2 số dương a, b thỏa mãn điều kiện a + b ≤ 3.

-T
20 7
Tìm giá trị nhỏ nhất của biểu thức: Q = b − a + + .
a b

Lời giải. a) Ta có
A1
a2 + b 2 (a − b)2 (a − b)2 (a − b)2 (a − b)2 (a2 + b2 )
− ab − 2 = − 2 = ≥0
2 a + b2 + 2 2 a + b2 + 2 2(a2 + b2 + 2)
n

a2 + b 2 (a − b)2
á

cho nên ≥ ab + 2 .
2 a + b2 + 2
To

b) Ta phân tích Q như sau:


   
20 7
Q = 5a + + 7b + − 6(a + b).
bộ

a b

Áp dụng bất đẳng thức AM-GM ta có


c

r
20 20
lạ

5a + ≥ 2 5a · = 20,
a a
u

r
7 7
7b + ≥ 2 7b · = 14.

b b
Dấu bằng xảy ra khi và chỉ khi a = 2, b = 1. Kết hợp với a + b ≤ 3 ta suy ra

Q ≥ 20 + 14 − 6 · 3 = 16.

Dấu bằng xảy ra khi và chỉ khi a = 2, b = 1.


104 Tuyển tập đề thi vào lớp 10 chuyên Toán

Câu 5
Đường tròn (I) nội tiếp tam giác ABC tiếp xúc với các cạnh AB, BC, CA lần lượt
tại D, E, F . Kẻ đường kính EJ của đường tròn (I). Gọi d là đường thẳng qua A
song song với BC. Đường thẳng JD cắt d, BC lần lượt tại L, H.

a) Chứng minh rằng E, F, L thẳng hàng.

iế n
b) JA, JF cắt BC lần lượt tại M, K. Chứng minh rằng M H = M K.

Lời giải. a) Do AL k BC nên ∠ABC = 180◦ − ∠DAL. Từ EJ là đường kính của (I)

K
dẫn đến ∠EDJ = 90◦ . Kết hợp với BI ⊥ DE ta suy ra DJ k BI. Như vậy ∠ADL =
∠ABI = 12 ∠ABC. Từ đó suy ra ∠ALD = 180◦ − ∠DAL − ∠ADL = ∠ABC −


1
2
∠ABC = 12 ∠ABC = ∠ADL và tam giác ADL cân tại A. Vì thế AL = AD = AF

-T
và do đó ∠AF L = 90◦ − 21 ∠LAC = 90◦ − 12 ∠F CE = ∠CF E = 180◦ − ∠AF E, dẫn
tới ∠AF L + ∠AF E = 180◦ . Như vậy, E, F, L thẳng hàng.

b) Gọi N là giao điểm của JF và đường thẳng d. Chứng minh tương tự ở câu a) ta được
AN = AF = AD = AL. Như vậy, A là trung điểm N L. Trong khi đó, sử dụng định
lý Thales ta được MALH = JM =M K
A1
. Từ đó suy ra M H = M K.
JA AN

G
n
N A L d
á
To

F
D
I
bộ

H B E M C K
c
lạ

Câu 6
u

Tìm tất cả các số nguyên dương x, y thỏa mãn phương trình 3x − y 3 = 1.


Lời giải. Chuyển vế ta có

3x = y 3 + 1 = (y + 1)(y 2 − y + 1).

Suy ra (
y + 1 = 3a ,
y 2 − y + 1 = 3b
Nguyễn Tiến Lâm - Trịnh Huy Vũ 105

với a, b ∈ N.
Đặt d = (y + 1, y 2 − y + 1). Ta có d | y 2 − y + 1 = (y + 1)(y − 2) + 3, và d | y + 1 dẫn đến
d | 3. Suy ra trong hai số tự nhiên a, b tồn tại ít nhất một số không vượt quá 1 (nếu cả
a và b đều lớn hơn 1, y + 1 và y 2 − y + 1 cùng chia hết cho 9, dẫn đến d chia hết cho 9,
trái với d | 3).

• a = 0. Khi đó y + 1 = 1, dẫn đến y = 0, loại do y nguyên dương.

iế n
• a = 1. Khi đó y + 1 = 3, dẫn đến y = 2. Suy ra 3x = 23 + 1 = 9, dẫn đến x = 2. Ta
có nghiệm (x, y) = (2, 2).

K
• b = 0. Khi đó y 2 − y = 0, dẫn đến y = 1. Suy ra 3x = 13 + 1 = 2, loại.

• b = 1. Khi đó y 2 − y = 3, dẫn đến y = 2. Ta lại có bộ nghiệm (x, y) = (2, 2).


Vậy phương trình có nghiệm (x, y) = (2, 2).

-T
A1
á n
To
bộ
c
lạ
u

106 Tuyển tập đề thi vào lớp 10 chuyên Toán

2.15. Đề thi tuyển sinh vào lớp 10 trường THPT


chuyên Khoa học Tự nhiên năm 2021, vòng 1

Câu 1
Giải phương trình

iế n
√ √ p
13 5 − x + 18 x + 8 = 61 + x + 3 (5 − x)(x + 8)

√ √

K
Lời giải. Điều kiện xác định: −8 ≤ x ≤ 5. Đặt a = 5 − x, b = x + 8 = b thì a, b ≥ 0
và a2 + 2b2 = 5 − x + 2x + 16 = 21 + x
Phương trình trở thành


13a + 18b = 40 + a2 + 2b2 + 3ab.

-T
hay tương đương
(a + b − 5)(a + 2b − 8) = 0.
Từ đây, ta có hai trường hợp
• Trường hợp 1: a + b = 5 và a2 + b2 = 13
A1
Rút a = 5 − b và thay vào phương trình thứ hai ta được (5 − b)2 + b2 = 13 hay
b2 − 5b + 6 = 0.
Giải ta tìm được b = 2 và b = 3. Tương ứng thì x = −4 và x = 1.
n

• Trường hợp 2: a + 2b = 8 và a2 + b2 = 13.


á

Tương tự, ta cũng có(8 − 2b)2 + b2 = 13 hay tương đương


To

5b2 − 32b + 51 = 0.
17 89
Giải ta tìm được b = 3 và b = . Tương ứng x = 1 và x = .
5 25
89
bộ

Thử lại các nghiệm đều thỏa mãn. Vậy phương trình có các nghiệm là −4, 1, .
25

Câu 2
c
lạ

Giải hệ phương trình 


x4 + y 4 + 6x2 y 2 = 1
x(x + y)4 = x − y
u

Lời giải. Ta sẽ trình bày hai cách giải.


Cách 1. Nhân chéo 2 vế của hệ phương trình ta thu được
x(x + y)4 = (x − y)(x4 + y 4 + 6x2 y 2 ).
Khai triển và rút gọn ta được

y(5x2 + 10x2 y 2 + y 4 ) = 0.
Từ đây ta có hai trường hợp
Nguyễn Tiến Lâm - Trịnh Huy Vũ 107

• Với y = 0 thì từ phương trình đầu tiên suy ra x4 = 1 hay x = 1 hoặc x = −1. Các
nghiệm này đều thoả mãn hệ.

• Với 5x4 + 10x2 y 2 + y 4 = 0 ⇒ x = y = 0 (do x2 > 0, y 2 > 0).


Ta loại nghiệm này do khi đó x4 + y 4 + 6x2 y 2 = 0.

Vậy hệ phương trình có các nghiệm (x, y) là (−1, 0); (1, 0).

iế n
Cách 2. Nếu x = 0 thì từ phương trình thứ hai suy ra y = 0 nhưng khi đó không thoả
mãn phương trình đầu tiên nên x 6= 0. Từ phương trình thứ hai suy ra
x−y

K
x4 + y 4 + 6x2 y 2 + 4xy(x2 + y 2 ) = .
x

Trừ phương trình trên cho phương trình đầu tiên, ta được


-T
y
4xy(x2 + y 2 ) = −
x

hay tương đương y(4x2 (x2 + y 2 ) + 1) = 0. Từ đây ta tìm được y = 0 và x = ±1. Vậy hệ
đã cho có hai nghiệm (x, y) là (−1, 0); (1, 0).
A1
Câu 3
Tìm số nguyên dương n nhỏ nhất biết rằng khi chia n cho 7, 9, 11, 13 ta nhận được
n

các số dư là 3, 4, 5, 6.
á
To

Lời giải. Vì n chia cho 7,9,11,13 được số dư tương ứng là 3,4,5,6. Nên ta suy ra 2n chia
cho 7,9,11,13 được số dư tương ứng là : 6,8,10,12. Suy ra 2n + 1 chia hết cho cả 7,9,11 và
13. Chú ý là 7,9,11 và 13 đôi một nguyên tố cùng nhau nên số nguyên dương nhỏ nhất
bộ

chia hết cho cả 7,9,11,13 là : 7.9.11.13= 9009 nên 2n + 1 = 9009 kéo theo

9009 − 1
n= = 4504.
2
c
lạ

Câu 4
u

Cho tam giác nhọn ABC có điểm P nằm trong tam giác (P không nằm trên

các cạnh). Gọi J, K, L lần lượt là tâm đường tròn nội tiếp của các tam giác
P BC, P CA, P AB.

1. Chứng minh rằng BJC


[ + CKA [ = 90◦ .
\ + ALB

2. Giả sử P B = P C và P C < P A. Gọi X, Y, Z lần lượt là hình chiếu vuông


góc của J, K, L trên các cạnh BC, CA, AB. Dựng hình bình hành XY W Z.
Chứng minh rằng W nằm trên phân giác BAC.
[
108 Tuyển tập đề thi vào lớp 10 chuyên Toán

A A

E
W
F

iế n
L K Y
Z
P L K
J P

K
J
B C B C
X


-T
Lời giải.
[ = 90◦ + 1 BP
1. Ta có BJC \ \ = 90◦ + 1 CP
C, CKA [ [ = 90◦ + 1 AP
A và ALB [ B. Từ đó ta
2 2 2

được BJC
[ + CKA+
\ ALB 1
[ = 270 + (BP
2
\ C + CP
[ A+ AP
A1 [ B) = 270 + 2 ·360◦ = 450◦ .
◦ 1

2. Lấy các điểm E, F lần lượt đối xứng với C, B qua Y, Z. Chú ý rằng ta có BZ =
BA+BP −P A −P A
2
và CY = CA+CP 2
, vì thế BF = BA + BP − P A và CE = CA +
CP − P A. Như vậy, AF = BA − BF = P A − P B và AE = CA − CE = P A − P C,
để ý rằng P B = P C nên ta thu được AF = AE. Gọi W1 là trung điểm của EF .
n

Do 4P BC cân tại P nên X là trung điểm của BC. Từ đó ta được XY, Y W1 , W1 Z


á

và ZX lần lượt là đường trung bình của 4CBE, 4ECF, 4F BE và 4BF C. Do


đó, XY k ZW1 (cùng k BE) và XZ k Y W1 (cùng k CF ), vì thế XY W1 Z là hình
To

bình hành và do vậy W1 ≡ W . Mặt khác, 4AEF cân tại A và có W ≡ W1 là trung


điểm của EF , như vậy AW chính là đường phân giác BAC.
[ Ta có điều phải chứng
minh.
bộ

Câu 5
c

Cho tập A = {1, 2, ..., 2021}. Tìm số nguyên dương k lớn nhất (k ≥ 2) sao cho ta có
lạ

thể chọn được k số phân biệt từ tập A mà tổng của hai số phân biệt bất kỳ trong
k số được chọn không chia hết cho hiệu của chúng.
u

Lời giải. Ta chứng minh giá trị lớn nhất của k là 674. Trong k số được chọn thì không

thể chứa cả hai cặp {x, x + 1} và {x, x + 2}. Do đó nếu x1 < x2 < · · · < xk là k số
được chọn thì xi+1 − xi ≥ 3 với mọi i = 1, 2, ..., k − 1. Vì x1 ≥ 1 nên từ đây ta có
x2 ≥ 4, x3 ≥ 7, ...., xk ≥ 3k − 2. Vì xk ≤ 2021 nên k ≤ 674. Ví dụ, ta có thể chọn 674 số
sau đây 1, 4, 7, ..., 2020 thì tổng hai số bất kỳ sẽ chia 3 dư 2, còn hiệu hai số thì chia hết
cho 3 vì tất cả các số được chọn đều chia 3 dư 1.
Nguyễn Tiến Lâm - Trịnh Huy Vũ 109

2.16. Đề thi tuyển sinh vào lớp 10 trường THPT


chuyên Khoa học Tự nhiên năm 2021, vòng 2

Câu 1
1. Với a, b, c là các số thực thoả mãn a + b + c 6= 0 và (a + b)(b + c)(c + a) = 1,

iế n
chứng minh rằng

a b 1 + abc + ab(a + b + c)
+ 2 = .
a2 (a + b + c) + 1 + abc b (a + b + c) + 1 + abc (a + b + c)2

K
2. Giải hệ phương trình


(
x2 + 4y 2 + 4xy + 2x2 y 2 = 11

-T
.
3xy(x + 2y) + 31 = 9x + 18y + 13xy.

Lời giải.
1. Ta sử dụng đẳng thức quan trọng sau
A1
(a + b)(b + c)(c + a) + abc = (a + b + c)(ab + bc + ca)
= ab(a + b) + bc(b + c) + ca(c + a) + 3abc
n

Vì (a + b)(b + c)(c + a) = 1 nên ta suy ra 1 + abc = (a + b + c)(ab + bc + ca). Do đó,


á
To

a2 (a + b + c) + 1 + abc = (a + b + c)(a2 + ab + bc + ca) = (a + b + c)(a + b)(a + c).


Tương tự
b2 (a + b + c) + 1 + abc = (a + b + c)(b + a)(b + c).
bộ

Suy ra vế trái của đẳng thức cần chứng minh bằng


a b a(b + c) + b(c + a)
+ = .
(a + b + c)(a + b)(c + a) (a + b + c).(a + b)(b + c) (a + b + c)(a + b)(b + c)(c + a)
c

ab + ab + bc + ca ab(a + b + c) + (ab + bc + ca)(a + b + c) 1 + abc + ab(a + b + c)


lạ

= = 2
= .
a+b+c (a + b + c) (a + b + c)2
Ta có điều phải chứng minh.
u

2. Đặt a = x + 2y, b = xy thì hệ đã cho có thể viết lại thành


(
a2 + 2b2 = 11
.
3ab + 31 = 9a + 13b.

Cộng hai phương trình và chuyển vế, phân tích nhân tử ta có


(a + b − 4)(a + 2b − 5) = 0.
Từ đây ta có hai trường hợp
110 Tuyển tập đề thi vào lớp 10 chuyên Toán

• Nếu a + b = 4 thì a = 4 − b và thay vào phương trình a2 + 2b2 = 11 ta được

3b2 − 8b + 5 = 0.
5 7
Giải tìm được b = 1, a = 3 và b = , a = .
3 3  
1
+) Nếu a = 3, b = 1 thì x + 2y = 3, x · 2y = 2 tìm được (x, y) là (1, 1), 2, .
2

iế n
7 5 7 10
+) Nếu a = , b = thì x + 2y = , x · 2y = không tồn tại x, y.
3 3 3 3
• Nếu a + 2b = 5 thì a = 5 − 2b và thay vào phương trình a2 + 2b2 = 11 ta được

K
3b2 − 10b + 7 = 0.


7 1
Giải tìm được b = 1, a = 3 và b = , a = .

-T
3 3
+) Nếu  a = 3, b = 1 thì x + 2y = 3, x · 2y = 2 tương tự như trên thì (x, y) là
1
(1, 1), 2, .
2
7 1 1 14
+) b = , a = thì x + 2y = , x · 2y =
A1 thì không tồn tại x, y.
3 3 3 3
 
1
Vậy hệ đã cho có 2 nghiệm (1, 1), 2, .
2
á n

Câu 2
To

1. Tìm x, y nguyên dương thoả mãn 3x + 29 = 2y .

2. Với a, b, c là các số thực dương thoả mãn điều kiện 2(a+b+c)+ab+bc+ca = 9,


bộ

tìm giá trị lớn nhất của biểu thức


a+1 b+1 c+1
M= + 2 + 2 .
a2 + 10a + 21 b + 10b + 21 c + 10c + 21
c
lạ

Lời giải.
1. Với x = 1 thì y = 5 và ta có nghiệm là (1; 5). Với x > 1 thì y > 5 xét theo mod9 vì
u

x > 1, ta suy ra 2 ≡ 2y (mod 9) nên 1 ≡ 2y−1 (mod 9). Mặt khác 26 ≡ 1 (mod 9)
nên y = 6n + 1, n ≥ 1.

Xét theo mod 32 vì y > 5 ta suy ra 3x − 3 ≡ 0 (mod 32) kéo theo 3x ≡ 3 (mod 32)
dẫn tới 3x−1 ≡ 1 (mod 32). Lại có 38 ≡ 1 (mod 32), nên x = 8m + 1, m ≥ 1.
Cuối cùng xét theo mod 7 cho ta 38m+1 + 1 ≡ 26n+1 (mod 7). Lại có 26 ≡ 1 (mod 7)
nên 26n+1 ≡ 2 (mod 7). Suy ra 38m+1 ≡ 1 (mod 7), điều này là mâu thuẫn với sự
kiện 36 ≡ 1 (mod 7), và 8m + 1 không thể là bội của 6. Suy ra phương trình không
có nghiệm x > 1, y > 5.
Vậy phương trình đã cho có nghiệm duy nhất là x = 1, y = 5.
Nguyễn Tiến Lâm - Trịnh Huy Vũ 111

X x
2. Đặt x = a + 1, y = b + 1, z = c + 1 thì xy + yz + zx = 12 và M = .
x2
+ 8x + 12
Áp dụng bất đẳng thức AM-GM, ta có x2 + 4 ≥ 4x, y 2 + 4 ≥ 4y, z 2 + 4 ≥ 4z. Suy ra
x y z
M≤ + + .
12x + 8 12y + 8 12z + 8

3
Ta sẽ chứng minh vế trái của bất đẳng thức trên không vượt quá . Quy đồng và

iế n
16
biến đổi tương đương, ta cần chứng minh

20(x + y + z) + 96 ≥ 27xyz.

K
p
Theo bất đẳng thức AM-GM thì xy+yz+zx ≥ 3 3 (xyz)2 dẫn tới 27xyz ≤ 216, trong
khi đó vì (x + y + z)2 ≥ 3(xy + yz + zx) = 36 nên 20(x + y + z) + 96 ≥ 216 ≥ 27xyz.


Dấu bằng xảy ra khi và chỉ khi x = y = z = 2 hay a = b = c = 1 và giá trị lớn nhất

-T
3
của M là .
16

Câu 3
A1
Cho hình thoi ABCD (BAD\ < 90◦ ) có đường tròn nội tiếp (O). Các điểm M, N lần
lượt thuộc các cạnh CB, CD sao cho M N tiếp xúc (O) tại P , và tam giác CM N
nhọn không cân. Đường thẳng M N lần lượt cắt các đường thẳng AB, AD tại E, F .
n

Gọi K, L theo thứ tự là trực tâm của các tam giác BM E và DN F .


á

1. Chứng minh rằng OP đi qua trung điểm I của KL.


To

2. Gọi H là trực tâm của tam giác CM N . Chứng minh rằng


OI EF 1
− =− .
bộ

CH 2M N 2

3. Gọi giao điểm của EK, F L với BD lần lượt là S, T . Gọi giao điểm của N S
và M T là Q. Đường tròn nội tiếp của tam giác CM N tiếp xúc với M N tại
c

G. Chứng minh rằng hai đường thẳng P Q và GH song song.


lạ
u

Lời giải.

1. Do K, L tương ứng là trực tâm của 4BM E và 4DN F nên BK ⊥ M E và DL ⊥


N F . Hơn nữa, ta có OP ⊥ M N . Như vậy, ba đường thẳng BK, DL và OP đôi một
song song với nhau. Vì O là trung điểm của BD nên theo tính chất của đường trung
bình của hình thang ta suy ra OP đi qua trung điểm I của KL.

2. Do BE k CN và M H ⊥ CN (do H là trực tâm của 4CM N ) nên ta thu được


M H ⊥ BE. Trong khi đó, lại chú ý rằng M K ⊥ BE do K là trực tâm của 4BM E,
từ đó ta suy ra M, H, K thẳng hàng. Chứng minh tương tự ta cũng được N, H, L
112 Tuyển tập đề thi vào lớp 10 chuyên Toán

iế n
B O D

K
E K
L
P
M


H N

-T
F

C
A1
thẳng hàng. Mặt khác, vì OI là đường trung bình của hình thang BDLK nên áp
dụng định lý Thales ta có
2OI BK DL BM DN ME NF ME + NF
= + = + = + = .
CH CH CH CM CN MN MN MN
n

Do đó,
á
To

2OI EF M E + N F − EF M E + N F − (M E + M N + N F ) −M N
− = = = = −1.
CH M N MN MN MN
Vì vậy,  
OI EF 1 2OI EF 1
bộ

− = − =− .
CH 2M N 2 CH MN 2
c

3. Gọi X là giao điểm của ES và F T . Khi đó ta có X chính là trực tâm của 4AEF .
lạ

Ta sẽ chứng minh XP k GH. Thật vậy, vì AE k CN và AF k CM nên 4AEF ∼


4CN M (g.g). Khi đó, chú ý rằng P , G tương ứng là tiếp điểm của đường tròn nội
u

tiếp 4AEF và 4CN M với các cạnh EF và N M , vì thế ta suy ra được PP EF


GN
= GM .
Ngoài ra, do XEF
\ = HN M (XE k HN ) và XF E = HM N (XF k HM ) nên

\ \ \
PE GN
4XEF ∼ 4HN M (g.g), kết hợp với P F = GM ta chứng minh được 4XP F ∼
4HGM (c.g.c). Do vậy, XP
\ \ và vì thế XP k GH.
F = HGM

Như vậy, bây giờ ta chỉ cần chứng minh X, P, Q thẳng hàng. Gọi Q1 là giao
điểm của N S và XP . Áp dụng định lý Menelaus cho 4XP E với N, S, Q1 thẳng
hàng ta được
Q1 X N P SE Q1 X N E SX
· · = 1 =⇒ = · .
Q1 P N E SX Q1 P N P SE
Nguyễn Tiến Lâm - Trịnh Huy Vũ 113

iế n
B S T O D

E Q

K
K
P L
M G


J N

-T
H
F
C
A1
Gọi Q2 là giao điểm của M T và XP . Chứng minh tương tự như trên ta cũng được

Q2 X MF T X
= · .
Q2 P MP T F
n

Ta có XST
[ = BSE \ − AEX
[ = ABD \ − AF
\ = ADB \ X = DT
[ F = XT
[S và vì thế
á

4XST cân tại X và SX = T X.


To

Ngoài ra ta có 4BSE ∼ 4DT F (g.g) nên

SE BE BE DA ME NE ME NE
= = · = · = · .
TF DF BA DF MF NF NF MF
bộ

Mặt khác, (O) cũng chính là đường tròn bàng tiếp ứng với đỉnh C của 4CN M nên
theo kết quả quen thuộc ta được M P = N G và N P = M G. Do đó,
c

MP GN PE P E − MP ME
= = = = .
lạ

NP GM PF P F − NP NF

Vì thế ta được
u

SE ME NE MP NE NE 1 MF 1

= · = · =⇒ · = · .
TF NF MF NP MF N P SE MP T F

Do đó,
Q1 X N E SX MF T X Q2 X
= · = · = .
Q1 P N P SE MP T F Q2 P
Từ đó ta suy ra Q1 ≡ Q2 = N S ∩ M T và vì thế Q ≡ Q1 ≡ Q2 ∈ XP . Vậy X, P, Q
thẳng hàng. Kết hợp với XP k GH ta được P Q k GH.
114 Tuyển tập đề thi vào lớp 10 chuyên Toán

Câu 4
Giả sử a1 , a2 , ..., a2021 là các số thực thoả mãn
a1 a2 a2021
+ 2 + ··· + 2 = 0.
a21 + 1 a2 + 1 a2021 + 1

Chứng minh rằng tồn tại số nguyên k (1 ≤ k ≤ 2021) sao cho

iế n

a1 2a2 kak 2k + 1
a2 + 1 + a2 + 1 + · · · + a2 + 1 ≤ 8 .

1 2 k

K
2ai
Lời giải. Đặt xi = thì |xi | ≤ 1 với mọi i = 1, 2, ..., 2021 và
1 + a2i


-T
x1 + x2 + · · · + x2021 = 0.
k
X sk − sk−1
Lại đặt s0 = 0, s1 = x1 và sk = ixi với k = 1, 2, ..., 2021. Ta có xk = với
i=1
A1 k
k = 1, 2, ..., 2021 nên ta có
1 1 1 1
s1 + s2 + · · · + s2020 + s2021 = 0.
2 6 2020.2021 2021
n

Như vậy tồn tại k ∈ {1, 2, ..., 2021} để sk−1 sk ≤ 0. Lúc này nếu điều cần chứng minh
á

không đúng thì có mâu thuẫn là


To

2k + 1 2(k − 1) + 1
|kxk | = |sk − sk−1 | = |sk | + |sk−1 | > + ≥ k.
4 4
bộ
c
lạ
u

Nguyễn Tiến Lâm - Trịnh Huy Vũ 115

2.17. Đề thi tuyển sinh vào lớp 10 trường THPT


chuyên Sư phạm Hà Nội năm 2021, vòng 2

Câu 1

1+ 5
Cho α = .

iế n
2
a) Tìm một đa thức bậc hai Q(x) với hệ số nguyên sao cho α là nghiệm của
Q(x).

K
b) Cho đa thức P (x) = x5 − x4 − x + 1. Tính giá trị của P (α).


-T
Lời giải.

1. Từ đề bài suy ra 2α − 1 = 5. Bình phương hai vế ta được

4α2 − 4α − 4 = 0
A1
hay tương đương
α2 − α − 1 = 0.
Từ đó chọn Q(x) = x2 − x − 1 là đa thức thoả mãn đề bài.
n

2. Ta viết
á

P (x) = (x5 −x4 −x3 )+(x3 −x2 −x)+(x2 −x−1)+x+2 = x3 .Q(x)+x.Q(x)+Q(x)+x+2.


To


5+ 5
Vì α là nghiệm của Q(x) nên P (α) = α + 2 = .
2
bộ

Câu 2
c

Cho A, B là hai điểm cố định nằm trên đường tròn tâm O, bán kính R. Giả sử C
là điểm cố định trên tia đối của tia BA. Một cát tuyến thay đổi qua C cắt đường
lạ

tròn (O) tại D và E (D nằm giữa C và E). Các đường tròn ngoại tiếp các tam giác
BCD và ACE cắt nhau tại giao điểm thứ hai M . Biết rằng bốn điểm O, B, M, E
u

tạo thành tứ giác OBM E. Chứng minh rằng


a) Tứ giác OBM E nội tiếp.

b) CD · CE = CO2 − R2 .

c) M luôn di chuyển trên một đường tròn cố định.

Lời giải.
116 Tuyển tập đề thi vào lớp 10 chuyên Toán

iế n
O D

K
M


A B C

-T
A1
a) Do các tứ giác ACM E, BCM D và ABDE là các tứ giác nội tiếp nên ta được EAB
[ =
[ = 180 − EM◦
EAC \ C, BM
\ C = BDC
\ = EAB.
[ Vì vậy, ta được
n

EM
\ B = EM
\ C − BM
\ C = 180◦ − EAB \ = 180◦ − 2EAB
[ − BDC [ = 180◦ − EOB.
\
á

Vậy EM
\ \ = 180◦ , do đó tứ giác OBM E là tứ giác nội tiếp.
B + EOB
To

b) Gọi N là trung điểm của DE. Do 4ODE cân tại O nên ON ⊥ DE, đồng thời ON là
phân giác của DOE.
\ Vì vậy ta được
bộ

CO2 − R2 = CO2 − OD2 = (ON 2 + CN 2 ) − (ON 2 + DN 2 ) = CN 2 − DN 2


= (CN − DN )(CN + DN ) = CD · (CN + N E) = CD · CE.
c

c) Vì O là tâm ngoại tiếp của 4BDE nên ta được EBD\ = 1 EOD \ = EON \ . Kết hợp
lạ

2
với việc các tứ giác OBM E và BCM D là cá tứ giác nội tiếp, ta suy ra M \ ON =
M OE − EON = M BE − EBD = M BD = M CD = M CN . Từ đó suy ra tứ giác
\ \ \ \ \ \ \
u

M CON là tứ giác nội tiếp. Vì thế OM


\ C = ON
\ C = 90◦ . Chú ý rằng O, C đều là các
điểm cố định. Như vậy M luôn chạy trên đường tròn cố định với đường kính OC.

Câu 3
Tìm tất cả các số nguyên dương N sa cho N có thể biểu diễn một cách duy nhất ở
x2 + y
dạng với x, y là hai số nguyên dương.
xy + 1
Nguyễn Tiến Lâm - Trịnh Huy Vũ 117

x2 + y
Lời giải. +) Với N = 1, ta chỉ ra cặp có hai cặp (x, y) = (1, 1), (1, 2) để N = .
xy + 1
+) Với N ≥ 2, vì N là số nguyên dương bất kỳ lớn hơn 1 thì N có thể biểu diễn thành
N4 + N x2 + y
N = 2 nên nếu chọn x = N 2 , y = N thì N = . Vậy với mọi N > 1 đều
N ·N +1 xy + 1
có ít nhất một cách biểu diễn dưới dạng đã cho.
Ta có

iế n
xy + 1 | x2 + y
x2 + y
nên xy + 1 | y(x2 + y) − x(xy + 1) = y 2 − x. Từ đây lưu ý rằng nếu x ≤ y thì ≤ 1.
xy + 1

K
x2 + y
Do đó để ≥ 2 thì x > y. Khi đó có
xy + 1


−xy − 1 < −x < y 2 − x < y 2 < xy + 1

-T
Từ đây ta phải có x = y 2 . Vậy ứng với mỗi N là số tự nhiên, N > 1 thì chỉ có đúng một
cách chọn là y = N, x = N 2 .
Vậy đáp số là các số nguyên dương N ≥ 2. A1
Câu 4
Cho a, b, c là ba số nguyên dương sao cho mỗi số đó đều biểu diễn được ở dạng luỹ
n
thừa của 2 với số mũ tự nhiên.
Biết rằng phương trình bậc hai ax2 − bx + c = 0 (1) có cả hai nghiệm đều là số
á

nguyên. Chứng minh rằng hai nghiệm của phương trình (1) bằng nhau.
To

Lời giải.
Cách 1. Ta viết a = 2m , b = 2n , c = 2k trong đó m, n, k là các số tự nhiên và gọi hai
bộ

nghiệm đã cho là x1 , x2 . Theo định lý Vieta, ta có x1 + x2 = 2n−m và x1 x2 = 2k−m . Từ


đây suy ra x1 , x2 > 0 và đều là luỹ thừa của 2. Từ đó ta có thể viết x1 = 2p và x2 = 2q
với p, q đều là các số tự nhiên và không mất tính tổng quát giả sử p ≥ q. Thế thì ta có
c

2p + 2q = 2n−m hay tương đương


lạ

2q (2p−q + 1) = 2n−m .

Nếu p > q thì 2p−q + 1 là số lẻ lớn hơn 1, trong khi đó vế phải của đẳng thức trên là luỹ
u

thừa của 2, mâu thuẫn. Suy ra p = q và ta có điều phải chứng minh.


Cách 2. Ta đặt a = 2x , b = 2y , c = 2z , với x, y, z là các số tự nhiên


Xét phương trình (1), ta có:

∆ = b2 − 4ac = 22y − 22+x+z = 22+x+z (22y−x−z−2 − 1)


Do phương trình (1) có 2 nghiệm đều là số nguyên nên ∆ là số chính phương. Dẫn tới số
mũ của 2 trong ∆ là chẵn, tức là 2 + x + z chẵn. Ta đặt x + z + 2 = 2k. Khi đó:

∆ = 22k (22y−2k − 1).


118 Tuyển tập đề thi vào lớp 10 chuyên Toán

Mà cả ∆ và 22k đều là số chính phương, 22k > 0 cho nên 22y−2k − 1 cũng phải là số chính
phương. Mà 22y−2k = 22(y−k) là số chính phương, cho nên 22y−2k = 1, và ∆ = 0 khiến 2
nghiệm của phương trình trùng nhau.

iế n
K

-T
A1
á n
To
bộ
c
lạ
u

Nguyễn Tiến Lâm - Trịnh Huy Vũ 119

2.18. Đề thi tuyển sinh vào lớp 10 chuyên Toán


Hà Nội, năm 2021

Câu 1

1. Giải phương trình x2 + x + 2 − 2 x + 1 = 0.

iế n
2. Cho ba số thực a, b và c thoả mãn ab + bc + ca = 1. Chứng minh
a−b b−c c−a
+ + = 0.

K
1+c 2 1+a 2 1 + b2


Lời giải.

-T
1. Điều kiện xác định: x ≥ −1.
Ta viết lại phương trình dưới dạng

x2 + ( x + 1 − 1)2 = 0.
A1
√ √
Vì x2 ≥ 0, ( x + 1 − 1)2 ≥ 0 nên ta phải có x2 = ( x + 1 − 1)2 = 0 và từ đó suy
ra x = 0.
n

2. Gọi vế trái của đẳng thức cần chứng minh là P. Vì ab + bc + ca = 1 nên a2 + 1 =


á

a2 + ab + bc + ca = a.(a + b) + c.(a + b) = (a + b)(a + c).


To

Tương tự b2 + 1 = (b + a)(b + c) và c2 + 1 = (c + a)(c + b).


Từ đó suy ra
bộ

a−b b−c c−a (a − b) b−c c−a


P = 2
+ 2
+ 2
= + + .
1+c 1+a 1+b (c + a)(c + b) (a + b)(a + c) (b + a)(b + c)

Quy đồng, ta được


c
lạ

(a − b)(a + b) + (b − c)(b + c) + (c − a)(c + a) a2 − b 2 + b 2 − c 2 + c 2 − a2


P = = = 0.
(a + b)(b + c)(c + a) (a + b)(b + c)(c + a)
u

Câu 2
1. Tìm tất cả các cặp số nguyên (x, y) thoả mãn x2 + 5xy + 6y 2 + x + 2y − 2 = 0.

2. Chứng minh rằng với mỗi số nguyên n, số n2 + n + 16 không chia hết cho 49.

Lời giải.
120 Tuyển tập đề thi vào lớp 10 chuyên Toán

1. Ta viết x2 +5xy+6y 2 = x2 +2xy+3xy+6y 2 = x(x+2y)+3y(x+2y) = (x+3y)(x+2y).


Từ đó phương trình đã cho có thể viết lại thành

(x + 2y)(x + 3y + 1) = 2.

Vì x, y là các số nguyên, nên x + 2y và x + 3y + 1 là các số nguyên. Nên ta chỉ cần


xét 4 trường hợp sau:

iế n
• Trường hợp 1. x + 2y = 1 và x + 3y + 1 = 2 và ta tìm được (x, y) = (1, 0).
• Trường hợp 2. Với x + 2y = 2 và x + 3y + 1 = 1 thì y = −2 và x = 6.

K
• Trường hợp 3. Với x + 2y = −1 và x + 3y + 1 = −2 thì y = −2 và x = 3.
• Trường hợp 4. Với x + 2y = −2 và x + 3y + 1 = −1 thì y = 0 và x = −2.


Vậy các cặp (x, y) cần tìm là (1, 0); (6, −2); (3, −2); (−2, 0).

-T
2. Giả sử tồn tại số nguyên n sao cho n2 + n + 16 chia hết cho 49. Suy ra 4.(n2 + n + 16)
chia hết cho 49. Mà
A1
4 · (n2 + n + 16) = 4n2 + 4n + 1 + 63 = (2n + 1)2 + 63

Vì 4 · (n2 + n + 16) chia hết cho 49 nên (2n + 1)2 + 63 chia hết cho 7, kéo theo
(2n + 1)2 chia hết cho 7. Do đó, (2n + 1)2 chia hết cho 49. Nhưng khi đó, vì 63
không chia hết cho 49 nên (2n + 1)2 + 63 không chia hết cho 49. Điều mâu thuẫn
n

này chứng tỏ giả sử là sai và từ đó ta có điều phải chứng minh.


á
To

Câu 3
2
1. Cho số thực x khác 0 thoả mãn x + và x3 là số hữu tỉ. Chứng minh x là số
bộ

x
hữu tỉ.

2. Cho các số thực không âm a, b và c thoả mãn a + b + c = 5. Chứng minh rằng


c

2a + 2ab + abc ≤ 18.


lạ

Lời giải.
u

2 x2 + 2 x4 + 2x2
1. Ta có: x + = = ∈ Q ⇒ x4 + 2x2 ∈ Q.
x x x3

2 4
Mặt khác do x + ∈ Q nên x2 + 2 ∈ Q. Suy ra
x x
4 8 x6 − 8 x7 − 8x
x4 + 2x2 − 2(x2 + ) = x 4
− = = ∈ Q.
x2 x2 x2 x3

Từ đó x7 − 8x ∈ Q hay x(x6 − 8) ∈ Q.
Chú ý x6 − 8 6= 0 do x3 ∈ Q nên x6 − 8 ∈ Q nên x ∈ Q.
Nguyễn Tiến Lâm - Trịnh Huy Vũ 121

(b + c + 2)2 (7 − a)2
2. Theo BĐT AM-GM, ta có 2ab + abc = ab(2 + c) 6 a =a . Do
4 4
đó,
(7 − a)2
2a + 2ab + abc 6 2a + a .
4

(7 − a)2
Giờ, ta sẽ chứng minh 2a + a 6 18. Bất đẳng này thức tương đương với

iế n
4

(a − 8)(a − 3)2 6 0

K
luôn đúng. Dấu bằng xảy ra khi a = 3, b = 2, c = 0.


-T
Câu 4
[ = 60◦ và AB < AC.
Cho tam giác nhọn ABC nội tiếp đường tròn (O), với BAC
Các đường thẳng BO, CO lần lượt cắt các đoạn thẳng AC, AB tại M, N . Gọi F là
điểm chính giữa của cung BC lớn.
A1
(a) Chứng minh năm điểm A, N, O, M và F cùng thuộc một đường tròn.

(b) Gọi P, Q lần lượt là các giao điểm thứ hai của hai tia F N, F M với đường tròn
(O). Gọi J là giao điểm của đường thẳng BC và đường thẳng P Q. Chứng minh
n

tia AJ là tia phân giác của góc BAC.


[
á
To

(c) Gọi K là giao điểm của đường thẳng OJ và đường thẳng CF . Chứng minh AB
vuông góc với AK.
bộ

Lời giải.
(a) Do O là tâm ngoại tiếp của 4ABC nên M \ON = BOC\ = 2BAC [ = 120◦ . Như vậy,
c

M
\ AN + M\ ON = 60◦ + 120◦ = 180◦ và vì thế A, M, O, N cùng thuộc một đường tròn.
lạ

Trong khi đó, vì F là trung điểm cung lớn BC nên F B = F C, hơn nữa kết hợp với
BF
\ [ = 60◦ ta thu được 4BF C đều. Từ đó, F O là phân giác BF
C = BAC \ C và do đó
u

◦ ◦
BF O = 30 . Như vậy, AM O = M BC + ACB = 30 + AF B = BF O + AF B = AF
\ \ \ [ [ \ [ [ O.

Từ đó suy ra A, F, M, O cùng thuộc một đường tròn. Vì vậy năm điểm A, N, O, M


và F cùng thuộc một đường tròn.

(b) Ta có AN
\ F = AOF
[ = 2ABF[ , suy ra 4N BF cân tại N . Do đó, AP [ F = ABF
[ =
F B, từ đó ta được AP k F B và AP BF là hình thang cân. Vì vậy, M
P[ \ QJ = F[
QP =
F[
BP = BF [ A = BCA
[ = JCM \ , từ đó suy ra tứ giác CM JQ nội tiếp và vì thế
CJM
\ = CQM \ = CQF [ = CBF \ = 60◦ = BAC [ và tứ giác ABJM nội tiếp. Vậy
◦ 1
JAM
\ = JBM \ = CBO \ = 30 = BAC [ và ta suy ra được AJ là phân giác của BAC.
2
[
122 Tuyển tập đề thi vào lớp 10 chuyên Toán

F
A

iế n
N

K
P O


-T
B J C

Q
A1
(c) Gọi D là giao điểm của AJ và F B. Vì các tứ giác N OJB, AM ON, AM JB là các tứ
n

giác nội tiếp nên ta được CJK


[ = CJO[ = BN \ O = AM
\ O = AM
\ B = AJB
[ = BJD.[
á

Lại chú ý rằng JBD


[ = JCK [ = 60◦ nên 4BJD ∼ 4CJK (g.g), kết hợp với tính
BD
= BJ = AB
To

chất phân giác ta suy ra được CK CJ AC


. Do đó, 4ABD ∼ 4ACK (c.g.c), như
vậy, CAK
\ = BAD \ = BAJ [ = 30◦ . Vì vậy, BAK
\ = BAC \ = 60◦ + 30◦ = 90◦ .
[ + CAK
bộ

Câu 5
Cho A là một con có 100 phần tử của tập hợp {1, 2, 3, ..., 178}.
c

1. Chứng minh A chứa hai số tự nhiên liên tiếp.


lạ

2. Chứng minh với mọi số tự nhiên n thuộc tập hợp {2, 3, ..., 22}, tồn tại hai
phần tử của A có hiệu bằng n.
u

Lời giải.
1. Ta chia tập {1, 2, ..., 178} thành 89 cặp liên tiếp {1, 2}, {3, 4}, ..., {177, 178}. Vì A
chứa 100 phần tử mà chỉ có 89 cặp nên theo nguyên lý Dirichlet phải có hai phần
tử cùng thuộc một cặp và hai phần tử này là hai số liên tiếp.

2. Xét số tự nhiên n, với 2 ≤ n ≤ 22. Giả sử 178 chia cho 2n dư k với k < 2n, ta đặt
178 = 2an + k.
Nguyễn Tiến Lâm - Trịnh Huy Vũ 123

Ta chia tập hợp {1, 2, ..., 178} thành các cặp dưới đây

(1, 1 + n), (2, 2 + n), ...(n, 2n), (2n + 1, 3n + 1), (2n + 2, 3n + 2), ..., (2an − n, 2an),

và thêm các nhóm

• {2an + 1}, {2an + 2}, ..., {2an + k} nếu k < n.


• (2an+k, 2an+k −n), (2an+k −1, 2an+k −1−n), ..., (2an+k −n, 2an+k −2n)

iế n
nếu k ≥ n.

Do 2an + k − 2an ≤ 2an nên các cặp trên sẽ phủ hết tập A. Và ta có

K
• an + k cặp nếu k < n
• an + n cặp nếu k ≥ n


-T
n+k
Số cặp luôn nhỏ hơn hoặc bằng an + , từ đó ta có
2

• Nếu n = 22, ta có 178 = 8.22 + 2 nên số cặp là 4.22 + 2 = 90 < 100, theo
nguyên lý Dirichlet, có 2 số thuộc cùng 1 cặp, vậy có 2 số có hiệu là 22
A1 n+k
• Nếu n ≤ 21, ta có 2an+n+k = 178+n ≤ 178+21 = 199 dẫn tới an+ <
2
100 nên số cặp nhỏ hơn hẳn 100. Do đó, theo nguyên lý Dirichlet thì sẽ có 2
số thuộc cùng 1 cặp, do đó chúng có hiệu là n.
á n
To
bộ
c
lạ
u

124 Tuyển tập đề thi vào lớp 10 chuyên Toán

2.19. Đề thi tuyển sinh vào lớp 10 chuyên Tin Hà


Nội, năm 2021

Câu 1

1. Giải phương trình 4 + 2x − x2 = x − 2.

iế n
(
x3 + 2 = 3y
2. Giải hệ phương trình .
y 3 + 2 = 3x

K
Lời giải.


1. Điều kiện xác định : 4 + 2x − x2 ≥ 0. Với x ≥ 2, bình phương hai vế của phương

-T
trình đã cho, ta có
4 + 2x − x2 = (x − 2)2
hay tương đương
2x2 − 6x = 0.
A1
Giải ta tìm được x = 0 và x = 3. Loại nghiệm x = 0 nên phương trình đã cho có
đúng một nghiệm x = 3.
n

2. Trừ vế với vế tương ứng của hệ phương trình ta được và phân tích thành thừa số
ta được
á

(x − y)(x2 − xy + y 2 + 3) = 0.
To

 y 2 3 2
Vì x2 − xy + y 2 + 3 = x − + y + 3 > 0 nên ta phải có x = y. Từ đây thay
2 4
vào phương trình đầu tiên của hệ, ta được
bộ

x3 + 2 = 3x

hay tương đương


c

(x − 1)2 (x + 2) = 0.
lạ

Giải phương trình trên, suy ra x = 1, x = −2. hệ đã cho có hai nghiệm (x, y) là
(1, 1) và (−2, −2).
u

Câu 2
1. Chứng minh rằng với mỗi số nguyên n, số n2 + 3n + 16 không chia hết cho 25.

2. Tìm tất cả các số nguyên x và y thoả mãn x2 − xy − 2y 2 + x + y − 5 = 0.

Lời giải.
Nguyễn Tiến Lâm - Trịnh Huy Vũ 125

1. Giả sử phản chứng n2 +3n+16 chia hết cho 25, thế thì 4(n2 +3n+16) = (2n+3)2 +55
cũng chia hết cho 25. Suy ra (2n + 3)2 + 55 chia hết cho 5. Vì 55 chia hết cho 5 nên
(2n + 3)2 chia hết cho 5, kéo theo 2n + 3 chia hết cho 5. Do đó, (2n + 3)2 chia hết
cho 25, nhưng khi đó (2n + 3)2 + 55 lại không chia hết cho 25, mâu thuẫn. Vậy, điều
giả sử là sai và từ đó ta có điều phải chứng minh

2. Viết lại phương trình đã cho dưới dạng

iế n
(x + y)(x − 2y + 1) = 5.

Từ đây, ta có 4 trường hợp

K
• Với x + y = 1 và x − 2y + 1 = 5 thì x = 2, y = −1.
• Với x + y = 5 và x − 2y + 1 = 1 thì 3y = 5 loại vì y nguyên.


• Với x + y = −1 và x − 2y + 1 = −5 thì 3y = 5 loại vì y nguyên.

-T
• Với x + y = −5 và x − 2y + 1 = −1 thì y = −1 và x = −4.

Vậy (x, y) là (2, −1) và (−4, −1).


A1
Câu 3
1. Cho a, b, c thực đôi một phân biệt. Chứng minh:
n
        
a+b b+c b+c c+a c+a a+b
+ + = −1.
á

a−b b−c b−c c−a c−a a−b


To

a b c
2. Cho biểu thức P = √ +√ +√ với a, b, c không âm và
1 + 2bc 1 + 2ca 1 + 2ab
a2 + b2 + c2 = 1.
bộ

Tìm giá trị lớn nhất của P .

Lời giải.
c

a+b b+c c+a


lạ

1. Đặt x = ,y = ,z = . Cần chứng minh: xy + yz + zx = −11.


a−b b−c c−a

2a 2b 2c
 x+1=
 ,y + 1 = ,z + 1 =
a−b b−c c−a
u

Khi đó 2b 2c 2a
 x−1=
 ,y − 1 = ,z − 1 =

a−b b−c c−a


Suy ra (x + 1)(y + 1)(z + 1) = (x − 1)(y − 1)(z − 1). Khai triển phá ngoặc, ta được
xy + yz + zx = −1.
1
2. Áp dụng bất đẳng thức Bunhiacovksy, ta có a2 + (b + c)2 > (a + b + c)2 . Suy ra
2

a a a 2a
√ =√ =p 6 .
1 + 2bc a2 + b2 + c2 + 2bc a2 + (b + c)2 a+b+c
126 Tuyển tập đề thi vào lớp 10 chuyên Toán

√ √
b 2b c 2c
Tương tự ta có √ 6 và √ 6 .
1 + 2ca a+b+c 1 + 2ab a+b+c

 
1 1
Từ đó, P 6 2 và dấu bằng xảy ra khi (a, b, c) là hoán vị của √ , √ , 0 . Vậy,
√ 2 2
giá trị lớn nhất của P là 2.

iế n
Câu 4
Cho tam giác nhọn ABC nội tiếp đường tròn (O) và AB < AC. Gọi I là tâm đường

K
tròn nội tiếp của tam giác ABC. Đường thẳng AI cắt đường tròn (O) tại điểm thứ
hai M (M khác A). Gọi D, E và F lần lượt là các hình chiếu của điểm I trên các
cạnh BC, CA và AB.


-T
(a) Chứng minh tam giác M BI là tam giác cân.

(b) Đường tròn ngoại tiếp tam giác AEF cắt đường tròn (O) tại điểm thứ hai P
(P khác A). Chứng minh P, M và D là ba điểm thẳng hàng.
A1
(c) Gọi H là giao điểm của đường thẳng IP và đường thẳng EF . Chứng minh HD
song song với AM .
n

A
á
To

P
bộ

E
H
F

I
c
lạ

B D C
u

Lời giải.
(a) Ta có phép biến đổi góc M
\ BI = M
\ BC + IBC
[ =M\AC + IBA
[ = IAB+
[ IBA[=M
\ IB,
vì thế 4M BI cân tại M .
Nguyễn Tiến Lâm - Trịnh Huy Vũ 127

(b) Do tứ giác AEF P nội tiếp nên AF [ P = AEP


[ hay P[ F B = P[ EC. Kết hợp với
P BF = P BA = P CA = P CE, nên ta suy ra được 4P F B ∼ 4P EC (g.g). Chú ý
[ [ [ [
rằng BF = BD và CE = CD, ta thu được PP B C
= BF
CE
= BD
CD
. Theo tính chất phân
giác ta suy ra P D là phân giác của BP
\ C. Mà M là trung điểm của cung BC không
chứa A, do vậy ta suy ra được ba điểm P, D, M thẳng hàng.

(c) Từ 4P F B ∼ 4P EC ta suy ra F[ P E = F[ P C + CP
[ E = F[P C + BP
[ F = BP
\ C và

iế n
PF PB
PE
= PC
, vì thế 4P F E ∼ 4P BC (c.g.c). Ta có IE = IF nên P I là phân giác
của F[P E. Ta chứng minh được 4P F H ∼ 4P BD (g.g) và 4IF H ∼ 4M BD (g.g).
Từ đó suy ra PP H = HF IH
= DM hay PHIH = DM
PD
. Theo định lý Thales đảo ta suy ra

K
D BD
HD k AM .


-T
Câu 5
Trên bàn có n viên kẹo. Hai bạn An và Bình cùng chơi một trò chơi như sau: Hai
bạn luân phiên lấy kẹo trên bàn, mỗi lần chỉ được lấy 1, 2, 3, 4 hoặc 5 viên kẹo và
phải lấy số viên kẹo khác với số viên kẹo của bạn còn lại vừa lấy ngay trước đó.
A1
Bạn đầu tiên không thể thực hiện được lượt chơi của mình là người thua cuộc. Nếu
An là người lấy kẹo trước,

1. Với n = 7, hãy chỉ ra chiến thuận chơi của Bình khiến An là người thua cuộc.
n

2. Với n = 22, hãy chỉ ra chiến thuận chơi của An khiến là người thua cuộc.
á
To

Lời giải.
1. Chiến thuật chơi của Bình
bộ

• Nếu An lấy 1 viên kẹo trong lượt đầu thì Bình lấy 3 viên, lúc này còn 3 viên
kẹo. Lượt sau An không thể lấy 3 viên nên chỉ có thể lấy k viên với 1 ≤ k ≤ 2,
khi đấy Bình sẽ lấy 3 − k viên còn lại và thắng (lưu ý 3 − k 6= k ∀k ∈ Z và
c

1 ≤ 3 − k ≤ 2 nên chiến thuật này có thể làm)


lạ

• Nếu An lấy ` viên kẹo trong lượt đầu (2 ≤ ` ≤ 5) thì Bình lấy nốt 7 − ` viên
còn lại. Lưu ý ` 6= 7 − ` ∀` ∈ Z và 2 ≤ 7 − ` ≤ 5 nên chiến thuật này có thể
làm.
u

2. Chiến thuật chơi của An như sau

• Lượt đầu, An bốc 2 viên, còn 20 viên.


• Tại lượt sau nếu Bình bốc k viên với k ≥ 2 thì An bốc 7 − k viên, còn 13
viên. Nếu Bình bốc 1 viên thì An bốc 3 viên. Khi đó nếu Bình bốc ` viên với
1 ≤ ` ≤ 2 thì An bốc 3 − ` viên. còn 13 viên. Nếu Bình bốc ` viên với 4 ≤ ` ≤ 5
thì An bốc 9 − ` viên (lưu ý khi này 4 ≤ 9 − ` ≤ 5), còn lại 7 viên. Như vậy
sau bước này thì số bi còn lại là 13 viên hoặc 7 viên và đến lượt Bình bốc.
128 Tuyển tập đề thi vào lớp 10 chuyên Toán

• Nếu số bi còn lại 7 viên thì An chỉ cần dựa theo chiến thuật tại câu 1) thì sẽ
chiến thắng. Nếu số bi còn lại 13 viên thì An sẽ bốc tiếp như sau. Nếu Bình
bốc m viên với m 6= 3 thì An bốc 6 − m viên, lúc này còn lại 7 viên và đến
lượt Bình bốc, khi đó An chỉ cần dựa theo chiến thuật câu 1) là thắng. Nếu
Bình bốc 3 viên thì An sẽ bốc 5 viên, còn lại 5 viên và đến lượt Bình bốc. Tuy
nhiên Bình chỉ có thể bốc t viên với 1 ≤ t ≤ 4 (do t 6= 5 do An bốc 5 viên ở
lượt trước), khi đó tại lượt sau An sẽ bốc 5 − t viên còn lại và chiến thắng.

iế n
K

-T
A1
á n
To
bộ
c
lạ
u

Nguyễn Tiến Lâm - Trịnh Huy Vũ 129

2.20. Đề thi tuyển sinh vào lớp 10 trường THPT


chuyên Khoa học Tự nhiên năm 2022, vòng 1

Câu 1: (4 điểm)

1. Giải hệ phương trình

iế n
(
6(xy + 5) + x3 y + 5x2 = 42
.
x3 + 5x2 y + 6x + 30y = 42

K
2. Giải phương trình
√ √


p
( 3 x + 6 + 3 3 − x)(2 + 3 3 (x + 6)(3 − x)) = 24.

-T
Lời giải.
1. Trừ vế với vế của 2 phương trình ta được A1
(x3 y − x3 ) + (6xy − 6x) − (5x2 y − 5x2 ) − (30y − 30) = 0

tương đương với


(y − 1)(x − 5)(x2 + 6) = 0.
n

Từ đây ta có y = 1 hoặc x = 5.
á

• Với y = 1 thì x = 1.
To

−113
• Với x = 5 thì y = .
155
 
−113
bộ

Vậy hệ phương trình có nghiệm (x, y) là (1, 1); 5, .


155
√ √
2. Đặt 3 x + 6 = a, 3 3 − x = b.
c

(
(a + b)(2 + 3ab) = 24
lạ

Khi đó ta có hệ phương trình . Cộng hai phương trình lại


a3 + b 3 = 9
ta suy ra (a + b)3 + 2(a + b) − 33 = 0 hay tương đương
u

(a + b − 3)((a + b)2 + 3(a + b) + 11) = 0.


Chú ý là (a + b)2 + 3(a + b) + 11 > 0 nên ta phải có a + b = 3, kéo theo ab = 2. Giải


ta tìm được (a, b) là (1, 2) hoặc (2, 1). Xét hai trường hợp

• a = 1, b = 2. Giải tìm được x = −5.


• a = 2, b = 1. Giải tìm được x = 2.

Vậy hệ phương trình có hai nghiệm là 2 và −5.


130 Tuyển tập đề thi vào lớp 10 chuyên Toán

Câu 2: (2 điểm)

1) Tìm tất cả các cặp số nguyên (x, y) thoả mãn đẳng thức

25y 2 + 354x + 60 = 36x2 + 305y + (5y − 6x)2022 .

2) Trên bàn có 8 hộp rỗng (trong các hộp không có viên bi nào). Người ta thực

iế n
hiện các lần thêm bi vào các hộp theo quy tắc sau: mỗi lần ta chọn ra 4 bất
kỳ và bỏ vào một hộp 1 viên, một hộp 2 viên, hai hộp còn lại mỗi hộp 3 viên.
Hỏi số lần thêm bi ít nhất có thể nhận được số bi ở 8 hộp trên là 8 số tự nhiên

K
liên tiếp?


Lời giải.

-T
1) Phương trình đã cho có thể viết lại dưới dạng

(5y − 6x − 1)(5y + 6x − 60) = (5y − 6x)2022 .

Rõ ràng (5y − 6x − 1, 5y − 6x) = 1 nên (5y − 6x − 1, (5y − 6x)2022 ) = 1, từ đó ta


A1
phải có 5y − 6x − 1 = ±1. Xét hai trường hợp

– Nếu 5y − 6x − 1 = 1 thì 5y + 6x − 60 = 22022 . Cộng lại, ta có 10y = 22022 + 62.


Tuy nhiên 22022 = (24 )505 .22 = 16505 .4 có tận cùng là 4 nên 22022 + 62 có tận
n

cùng là 8 không chia hết cho 10. Trường hợp này phương trình vô nghiệm.
á

– Nếu 5y − 6x − 1 = −1 thì 5y + 6x − 60 = 0. Từ đây tìm được y = 6, x = 5.


To

Vậy phương trình đã cho có đúng một nghiệm là (5; 6).

2) Gọi số lần thêm ít nhất là n thì các hộp sẽ có số bi tương ứng là a, a+1, a+2, . . . , a+7.
bộ

Mỗi lần thêm tất cả là 9 bi nên ta có

a + (a + 1) + (a + 2) + · · · + (a + 7) = 9n
c

hay tương đương


lạ

8a + 28 = 9n.
Suy ra 4(2a + 7) chia hết cho 9, kéo theo 2a + 7 chia hết cho 9. Từ đó ta phải có
u

2a + 7 ≥ 9 dẫn tới n ≥ 4. Ta chứng minh n = 4 thoả mãn.


Gọi các hộp tương ứng là 1, 2, 3, 4, 5, 6, 7, 8 và thực hiện thêm bi ở các lần như sau

– Lần 1: 1, 2, 0, 0, 0, 0, 3, 3;
– Lần 2: 0, 0, 1, 0, 2, 3, 0, 3;
– Lần 3: 0, 0, 2, 1, 3, 0, 3, 0;
– Lần 4: 0, 0, 0, 3, 0, 3, 1, 2.
Nguyễn Tiến Lâm - Trịnh Huy Vũ 131

Câu 3: (3 điểm)

Cho hình chữ nhật ABCD (AB < AD) nội tiếp trong đường (O). Trên cạnh AD
lấy hai điểm E và F (E, F không trùn với A, D) sao cho E nằm giữa A và F, đồng
thời ABE
[ + DCF \ = 1 BOC.\
2

1) Chứng minh BE và CF cắt nhau tại một điểm nằm trên đường tròn (O).

iế n
2) Đường thẳng qua O song song với BC cắt BE, CF theo thứ tự tại M, N.
Chứng minh rằng DAM
\ + ADN\ + 1 AOD \ = 180◦ .
2

K
3) Dựng hình chữ nhật M N P Q sao cho N Q song song với BD, đồng thời M P
song song với AC. Chứng minh rằng đường tròn ngoại tiếp hình chữ nhật
M N P Q tiếp xúc với đường tròn (O).


-T
K

D
A E F
A1
O
M N
á n

J
To

X
B C
Q P
bộ

Lời giải.
c

1) Đặt BE ∩ CF = K. Gọi H là hình chiếu của K trên AD. Để ý rằng KH k AB avà


\ = 1 BOC.
lạ

KH k CD nên ta được BKC


\ = BKH \ + CKH \ = ABE [ + DCF
2
\ Từ đó
suy ra K ∈ (O).
u

2) Trước hết để ý rằng KM\ O = KBC


\ = KAC \ = KAO \ ta thu được tứ giác KAM O
là tứ giác nội tiếp. Chứng minh tương tự, ta cũng có KDN O là tứ giác nội tiếp.

Đặt AM ∩ DN = T . Khi đó, ta có T[ AC = M \ AO = M\ \−N


KO = BKC \KO =
BDC − N DO = T DC. Từ đó suy ra T ∈ (O). Vì vậy,
\ \ [

DAM \ + 1 AOD
\ + ADN \ = 180◦ − AT
[ D+
1\
AOD = 180◦ .
2 2

3) Gọi M P ∩ N Q = J. Để ý rằng JM
\ N = OCB
\ = OBC \ = JN \ M nên JM = JN .
Như vậy, ta quy về chứng minh đường tròn (J, JM ) tiếp xúc với (O).Thật vậy, ta
132 Tuyển tập đề thi vào lớp 10 chuyên Toán

cần chứng minh JM + OJ = R với R là bán kính của (O). Đặt KO ∩ BC = X.


Khi đó, để ý rằng OM
ON
= XB
XC
, ta thấy 4JM N ∪ O ∼ 4OBC ∪ X. Do đó, ta được

KO OM MN JO JM JM JM + JO JM + JO
= = = = = = = .
KX BX BC OX OB OK OK + OX KX

Suy ra , JM + JO = OK = R. Như vậy, (M N P Q) ≡ (J, JM ) tiếp xúc trong với


(O).

iế n
Câu 4: (1 điểm)

K
Cho a, b, c là các số thực dương. Chứng minh rằng


2a a + b 6a + 2c 4a + 3b + c 32a
+ + + ≥ .

-T
a+b a+c 3b + c b+c 2a + b + c

Lời giải.
Ta viết lại vế trái dưới dạng
A1
     
2a 2a b+c 2(a + c) 4a 4a 3b + c 3b + c
P = + + −1 + + + + +
a+b a+c a+c 3b + c 3b + c b + c 2(b + c) 2(b + c)
n
 
2a 2a 4a 4a 3b + c b + c 2(a + c) 3b + c
= + + + + + + + − 1.
a + b a + c 3b + c b + c 2(b + c) a+c 3b + c 2(b + c)
á
To

b + c 2(a + c) 3b + c
Áp dụng bất đẳng thức AM-GM thì + + ≥ 3 nên
a+c 3b + c 2(b + c)

2a 2a 4a 4a 3b + c
P ≥ + + + + + 2.
bộ

a + b a + c 3b + c b + c 2(b + c)
r r
2a 2a 8a 4a 3b + c 2a 4a 2a
Lưu ý là + ≥ , + ≥2 và +2 ≥ 4
a+b a+c 2a + b + c 3b + c 2(b + c) b+c b+c b+c
c

nên
lạ

r
8a 2a 8a 12a 8a 24a 32a
P ≥ +6 = +p ≥ + = .
2a + b + c b+c 2a + b + c 2a(b + c) 2a + b + c 2a + b + c 2a + b + c
u

Phép chứng minh hoàn tất.



Nguyễn Tiến Lâm - Trịnh Huy Vũ 133

2.21. Đề thi tuyển sinh vào lớp 10 trường THPT


chuyên Khoa học Tự nhiên năm 2022, vòng 2

Câu 1: (3.5 điểm)


1 1 1
1) Với a, b, c là những số thực dương thỏa mãn điều kiện + + = 1. Chứng

iế n
a b c
minh rằng
  s
1 1 1 1 abc

K
+ + = .
2 a + bc b + ca c + ab (a + bc)(b + ca)(c + ab)


2) Giải hệ phương trình

-T
(
2x2 + 3xy + y 2 = 6

3x + 2y + 1 = 2 2x + y + 6.
A1
Lời giải.
1) Từ điều kiện ta có được ab + bc + ca = abc. Từ đó
 
1 1 1 1
n
VT = + +
2 a + bc b + ca c + ab
á

 
1 a b c
= + +
To

2 a2 + ab + bc + ca b2 + ab + bc + ca c2 + ab + bc + ca
ab + bc + ca
=
(a + b)(b + c)(c + a)
s
bộ

a2 b 2 c 2
=
(a2 + ab + bc + ca) (b2 + ab + bc + ca) (c2 + ab + bc + ca)
s
a2 b 2 c 2
c

=
(a2 + abc) (b2 + abc) (c2 + abc)
lạ

s
abc
= .
(a + bc)(b + ca)(c + ab)
u

2) Điều kiện xác định: 2x + y + 6 ≥ 0. Hệ đã cho có thể viết lại dưới dạng

(x + y)(2x + y) = 6 p
(x + y) + (2x + y) − 2 2x + y + (x + y)(2x + y) + 1 = 0

(x + y)(2x + y)p=6
⇐⇒
(x + y + 1) − 2 (x + y + 1)(2x + y) + (2x + y) = 0.

Từ trên suy ra (x+y +1)+(2x+y) và (x+y +1)(2x+y) đều không âm nên (x+y +1),
(2x + y) là các số không âm. Do đó, phương trình thứ hai của hệ có thể viết lại thành
134 Tuyển tập đề thi vào lớp 10 chuyên Toán

√ √
( x + y + 1 − 2x + y)2 = 0. Từ đây ta có ngay x + y + 1 = 2x + y hay x = 1.
Từ đó ta giải ra được x = 1, y = 1 hoặc x = 1, y = −4. Ta loại nghiệm (1, −4) vì
3 · 1 + 2 · (−4) + 1 < 0. Vậy hệ đã cho có đúng một nghiệm (x, y) là (1, 1).

Câu 2: (2.5 điểm)

iế n
1) Tìm tất cả các cặp số nguyên dương (x, y) thỏa mãn đẳng thức

(x + y)(5x + y)3 + xy 3 = (5x + y)3 + x2 y 3 + xy 4 .

K
2) Với a, b, c là những số thực dương thoả mãn các điều kiện sau


(

-T
c ≤ b ≤ a ≤ 3, b2 + 2a ≤ 10, b2 + 2a + 2c ≤ 14,
(a2 + 1)(b2 + 1) + 4ab ≤ 2a3 + 2b3 + 2a + 2b.

Tìm giá trị lớn nhất của biểu thức A1


P = 4a2 + b4 + 2b2 + 4c2 .
n

Lời giải.
á

1) Ta viết lại phương trình thành (5x + y)3 (x + y − 1) − xy 3 (x + y − 1) = 0, tức là


To

(x + y − 1)[(5x + y)3 − xy 3 ] = 0. Vì x, y nguyên dương nên x + y − 1 > 0 nên


xy 3 = (5x + y)3 , suy ra (5x + y)3 chia hết cho y 3 dẫn đến 5x + y chia hết cho y.
 3
5x + y
bộ

Vì vậy x = là lập phương của một số nguyên dương. Đặt x = a3 , với


y
a nguyên dương. Ta suy ra ay = 5a3 + y tương đương với 5a3 = y(a − 1), do đó
5a3 = 5(a3 − 1) + 5 chia hết cho (a − 1) nên a − 1 là ước dương của 5. Dẫn đến
c

a ∈ {2, 6}.
lạ

Từ đó ta tìm được (x, y) = (8, 40), (216, 216).


u

2) Điều kiện thứ hai tương đương với


(a2 + 1 − 2b)(b2 + 1 − 2a) ≤ 0.

Tuy nhiên do a2 + 1 − 2b ≥ 2a − 2b ≥ 0 nên ta có b2 + 1 ≤ 2a.

Đặt x = 2a, y = b2 + 1, z = 2c. Ta suy ra x ≥ y từ nhận xét trên. Từ điều kiện


thứ nhất ta suy ra x + y = b2 + 2a + 1 ≤ 11, x + y + z = 2a + b2 + 1 + 2c ≤ 15,
x = 2a ≤ 6. Ta cũng có y = b2 + 1 ≥ 2b ≥ 2c = z, hay y ≥ z.
Nguyễn Tiến Lâm - Trịnh Huy Vũ 135

Ta có

P = 4a2 + (b2 + 1)2 + 4c2 − 1


= x2 + y 2 + z 2 − 1
= x(x − 6) + y(y − 5) + z(z − 4) + 6x + 5y + 4z − 1
= (x − y)(x − 6) + (y − z)(x + y − 11) + z(x + y + z − 15) + 4(x + y + z) + (x + y) + x − 1
≤ 0 + 0 + 0 + 4 · 15 + 11 + 6 − 1

iế n
= 76

Vậy Max P = 76 đạt được x = 6, y = 5, z = 4 hay a = 3, b = c = 2.

K
Câu 3: (3 điểm)


-T
Cho tam giác ABC nhọn, không cân, nội tiếp đường tròn (O). Điểm P nằm trong
tam giác ABC. Gọi E, F lần lượt là hình chiếu vuông góc của P trên các cạnh
CA, AB. Giả sử tứ giác BCEF nội tiếp trong đường tròn (K).

1) Chứng minh rằng AP vuông góc với BC.


A1
2) Chứng minh rằng AP = 2OK.

3) Đường thẳng qua P vuông góc với AP cắt đường tròn (O) tại hai điểm Q và
n
R. Chứng minh rằng đường tròn tâm A bán kính AP tiếp xúc với đường tròn
ngoại tiếp tam giác KQR.
á
To

X
bộ

A
t
c

D
lạ

J
E
F
u

Q R
P

K
B C

T
136 Tuyển tập đề thi vào lớp 10 chuyên Toán

Lời giải.
1) Để ý rằng các tứ giác AEP F và BCEF là các tứ giác nội tiếp nên ta được P[
AB =
◦ ◦ ◦
P AF = 90 − AP F = 90 − AEF = 90 − ABC. Từ đó suy ra AP ⊥ BC.
[ [ [ [

2) Trước hết, ta có OK là đường trung trực của BC, vì thế OK ⊥ BC và kéo theo
[ = 180◦ −AOC
OK k AP . Để ý rằng OAC
\
= 90◦ − ABC
[ = 90◦ − AEF [ , ta thu được
2
OA ⊥ EF . Mặt khác, gọi J là trung điểm của AP . Khi đó, vì J, K tương ứng là tâm

iế n
của các đường tròn (AEP F ) và (BCEF ) nên JK ⊥ EF , từ đó suy ra JK k AO.
Kết hợp AP k OK đã chứng minh ở trên ta suy ra AJKO là hình bình hành. Vì vậy
OK = AJ = 21 AP hay AP = 2OK.

K
3) Dựng đường kính AT của (O), vì OJ là đường trung bình của 4AP T nên P T k OJ.
Do đó, theo tiên đề Euclid thì P, K, T thẳng hàng và hơn nữa P T = 2OJ = 2P K nên


K cũng là trung điểm P T . Gọi D là giao điểm thứ hai của P T với đường tròn (O).

-T
Khi đó, vì AT là đường kính của (O) nên ADP\ = ADT [ = 90◦ .
Gọi X là điểm đối xứng với P qua D. Khi đó, vì ADP\ = 90◦ nên 4AXP cân tại A,
kéo theo AX = AP hay X ∈ (A, AP ). Mặt khác, ta có P X · P K = 2P D · P K =
P D · 2P K = P D · P T = P Q · P R, kéo theo tứ giác XQKR là tứ giác nội tiếp. Như
vậy X ∈ (KQR).
A1
Dựng tia tiếp tuyến Xt của (A, AP ), khi đó Xt ⊥ AX. Để ý rằng vì OK k AP, AP ⊥
QR nên OK ⊥ QR, dẫn đến OK chính là trung trực của QR. Như vậy KQ = KR,
\ 4XQP ∼ 4XKR và dẫn đến XP
kéo theo XK là phân giác của QXR, \ Q = XRK.
\
n

Từ đó ta được KXt
[ =P [ Xt = 90◦ − AXP
\ = 90◦ − AP \ X = XP
\ Q = XRK.
\ Như vậy,
á

KXt
[ = KRX,\ từ đó kéo theo Xt cũng là một tiếp tuyến của (KQR). Do vậy, (A, AP )
To

tiếp xúc với (KQR) tại điểm X.

Câu 4: (1 điểm)
bộ

Cho các điểm A1 , A2 , ..., A30 theo thứ tự nằm trên một đường thẳng sao cho độ
dài các đoạn Ai Ai+1 bằng k (đơn vị dài), với k = 1, 2, ..., 29. Ta tô màu mỗi đoạn
thẳng A1 A2 , A2 A3 , ..., A29 A30 bởi một trong ba màu (mỗi đoạn được tô bởi đúng
c

một màu). Chứng minh rằng với mọi cách tô màu, ta luôn chọn được hai số nguyên
lạ

dương 1 ≤ i ≤ j ≤ 29 sao cho hai đoạn Ai Ai+1 , Aj Aj+1 được tô cùng màu và i − j
là bình phương của một số nguyên dương.
u

Lời giải. Giả sử phản chứng, gọi các màu tương ứng là 1, 2, 3. Giả sử đoạn thẳng Ai Ai+1

được tô màu ai với ai ∈ {1, 2, 3}. Thế thì

• a1 , a10 , a26 đôi một phân biệt;

• a1 , a17 , a26 đôi một phân biệt;

Suy ra a10 = a17 = x1 . Chứng minh tương tự ta có

a11 = a18 = x2 , a12 = a19 = x3 , a13 = a20 = x4 .


Nguyễn Tiến Lâm - Trịnh Huy Vũ 137

Do 19 − 10 = 9 là số chính phương nên x1 6= x3 . Tương tự thì x1 6= x2 , x2 6= x3 . Suy ra


x1 , x2 , x3 đôi một khác nhau. Tương tự thì x2 , x3 , x4 đôi một khác nhau. Do đó, x1 = x4
hay a13 = a17 nhưng 17 − 13 = 4 là số chính phương nên ta có điều vô lí. Vậy điều giả sử
là sai và do đó ta có điều phải chứng minh.

iế n
K

-T
A1
á n
To
bộ
c
lạ
u

138 Tuyển tập đề thi vào lớp 10 chuyên Toán

2.22. Đề thi tuyển sinh vào lớp 10 trường THPT


chuyên Sư phạm Hà Nội năm 2022, vòng 2

Câu 1: (2,5 điểm)


p
3

a) Không sử dụng máy tính, hãy tính giá trị của biểu thức P = 7+5 2+

iế n
p
3
7 − 5 2.

b) Cho đa thức P (x) = ax2 + bx + c. Chứng minh rằng nếu P (x) nhận giá trị

K
nguyên với mỗi số nguyên x thì ba số 2a, a + b, c đều là những số nguyên. Sau
đó, chứng tỏ rằng nếu 2a, a + b, c là những số nguyên thì P (x) cũng nhận giá
trị nguyên với mỗi số nguyên x.


-T
Lời giải.
a) Sử dụng hằng đẳng thức (a + b)3 = a3 + b3 + 3ab(a + b), ta có
3
q √ √
P = 14 + 3 (7 + 5 2)(7 − 5 2).P
A1
kéo theo
P 3 + 3P − 14 = 0.
Phân tích nhân tử phương trình trên ta được (P − 2)(P 2 + 2P + 7) = 0. Lưu ý là
P 2 + 2P + 7 = (P + 1)2 + 6 > 0 nên ta phải có P = 2 là số nguyên.
á n

b) Vì P (x) ∈ Z với mọi x ∈ Z nên P (0), P (1), P (−1) là các số nguyên. Suy ra c, a +
b + c, a − b + c là các số nguyên. Từ đó a + b, a − b cũng nguyên. Suy ra 2a =
To

(a + b) + (a − b) ∈ Z. Vậy, 2a, a + b, c là các số nguyên.


Đảo lại nếu 2a, a + b, c nguyên thì ta viết
x(x − 1)
bộ

P (x) = 2a. + (a + b)x + c


2
và lưu ý rằng x(x − 1) là tích hai số nguyên liên tiếp nên x(x − 1) chia hết cho 2.
Từ đây thì P (x) ∈ Z với mọi x nguyên.
c
lạ

Câu 2: (3,0 điểm)

Cho tam giác đều ABC ngoại tiếp đường tròn (O). Cung nhỏ OB của đường tròn
u

ngoại tiếp tam giác tam giác OBC cắt đường tròn (O) tại điểm E. Tia BE cắt

đường tròn (O) tại điểm thứ hai F .

a) Chứng minh tia EO là tia phân giác của góc CEF.

b) Chứng minh tứ giác ABOF nội tiếp.

c) Gọi D là giao điểm thứ hai của CE với đường tròn (O). Chứng minh ba điểm
A, F, D thẳng hàng.
Nguyễn Tiến Lâm - Trịnh Huy Vũ 139

iế n
O

K
E
D
C


B

-T
Lời giải.
a) Để ý rằng O vừa là tâm nội tiếp, vừa là tâm ngoại tiếp của tam giác đều ABC. Vì
thế OB = OC hay 4OBC cân tại O. Từ đó, để ý rằng tứ giác OEBC là tứ giác nội
tiếp nên OEC
[ = OBC
\ = OCB \ = OEF [ . Vì thế, tia EO là tia phân giác của CEF
[.
A1
b) Vì AO, CO tương ứng là các tia phân giác của BAC,
[ ACB[ nên OAB[ = 1 BAC[ = 30◦
2
\ = 1 ACB
và OCB [ = 30◦ . Mặt khác, vì 4OEF cân tại O và tứ giác OEBC nội tiếp
2
nên ta thu được OF
\ B = OF
[ E = OEF
[ = OCB \ = 30◦ . Như vậy, OF
\ [ = 30◦ ,
B = OAB
n

vì vậy tứ giác AF OB nội tiếp.


á

c) Vì 4OEF và 4OED cân tại O và OEF [ = OED \ nên ta được EOF [ = EOD \ và
To

4OEF = 4OED. Như vậy, EF = ED hay 4DEF cân tại E. Hơn nữa, DEF \ =
180◦ − BEC
\ = 180◦ − BOC \ = 180◦ − 120◦ = 60◦ . Như vậy, 4DEF là tam giác đều
và do đó DF\ E = 60◦ . Mặt khác, ta lại có AF
[ [ = 120◦ (vì tứ giác ABOF
B = AOB
bộ

nội tiếp). Do vậy, AF


\ D = AF
[ B + DF
\ E = 120◦ + 60◦ = 180◦ và từ đó suy ra A, F, D
thẳng hàng.
c

Câu 3: (2,0 điểm)


lạ

Cho a, b, c, d là các số nguyên dương thoả mãn ab = cd. Chứng minh rằng
u

N = a2022 + b2022 + c2022 + d2022 là hợp số.


Lời giải. Đây là bài toán quen thuộc và có nhiều cách giải. Từ giả thiết, ta suy ra
a d
=
c b
a b
và ta viết hai phân số , dưới dạng phân số tối giản, tức là
c d
a d x
= = ,
c b y
140 Tuyển tập đề thi vào lớp 10 chuyên Toán

trong đó x, y là các số nguyên dương nguyên tố cùng nhau. Suy ra tồn tại các số nguyên
dương m, n sao cho a = mx, c = my, d = nx, b = ny. Từ đó
N = (mx)2022 + (ny)2022 + (my)2022 + (nx)2022 = (m2022 + n2022 )(x2022 + y 2022 ).
Lưu ý là do m, n, x, y nguyên dương nên m2022 + n2022 > 1 và x2022 + y 2022 > 1, kéo theo
ngay N là hợp số.

iế n
Câu 4: (2,0 điểm)

Ta viết 10 số 0, 1, 2, . . . , 9 vào mười ô tròn trong hình bên, mỗi số được viết đúng

K
một lần. Sau đó ta tính tổng ba số trên mỗi đoạn thẳng để nhận được 6 tổng. Có
hay không một cách viết 10 số như thế sao cho 6 tổng nhận được là bằng nhau.


-T
A1
á n
To

Lời giải. Phản chứng, giả sử có một cách viết thoả mãn đề bài. Gọi số ở tâm và ở 3
đỉnh của tam giác đều lần lượt là a, b, c, d. Ta xét các số hạng trong 6 tổng được nhắc đến
trong đề bài, ta thấy các số a, b, c, d mỗi số xuất hiện đúng 3 lần trong 6 tổng này, còn 6
số còn lại mỗi số xuất hiện một lần. Khi đó nếu gọi giá trị của 6 tổng này là x ∈ N, ta
bộ

cộng vế cả 6 tổng lại thì có


3(a + b + c + d) + Tổng 6 số còn lại = 6x
Mà tổng của cả 10 số bằng 0 + 1 + · · · + 9 = 45 nên ta có
c
lạ

2(a + b + c + d) + 45 = 6x,
vô lý do vế trái là số lẻ còn vế phải là số chẵn.
u

Câu 5: (1,0 điểm)


a) Trong mặt phẳng cho 5 điểm sao cho không có ba điểm nào thẳng hàng.
Chứng minh rằng tồn tại ít nhất một tam giác tù có các đỉnh được lấy từ 5
điểm đã cho.

b) Trong mặt phẳng cho 2022 điểm sao cho không có ba điểm nào thẳng hàng.
Chứng minh rằng tồn tại ít nhất 2018 tam giác tù mà mỗi tam giác tù đó có
các đỉnh được lấy từ 2022 điểm đã cho.
Nguyễn Tiến Lâm - Trịnh Huy Vũ 141

Lời giải.
B A
C
A D

E
C
D E
E B

iế n
A
B C
D

K
a) Gọi 5 điểm đã cho là A, B, C, D, E. Xét bao lồi của 5 điểm này. Có các khả năng
sau đây


• Bao lồi là một ngũ giác lồi, giả sử là ABCDE. Thế thì tổng các góc trong

-T
ngũ giác lồi này bằng 540◦ nên trong các góc này tồn tại một góc lớn hơn 90◦ .
Không mất tính tổng quát giả sử EAB[ > 90◦ , tức là tam giác EAB là tam
giác tù.
• Bao lồi là một tứ giác lồi, giả sử là tứ giác ABCD. Nếu tồn tại một đỉnh của
A1
tứ giác mà góc ở đỉnh đó là góc tù, giả sử giả sử DAB \ > 90◦ , tức là tam
giác DAB là tam giác tù. Nếu không thì tất cả các góc của tứ giác này đều
bằng 90◦ , vì nếu không sẽ có một góc lớn hơn 90◦ do tổng các góc trong tứ
giác bằng 360◦ . Khi đó tứ giác ABCD là hình chữ nhật. Điểm E nằm trong
n

tứ giác ABCD và gọi α = max{AEB, [ BEC,\ CED,


\ DEA} \ thì do tổng bốn
◦ ◦
góc này bằng 360 nên ta có ngay α ≥ 90 . Không mất tính tổng quát, giả sử
á

[ Nếu α = 90◦ thì AEB


α = AEB. [ = BEC \ = CED \ = DEA \ = 90◦ . Nhưng khi
To

ấy thì các điểm A, E, C lại thẳng hàng, trái với giả thiết nên α > 90◦ thì tam
giác AEB là tam giác tù.
• Bao lồi là một tam giác ABC. Khi đó hai điểm D, E nằm trong tam giác. Ta
bộ

thấy ADB
\ + BDC\ + CDA \ = 360◦ nên trong ba góc ADB,
\ BDC, \ CDA \ có ít
nhất một góc lớn hơn 90◦ . Không mất tính tổng quát, giả sử là ADB
\ > 90◦
thì tam giác ADB là tam giác tù.
c

Tóm lại trong mọi trường hợp, đều tồn tại ít nhất một tam giác tù.
lạ

b) Ta chứng minh bằng quy nạp nếu có n ≥ 5 điểm thoả mãn thì sẽ tạo ra được n − 4
tam giác tù có các đỉnh các là các điểm đã cho. Với n = 5, đã chứng minh ở câu a).
u

Giả sử khẳng định đúng đến n − 1, ta chứng minh khẳng định cũng đúng với n. Xét

n điểm A1 , A2 , ..., An . Xét 5 điểm đầu tiên A1 , A2 , A3 , A4 , A5 ta tìm được một tam
giác tù, giả sử là A1 A2 A3 với đỉnh A1 tù. Bỏ đi điểm A1 , trong n − 1 điểm còn lại
thì theo giả thiết quy nạp ta tìm được ít nhất n − 5 tam giác tù và các tam giác tù
này có các đỉnh thuộc tập {A2 , A3 , ..., An } không trùng với tam giác tù đỉnh A1 . Do
đó, có ít nhất n − 4 tam giác tù. Theo nguyên lý quy nạp, khẳng định được chứng
minh. Áp dụng bài toán với n = 2022 ta thu được điều phải chứng minh.
142 Tuyển tập đề thi vào lớp 10 chuyên Toán

2.23. Đề thi tuyển sinh vào lớp 10 chuyên Toán


Hà Nội, năm 2022

Câu 1: (2.0 điểm)



1. Giải phương trình x2 − 4x + 2 2x − 1 + 1 = 0.

iế n
2. Cho các số thực a, b và c thỏa mãn điều kiện ab + bc + ca = 1. Tính giá trị của
biểu thức
a b c 2
P = + + −

K
1+a 2 1+b 2 1+c 2 a + b + c − abc


Lời giải.

-T
1
1. Điều kiện xác định: x ≥ .
2
Phương trình đã cho tương đương với

x2 − 4x + 2 2x − 1 + 1 = 0
A1 √
⇔ x2 − 2x + 1 = 2x − 1 − 2 2x − 1 + 1
√ 2
⇔ (x − 1)2 = 2x − 1 − 1 .
n
Tới đây có hai trường hợp
√ √
á

• Trường hợp 1. x − 1 = 2x − 1 − 1, khi đó ta có x = 2x − 1 tương đương với


x2 − 2x + 1 = 0, vậy x = 1 và nghiệm này thoả mãn.
To

√ √
• Trường hợp 2. 1 − x = 2x − 1 − 1, khi đó ta có 2 − x = 2x − 1, với điều kiện
x ≤ 2, bình phương ta có x2 − 4x + 4 = 2x − 1 ⇔ x2 − 6x + 5 = 0. Chú ý là x ≤ 2
nên ta tìm được x = 1.
bộ

Vậy phương trình đã cho có đúng một nghiệm x = 1.


2. Do ab + bc + ca = 1, ta có
c

a b c 2
lạ

P = + + −
1 + a2 1 + b2 1 + c2 a + b + c − abc
a b c 2
= 2 + 2 + 2 −
a + ab + bc + ca b + ab + bc + ca c + ab + bc + ca a + b + c − abc
u

a b c 2

= + +
(a + b)(a + c) (b + c)(b + a) (c + a)(c + b) a + b + c − abc
2 2
= − .
(a + b)(b + c)(c + a) a + b + c − abc

Lại có
(a + b)(b + c)(c + a) = (a + b + c)(ab + bc + ca) − abc
Mà do ab + bc + ca = 1, thay vào ta có P = 0.
Nguyễn Tiến Lâm - Trịnh Huy Vũ 143

Câu 2: (2.0 điểm)

1) Chứng minh rằng nếu n là số tự nhiên lẻ thì 32n+1 − 7 chia hết cho 20.

2) Tìm tất cả cặp số nguyên dương (x, y) sao cho y(x2 + x + 1) = (x + 1)(y 2 − 1)

Lời giải.

iế n
1) Ta có 32n+1 ≡ (−1)2n+1 ≡ −1 (mod 4). Do đó 32n+1 − 7 ≡ −1 − 7 ≡ 0 (mod 4)
Vì n lẻ, đặt n = 2a + 1 với a là số tự nhiên. Khi đó 32n+1 = 34a+3 = 81a .27 ≡ 1.2 ≡ 2

K
(mod 5), vì thế 32n+1 − 7 ≡ 0 (mod 5). Mặt khác (4, 5) = 1, do đó 32n+1 − 7 chia hết
cho 20.


2) Đặt (y, y 2 − 1) = d, khi đó y và y 2 − 1 đều chia hết cho d, dẫn đến 1 chia hết cho d,

-T
nên d = 1. Vậy (y, y 2 − 1) = 1. Tương tự ta có (x2 + x + 1, x + 1) = 1
Ta có y(x2 + x + 1) = (x + 1)(y 2 − 1) và x, y nguyên dương nên y(x2 + x + 1) chia hết
cho x + 1 và (x + 1)(y 2 − 1) chia hết cho y. Mà (x2 + x + 1, x + 1) = (y 2 − 1, y) = 1,
suy ra y chia hết cho x + 1 và x + 1 chia hết cho y, dẫn đến x + 1 = y. Vì vậy ta có
(
x+1=y
A1
, từ đây ta tìm được x = 1, y = 2.
y 2 − 1 = x2 + x + 1
n

Câu 3: (2.0 điểm)


á

m3 n3
To

1) Tìm hai số nguyên dương m, n sao cho và đều là các số nguyên


m+n m+n
tố.

2) Với a, b và c là các số thực không âm thoả mãn điều kiện a + b + c = 3, tìm


bộ

giá trị lớn nhất của biểu thức P = ab + 2bc + 3ca − 3abc.

Lời giải.
c

m3
lạ

1. Đặt = p là số nguyên tố thì m3 = p(m + n). Suy ra m3 chia hết cho p, kéo theo
m+n
m
m chia hết cho p. Do đó, m + n = .m2 . Suy ra m + n chia hết cho m, kéo theo n
p
u

chia hết cho m. Chứng minh tương tự thì m cũng chia hết cho n nên m = n. Suy ra

m3 m2
= = p là số nguyên tố. Vì m2 = 2p nên m2 chia hết cho 2 kéo theo m chia
m+n 2
hết cho 2. Suy ra m2 chia hết cho 4 và do đó p chia hết cho 2. Do p là số nguyên tố
nên p = 2 và từ đó tìm được m = n = 2.

2. Áp dụng bất đẳng thức AM-GM, ta có

(a + b + c)2 (a + b + c)2 27
P ≤ a(b + c) + 2c(a + b) ≤ +2 = .
4 4 4
144 Tuyển tập đề thi vào lớp 10 chuyên Toán

3 27
Dấu bằng xảy ra khi và chỉ khi a = c = , b = 0. Vậy giá trị lớn nhất của P là .
2 4

Câu 4: (3.0 điểm)

Cho tam giác ABC nhọn với AB < AC. Đường tròn (I) nội tiếp tam giác ABC,
tiếp xúc với ba cạnh BC, CA và AB lần lượt tại ba điểm D, E, F.

iế n
1) Gọi M là giao điểm của hai đường thẳng AI và DF. Chứng minh đường thẳng
CM vuông góc với đường thẳng AI.

K
2) Gọi N là giao điểm của hai đường thẳng AI và DE. Gọi K là trung điểm của
đoạn thẳng BC. Chứng minh tam giác KM N là tam giác cân.


3) Các tiếp tuyến tại M và N của đường tròn (K; KM ) cắt nhau tại điểm S.

-T
Chứng minh đường thẳng AS song song với đường thẳng ID.

A A1
E
n

F
I
á
To

J
N

B K C
S D
bộ

M
Q P
c

Lời giải.
lạ

[ = 90◦ + A . Suy ra
b
1. Ta có I là tâm đường tròn nội tiếp tam giác ABC nên AIC
2
u

[ = 90◦ − A . Tam giác BDF cân tại B nên BDF


\ = 90◦ − A . Do
b b
M
\ IC = 180◦ − AIC

2 2
đó, M
\ IC = BDF
\ = CDM\ . Từ đó suy ra tứ giác DICM là tứ giác nội tiếp. Vì thế
AM
\ C = IM
\ [ = 90◦ .
C = IDC
2. Gọi J là trung điểm của CI. Ta có JK là đường trung bình của tam giác IBC nên
JK k IB. Để ý, DM ≡ DF ⊥ IB, ta thu được JK ⊥ DM. Vì C, I, D, M cùng
thuộc một đường tròn đường kính CI nên J thuộc trung trực của DM. Suy ra JK
chính là trung trực của DM. Do đó KD = KM . Chứng minh tương tự ta cũng được
KD = KN . Vì vậy, KM = KN = KD.
Nguyễn Tiến Lâm - Trịnh Huy Vũ 145

3. Gọi P là giao điểm của M S và AC, còn Q là giao điểm của N S và AB. Để ý rằng K
chính là tâm ngoại tiếp của 4DM N , ta suy ra N
\ KD = 2N\M D = 2IM\ D = 2ICD
[=
[ Từ đó thu được KN k AC và kéo theo N S ⊥ AC. Chứng minh tương tự thì
ACB.
M S ⊥ AB. Chú ý rằng hai tam giác AN Q và AM P đồng dạng nên AQ AP
AN
= AM . Hai
AB AN AQ AB
tam giác AN B và AM C đồng dạng nên AC = AM . Suy ra AP = AC nên theo định lý
Thales đảo thì BC k P Q. Lại có trong tam giác AP Q có QS ⊥ AC, P S ⊥ AB nên S
là trực tâm tam giác AP Q. Suy ra AS ⊥ P Q. Mà BC k P Q nên AS ⊥ BC. Lại có

iế n
BC ⊥ ID nên AS k ID.

K
Câu 5: (1.0 điểm)

Cho tập hợp A gồm 70 số nguyên dương không vượt quá 90. Gọi B là tập hợp gồm


các số có dạng x + y với x, y ∈ A và x, y không nhất thiết phân biệt,

-T
1) Chứng minh 68 ∈ B.

2) Chứng minh B chứa 91 số nguyên liên tiếp.


A1
Lời giải. Ta có các nhận xét sau đây:
Nhận xét 1. Các số lẻ từ 43 tới 139 xuất hiện trong B.
Giả sử a lẻ và 43 ≤ a ≤ 139.  
a−1 a+1
n

+) Nếu a < 90 xét các cặp (1, a−1), (2, a−2), . . . , , và các số a, a+1, . . . , 90
2 2
á

a−1
thì có cặp và 91 − a số. Nếu a ∈ / B thì A chỉ chứa tối đa một số từ các cặp trên.
To

2
Suy ra số phần tử của A không vượt quá
a−1 181 − a 181 − 43
+ 91 − a = ≤ = 69,
2 2 2
bộ

mâu thuẫn với giả thiết.


+) Nếu a > 90. Xét các cặp
c

 
a−1 a+1
lạ

(a − 90, 90), (a − 89, 89), . . . , ,


2 2

và các số
u

1, 2, . . . , a − 91

181 − a
thì có tất cả là cặp và a − 91 số. Nếu a ∈
/ B thì A chỉ chứa tối đa một số từ các
2
cặp trên, suy ra số phần tử của A không vượt quá
181 − a a−1 139 − 1
+ a − 91 = ≤ = 69,
2 2 2
mâu thuẫn với giả thiết.
Từ các lập luận trên thì a ∈ B.
146 Tuyển tập đề thi vào lớp 10 chuyên Toán

Nhận xét 2. Các số chẵn từ 44 đến 138 đều thuộc B.


Xét a chẵn và 44 ≤ a ≤ 138.
+) nếu a ≤ 90 thì xét các cặp
 
a−2 a+2
(1, a − 1), (2, a − 2), . . . , ,
2 2
a a−2

iế n
và các số , a, . . . , 90 thì có cặp và 92 − a số. Nếu a ∈
/ B thì A chỉ chứa tối đa một
2 2
số trong mỗi cặp nên số phần tử của A không vượt quá

K
a−2 182 − a 182 − 44
+ 92 − a = ≤ = 69,
2 2 2
mâu thuẫn với giả thiết.


+) Nếu a > 90 thì tương tự xét các cặp

-T
 
a−2 a+2
(a − 90, 90), (a − 89, 89), . . . , ,
2 2

và các số
A1 a
1, 2, . . . , a − 91,
2
thì ta cũng chỉ ra được nếu a ∈
/ B thì số phần tử của A không vượt quá 69.
Như vậy các số lẻ từ 43 tới 139 và các số chẵn từ 44 đến 138 đều thuộc B.
á n

1. Theo chứng minh trên thì hiển nhiên 68 ∈ B.


To

2. Các số từ 43 tới 139 thuộc B và đây là 97 số tự nhiên liên tiếp.


bộ
c
lạ
u

Nguyễn Tiến Lâm - Trịnh Huy Vũ 147

2.24. Đề thi tuyển sinh vào lớp 10 chuyên Tin Hà


Nội, năm 2022

Câu 1: (2.0 điểm)


p
1) Giải phương trình x2 − 2x + 2 = (x2 + 4)(x + 1).

iế n
2) Với a, b và c là các số thực dương thỏa mãn abc = 3, tính giá trị của biểu thức
1 1 1
P = + 2 + 2 .

K
a2 (b + c) + 3 b (c + a) + 3 c (a + b) + 3


Lời giải.

-T
1. Điều kiện xác định là x ≥ −1. Đặt x2 + 4 = a2 và x + 1 = b2 với a > 0 và b ≥ 0, ta có
phương trình đã cho tương đương
p
(x2 + 4) − 2 (x + 1) − (x2 + 4) (x + 1) = 0,
a2 − 2b2 − ab = 0,
A1
(a − 2b)(a + b) = 0.
Lưu ý là a + b > 0, ta suy ra a = 2b, hay
x2 + 4 = 4 (x + 1),
á n

tương đương với


x (x − 4) = 0.
To

Giải ta tìm được x = 0, 4 đều thoả mãn điều kiện bài toán. Vậy phương trình có hai
nghiệm x = 0 và x = 4.
bộ

2. Ta có
1 1 1
P = + 2 + 2
a2 (b
+ c) + 3 b (c + a) + 3 c (a + b) + 3
c

1 1 1
= 2 + 2 + 2
a (b + c) + abc b (c + a) + abc c (a + b) + abc
lạ

bc ca ab
= + +
abc (ab + bc + ca) abc (ab + bc + ca) abc (ab + bc + ca)
u

bc + ca + ab 1 1
= = = .

abc (ab + bc + ca) abc 3

Câu 2: (2.0 điểm)

1) Với p là số nguyên tố lớn hơn 3 , chứng minh số A = 5p + p2 chia hết cho 6.

2) Tìm tất cả cặp số nguyên (x, y) thỏa mãn x3 − x2 y + 2x = 5x2 − 2y − 1.


148 Tuyển tập đề thi vào lớp 10 chuyên Toán

Lời giải.
1. Vì p là số nguyên tố lớn hơn 3 nên p không chia hết cho 3 và p lẻ. Vì 5p và p2 đều lẻ
nên 5p + p2 chia hết cho 2.
Lại có 5p ≡ (−1)p ≡ −1 (mod 3) (vì p lẻ) và p2 là số chính phương không chia hết cho
3 nên p2 ≡ 1 (mod 3). Từ đây ta suy ra 5p + p2 chia hết cho 3.
Chú ý (2, 3) = 1, từ các lý luận trên ta suy ra A chia hết cho 6.

iế n
2. Ta có x3 − 5x2 + 2x + 1 = y(x2 − 2). Vì x là số nguyên nên x2 6= 2. Khi đó

x3 − 5x2 + 2x + 1 x(x2 − 2) − 5(x2 − 2) + 4x − 9 4x − 9

K
y= 2
= 2
=x−5+ 2
x −2 x −2 x −2

là số nguyên. Suy ra 4x − 9 chia hết cho x2 − 2, kéo theo (4x + 9)(4x − 9) = 16x2 − 81 =


16(x2 − 2) − 49 chia hết cho x2 − 2. Từ đây ta tìm được x2 ∈ {1, 9}. Thử các trường

-T
hợp của x, khi đó (x, y) là (−1, 7); (1, 1); (−3, −11).

Câu 3: (2.0 điểm)


A1
1) Với a, b, c là các số thực dương thỏa mãn abc = 2, chứng minh
9
a2 + b2 + c2 − 3 (a + b − c) ≥ − .
n
4
á

2) Tìm tất cả các số nguyên dương a, b và c sao cho các phương trình x2 − 2ax +
To

b = 0, x2 − 2bx + c = 0 và x2 − 2cx + a = 0 đều có nghiệm là các số nguyên


dương
bộ

Lời giải.
1. Bất đẳng thức cần chứng minh tương đương với
c

 2
2 4 3
lạ

a − 3a + + b − c − ≥ 0.
a 2

4 (a − 2)2 (a + 1)
u

Chú ý là a2 − 3a + = ≥ 0 nên bất đẳng thức trên luôn đúng. Dấu


a a

1
bằng xảy ra khi và chỉ khi a = b = 2, c = .
2
2. Ta có biệt thức ∆0 của các phương trình tương ứng là a2 − b, b2 − c, c2 − a. Phương
trình muốn có nghiệm nguyên dương thì các biệt thức đó phải là các số chính phương.
Không mất tính tổng quát có thể giả sử a = max(a, b, c). Ta có a2 − b < a2 nên
a2 − b ≤ (a − 1)2 từ đó 2a ≤ b + 1 ≤ a + 1 hay a ≤ 1. Vì a nguyên dương nên a = 1,
kéo theo a = b = c = 1. Kiểm tra lại thì các giá trị này thoả mãn. Vậy, a = b = c = 1.
Nguyễn Tiến Lâm - Trịnh Huy Vũ 149

Câu 4: (3.0 điểm)

Cho tam giác ABC với AB < AC nội tiếp đường tròn (O). Ba đường cao AD, BE
và CF của tam giác ABC cùng đi qua điểm H. Gọi I và K lần lượt là trung điểm
của EF và BC.
AI HI
1) Chứng minh = .

iế n
AK HK
2) Chứng minh đường thẳng AH là tiếp tuyến của đường tròn ngoại tiếp tam
giác HIK.

K
3) Gọi P là chân đường vuông góc kẻ từ điểm H đến đường thẳng EF. Chứng
minh rằng đường thẳng DP song song với đường thẳng AI.


-T
A

A1
E

G I
O
P
n

F
á

H
To

L B D C
K
bộ

Lời giải.
c

1. Dễ thấy hai tam giác AEF và ABC đồng dạng theo trường hợp góc-góc và AI, AK
lạ

là hai trung tuyến tương ứng nên hai tam giác AEI và ABK đồng dạng. Suy ra
AI EI EF
= = .
u

AK BK BC

Chứng minh tương tự thì


HI EF
=
HK BC
nên
AI HI
= .
AK HK
2. Ta có tam giác HEF đồng dạng với tam giác HCB do tứ giác EF BC nội tiếp. Suy ra
hai tam giác HEI và HKC đồng dạng. Do B, C, E, F cùng thuộc đường tròn đường
150 Tuyển tập đề thi vào lớp 10 chuyên Toán

kính BC nên K thuộc trung trực của EF. Suy ra IK ⊥ EF, dẫn tới

HIK [ − 90◦ = HKC


[ = HIE \ − 90◦ = KHD.
\

Suy ra HD là tiếp tuyến của đường tròn ngoại tiếp tam giác HIK, tức là HA là tiếp
tuyến của đường tròn ngoại tiếp tam giác HIK.

3. Gọi L là giao điểm của EF và BC, còn G là giao điểm của AL và đường tròn ngoại

iế n
tiếp tam giác ABC, với G 6= A. Kẻ đường kính AV của đường tròn ngoại tiếp tam
giác ABC.

K
Ta có LE · LF = LB · LC = LA · LG nên AGF E là tứ giác nội tiếp. Do đó G thuộc
\ = 90◦ . Vì AGV
đường tròn đường kính AH nên AGH [ = 90◦ nên G, H, V thẳng hàng.
Do V, H đối xứng qua K nên G, H, K thẳng hàng. Suy ra HK ⊥ AL. Mà AH ⊥ KL


nên H là trực tâm tam giác AKL. Do đó, LH ⊥ AK.

-T
Vì tam giác AKB đồng dạng với tam giác AIE nên ta có

IP
[ D = 90◦ + HP
\ D = 90◦ + HLD
\ = 90◦ + 90◦ − AKL
[
= 180◦ − AKL
[ = 180◦ − AKB
\ = 180◦ − AIE
[ = AIP
[.
A1
Từ đây suy ra DP k AI.
n

Câu 5: (1.0 điểm)


á

Trên bảng có hai số tự nhiên m và n . An và Bình chơi một trò chơi như sau : Mỗi
To

lượt chơi, một bạn chọn một trong hai số trên bảng để xóa và viết lên bảng một
số mới là hiệu không âm của số vừa xóa với một ước số tự nhiên bất kỳ của số
vừa xóa. Hai bạn luân phiên thực hiện lượt chơi. Bạn đầu tiên không thể thực hiện
bộ

được lượt chơi của mình là người thua cuộc , người còn lại là người thắng cuộc .
Biết rằng An là người thực hiện lượt chơi đầu tiên.

1) Với m = 2022, n = 2023 hãy chỉ ra chiến thuật chơi của An để An là người
c

thắng cuộc.
lạ

2) Vơi m = 2022, n = 1981 hãy chỉ ra chiến thuật chơi của An để An là người
thắng cuộc.
u

Lời giải.
1. Bước đầu tiên , An chọn số 2023, xóa số đó đi , và viết vào bảng số 2022 (chú ý
2022=2023-1). Trên bảng lúc này có hai số 2022. Sau đó, ở các bước tiếp theo, nếu
Bình thực hiện bất cứ tác động nào lên 1 số trên bảng thì thì An chỉ cần làm giống
như Bình đối với số còn lại. Như vậy, An luôn bảo đảm sau mỗi lượt chơi của An thì
trên bảng luôn có hai số bằng nhau. Các bước diễn ra như vậy, khi đó An luôn có thể
thực hiện lượt chơi của mình, nên An không thể thua, tức là đến một thời điểm nào
đó, Bình sẽ thua cuộc.
Nguyễn Tiến Lâm - Trịnh Huy Vũ 151

2. Đầu tiên An chọn số 2022 , xóa nó đi và viết thêm vào bảng số 2021. Khi đó trên bảng
có hai số lẻ. Chú ý mọi ước tự nhiên của 1 số lẻ luôn là số lẻ, vì thế sau lượt chơi của
Bình thì trên bảng có 2 số gồm 1 số chẵn và 1 số lẻ. giả sử số chẵn này lớn hơn 0, bởi
nếu số chẵn này bằng 0 thì An sẽ thắng ở ngay lượt chơi tiếp theo. Khi đó, ở lượt tiếp
theo, An sẽ xóa số chẵn đó đi và thay bởi hiệu của số chẵn đó và 1. Như vậy, sau lượt
chơi của An , trên bảng luôn có hai số lẻ. Ở đây nếu Bình tạo ra trạng thái, sau lượt
chơi của Bình thì trên bảng có một số 0 và một số lẻ lớn hơn 1 thì ngay lập tức An

iế n
sẽ thắng cuộc ở bước tiếp theo. Trường hợp còn lại , quá trình trên tiếp tục diễn ra
đến khi sau lượt chơi của An trên bảng chỉ còn 2 số 1 và 1. Tại lượt chơi tiếp theo của
Bình thì trên bảng có 2 số 1 và 0. Và An sẽ thắng cuộc sau lượt chơi tiếp theo.

K

-T
A1
á n
To
bộ
c
lạ
u

152 Tuyển tập đề thi vào lớp 10 chuyên Toán

2.25. Đề thi tuyển sinh vào lớp 10 chuyên Toán


Thành phố Hồ Chí Minh, năm 2022

Câu 1
p
Cho x, y là hai số thực thỏa mãn xy + (1 + x2 ) (1 + y 2 ) = 1. Tính giá trị của biểu

iế n
thức  p  √ 
M = x+ 1+y 2 y+ 1+x . 2

K
Lời giải. Ta có:
 p 2
(1 + x2 ) (1 + y 2 ) = 12


xy +

-T
p
⇔ 2x2 y 2 + x2 + y 2 + 2xy (1 + x2 ) (1 + y 2 ) = 0
 p √ 2
⇔ x 1 + y 2 + y 1 + x2 = 0
p √
⇔ x 1 + y 2 + y 1 + x2 = 0
A1
Từ đó suy ra M = 1.

Câu 2
n


a) Giải phương trình x + 4 + |x| = x2 − x − 4.
á

x

To


 = 2x − 1
y + z


y
b) Giải hệ phương trình = 3y − 1
z+x

 z
= 5z − 1.

bộ


x+y

Lời giải.
c

a) ĐKXĐ: x > −4.


lạ

• Nếu −4 6 x < 0, khi đó ta có



x + 4 − x = x2 − x − 4
u


⇔ x + 4 = x2 − 4


⇔ x + 4 + x + 4 = x2 + x
 √  √ 
⇔ x− x+4 x+ x+4+1 =0

TH1: x = x + 4; khi đó x > 0, loại.

TH2: x + 4 = −1 − x; khi đó −4 6 x 6 −1. Khi đó√bình phương lên ta được
−1 − 13
x2 + x − 3 = 0. Ta giải ra và nhận nghiệm x = . (Thỏa mãn ĐKXĐ)
2
Nguyễn Tiến Lâm - Trịnh Huy Vũ 153

• Nếu x > 0, khi đó


x + 4 = x2 − 2x − 4

⇔ x + 4 + x + 4 = x2 − x
 √  √ 
⇔ x+ x+4 x− x+4−1 =0

iế n

TH1: x + 4 = −x, khi đó x 6 0, loại.
√ 2
TH2: x − 1 = x + 4. Khi √ đó x > 1. Bình phương lên ta được x − 3x − 3 = 0.

K
3 + 21
Giải ra ta được x = . (Thỏa mãn ĐKXĐ)
2


( √ √ )

-T
−1 − 13 3 + 21
Vậy S = ; .
2 2

b) Từ hệ phương trình ta có được 2x(y + z) = 3y(z + x) = 5z(x + y) = x + y + z; và


A1
x, y, z khác 0.

Từ 2x(y + z) = 3y(z + x), suy ra 2xz = 3yz + xy.


n

Lại từ 3y(z + x) = 5z(x + y), suy ra 3xy = 5xz + 2yz.


á

Từ hai điều trên ta có xz = 11yz, lại do z khác 0 nên x = 11y. Khi đó 22y(y+z) = 60yz,
To

từ đó ta cũng có được 22y = 38z hay 11y = 19z. Từ đó

x = 11y = 19z.
bộ

 
19z 239 239 239
Khi đó = 38z − 1. Giải ra ta được (x, y, z) = , , .
19z 60 660 1140
+z
c

11
lạ
u

Câu 3

Cho hình vuông ABCD. Trên các cạnh BC, CD lần lượt lấy các điểm M và N sao
cho M
\ AN = 45◦ .

a) Chứng minh rằng M N tiếp xúc với đường tròn tâm A bán kính AB.

b) Kẻ M P k AN (P ∈ AB) và kẻ N Q k AM (Q ∈ AD). Chứng minh rằng


AP = AQ.
154 Tuyển tập đề thi vào lớp 10 chuyên Toán

A P B

iế n
Q
E

K
F D N C


-T
Lời giải.
a) Gọi E là hình chiếu của A trên M N . Đường thẳng đi qua A vuông góc với AM cắt
đường thẳng CD tại F . Ta có AB = AD và BAM \ = DAF
A1 \ = (90◦ − DAM \ ) nên
4ABM = 4ADF (g.c.g) và kéo theo AM = AF . Hơn nữa lại để ý rằng M \ AN =
N
\ AF (= 45◦ ) nên ta được 4AM N = 4AF N (c.g.c). Từ đó suy ra AN
\ E = AN
\ D và
kéo theo 4ADN = 4AEN (ch.gn). Do đó AB = AD = AE hay E ∈ (A, AB). Từ
AE ⊥ M N tại E, ta thu được M N tiếp xúc với (A, AB) tại E.
n

b) Để ý M P k AN nên AN M P là hình thang. Mặt khác, vì AN M = AN D=N AP nên


á

\ \ \
AN M P là hình thang cân, vì thế AP = M N . Chứng minh tương tự như trên ta cũng
To

có AM N Q là hình thang cân, kéo theo AQ = M N . Như vậy AP = AQ(= M N ).


bộ

Câu 4
Cho ba số thực dương a, b, c thỏa a + b + c = 3.

a) Chứng minh rằng ab + bc + ca ≤ 3.


c
lạ

a b c
b) Tìm giá trị nhỏ nhất của biểu thức P = + + .
b 2 + 1 c 2 + 1 a2 + 1
u

Lời giải.
1
a) Ta có (a + b + c)2 − 3(ab + bc + ca) = [(a − b)2 + (b − c)2 + (c − a)2 ] ≥ 0, vì thế
2

(a + b + c)2
ab + bc + ca ≤ = 3.
3

Đẳng thức xảy ra khi a = b = c = 1.


Nguyễn Tiến Lâm - Trịnh Huy Vũ 155

b) Theo bất đẳng thức AM-GM thì b2 + 1 ≥ 2b. Khi đó

a a(b2 + 1) − ab2 ab2 ab2 ab


2
= 2
= a − 2
≥ a − =a− .
b +1 b +1 b +1 2b 2
b bc c ca
Chứng minh tương tự ta có ≥b− và 2 ≥c− .
c2+1 2 a +1 2

iế n
ab + bc + ca
Vì thế P ≥ (a + b + c) − . Mặt khác a + b + c = 3, ab + bc + ca ≤ 3 nên
3
3 3
P ≥ 3 − = . Đẳng thức xảy ra khi a = b = c = 1.
2 2

K
3
Vậy giá trị nhỏ nhất của P là đạt được khi a = b = c = 1.
2


-T
Câu 5
Cho tam giác ABC nhọn (AB < AC) có các đường cao AD, BE, CF cắt nhau tại
H. Đường thẳng EF cắt đường thẳng BC tại I. Đường thẳng qua A vuông góc với
IH tại K và cắt BC tại M .
A1
BI CI
a) Chứng minh rằng tứ giác IKF C nội tiếp và BD
= CD
.

b) Chứng minh M là trung điểm của BC.


á n

A
To

E
bộ

F K
c

H
lạ

I B D M C
u

Lời giải.
a) Vì AKI [ = 90◦ nên tứ giác AKDI nội tiếp. Vì thế KIC
[ = ADI [ = KID
[ = KAD\ =

KAH.
\ Ngoài ra ta cũng có AF
\ H = AKH
\ = 90 nên tứ giác AF HK nội tiếp, vì thế
KF
\ C = KF
\ H = KAH
\ = KIC.
[ Do đó tứ giác IKF C nội tiếp.
IB EC F A
Áp dụng định lý Menelaus cho 4ABC với I, E, F thẳng hàng ta suy ra · ·
IC EA F B
= 1.
DB EC F A
Áp dụng định lý Ceva cho 4ABC với AD, BE, CF đồng quy ta suy ra · ·
DC EA F B
= 1.
Như vậy ta thu được IB
IC
= DB
DC
BI
hay BD CI
= CD .
156 Tuyển tập đề thi vào lớp 10 chuyên Toán

b) Để ý rằng bốn điểm E, F, H, K cùng nằm trên đường tròn đường kính AH và hơn
nữa tứ giác HKM D nội tiếp. Do đó, ID · IM = IH · IK = IE · IF . Từ đó ta được
tứ giác DM EF nội tiếp. Vì thế ta được M
\ FC = M
\ F E − CF
[ E=M \ DE − HF
\ E=
BAC − HAE = HAF = M CF . Từ đó 4M F C cân tại M nên M F = M C. Hơn nữa
[ \ \ \
ta cũng có M
\ BF = 90◦ − M
\ CF = 90◦ − M \FC = M\F B nên 4M F B cân tại M . Như
vậy M B = M F = M C.

iế n
Câu 6
Số nguyên dương n được gọi là "số tốt" nếu n + 1 và 8n + 1 đều là các số chính

K
phương.

a) Hãy chỉ ra ví dụ ba "số tốt" lần lượt có 1, 2, 3 chữ số.


-T
b) Tìm các số nguyên k thỏa mãn |k| ≤ 10 và 4n + k là hợp số với mọi n là "số
tốt".

Lời giải. A1
a) Ta có 3 là "số tốt" nhỏ nhất cũng là "số tốt" có 1 chữ số.
15 là "số tốt" có 2 chữ số, vì 15 + 1 = 16 là số chính phương và 8 · 15 + 1 = 121 là số
chính phương.
120 là "số tốt" có 3 chữ số vì 120+1 = 121 là số chính phương và 8·120+1 = 961 = 312
n

là số chính phương.
á

b) Vì 3 "số tốt" nhỏ nhất nên với n là "số tốt" thì 4n + k ≥ 12 + k.


To

Xét n là một "số tốt" bất kỳ. Vì số chính phương chia cho 3 chỉ có thể dư 0 hoặc 1,
mà n + 1, 8n + 1 là số chính phương nên n không thể chia cho 3 dư 1, 2. Vì vậy n phải
chia hết cho 3.
bộ

Với k ∈ {−8, −6, −4, −2, 0, 2, 4, 6, 8, 10} thì 4n + k luôn là số nguyên lớn hơn 2 và chia
hết hết cho 2 với mọi n là "số tốt", nên 4n + k là hợp số với mọi n là số tốt.
Với k ∈ {−3, 0, 3, 9} thì 4n + k luôn là số nguyên lớn hơn 3 và chia hết cho 3 với mọi
c

n là số tốt, nên 4n + k là hợp số với mọi n là "số tốt" .


lạ

Với k = −10, khi n = 3 thì 4n + k = 2 là số nguyên tố nên k = −10 loại.


Với k = −9, khi n = 3 thì 4n + k = 3 là số nguyên tố nên k = −9 loại.
Với k = −7, khi n = 3 thì 4n + k = 5 là số nguyên tố nên k = −7 loại.
u

Với k = −5, khi n = 3 thì 4n + k = 7 là số nguyên tố nên k = −5 loại.


Với k = −1, khi n = 3 thì 4n + k = 11 là số nguyên tố nên k = −1 loại.


Với k = 1, khi n = 3 thì 4n + k = 13 là số nguyên tố nên k = 1 loại.
Với k = 5, khi n = 3 thì 4n + k = 17 là số nguyên tố nên k = 5 loại.
Với k = 7 khi n = 3 thì 4n + k = 19 là số nguyên tố nên k = 7 loại.
Vậy k ∈ {−8, −6, −4, −3, −2, 0, 2, 3, 4, 6, 8, 9, 10}.

You might also like